You are on page 1of 185

EMLE Trial Exams December 2020 (1000 Qs) Version (1.

0) 22/12/2020

EMLE Trial Exams


December 2020
Exams 1000 Qs
Version (1.0)
22/12/2020
If you have any comment or suggestions don’t hesitate to contact us through:

emle.question@gmail.com
Or
https://www.facebook.com/emle.questions
(*) For every time repetition.
(#) For repeated question for pervious exams.
(^) For questions with incomplete sentences or same idea – choices.

1
EMLE Trial Exams December 2020 (1000 Qs) Version (1.0) 22/12/2020

Contents
Exam 1 ..................................... 3
Exam 2 ................................... 22
Exam 3 ................................... 41
Exam 4 ................................... 59
Exam 5 ................................... 78
Exam 6 ................................... 96
Exam 7 ................................. 114
Exam 8 ................................. 132
Exam 9 ................................. 150
Exam 10 ............................... 168

2
EMLE Trial Exams December 2020 (1000 Qs) Version (1.0) 22/12/2020

Exam 1
1) All of the following are findings in iron deficiency anemia except:*
a) Pallor and easy fatigability
b) Large hepatosplenomegaly
c) Hypochromic microcytic anemia
d) Decreased serum ferritin
e) Increased total iron binding capacity
The answer is: Large hepatosplenomegaly
2) A 74-year-old woman presents to the emergency department complain-ing of dizziness and tiredness.
On direct questioning she tells you she has recently been having black stools. She is currently awaiting a
total hip replacement for arthritis. Which of the following would be most useful in establishing the
pri-mary cause of her symptoms?*
a) Barium meal
b) Coagulation screen
c) Faecal occult blood
d) Full blood count
e) Oesophago-gastro-duodenoscopy
The answer is: Oesophago-gastro-duodenoscopy
3) In acute laryngotracheobronchitis (croup), all are true except:
a) Is usually caused by viral infection
b) Is usually preceded by other upper respiratory symptoms
c) May benefit from corticosteroid therapy
d) Restlessness is an indication for sedation
e) You hear stridor
The answer is: Restlessness is an indication for sedation
4) A 40 year-old patient came to the clinic complaining of fatigue and pallor. CBC reveals HB is 9 g/dl, MCV
is low. Iron studies show high serum iron, high serum ferritin, high transferrin saturation and low TIBC.
The patient has a history of celiac disease. What is the diagnostic cell abnormality in the bone marrow?
a) Shistocytes
b) Ringed sideroblast
c) Spherocytes
d) Blast cells
e) Heniz bodies
The answer is: Ringed sideroblast
5) Bone age is advanced in:
a) Hypothyroidism
b) In those with poor social conditions
c) In congenital adrenal hyperplasia
d) In familial dwarfism
e) As an effect of long-term cortisone therapy
The answer is: In congenital adrenal hyperplasia
6) Stridor in the newborn may be caused by all except:
a) Laryngomalacia
b) Laryngeal web
c) Laryngeal paralysis (secondary to trauma)
d) Laryngeal papilloma in newborn

3
EMLE Trial Exams December 2020 (1000 Qs) Version (1.0) 22/12/2020

e) Bronchial asthma
The answer is: Bronchial asthma
7) Mechanism of action of copper IUCD includes:
a) Tubal block
b) Inhibition of ovulation
c) Increased tubal motility
d) Cervical mucus hostile to the sperms
e) Mechanical inflammatory reaction of the endometrium
The answer is: Mechanical inflammatory reaction of the endometrium
8) Conservative treatment of rupture kidney is indicated in:
a) Complete tear.
b) Deep tear.
c) lntraperitoneal rupture with minimal shock.
d) Retroperitoneal rupture with minimal shock.
e) non of the above.
The answer is: Deep tear.
9) A previously healthy, obese 26-year-old G 1 PO has a positive glucose challenge test at 28 weeks
gestational age (GA) . Follow-up testing confirms the diagnosis of gestational diab etes mellitus (GDM ) .
The patient asks what this diagnosis will mean for her baby. Which of the following can be a significant
consequence of poorly controlled GDM to the fetus?
a) Hyperglycemia
b) Macrosomia
c) Cataracts
d) Cerebral edema
e) Cephalohematoma
The answer is: Macrosomia
10) A 57-year-old man presents with hematemesis. Which of the following features would categorize him
into a high risk group?
a) A blood pressure of 100/70 mmHg
b) A history of ischemic heart disease.
c) A plasma glucose of 250 mg/dl
d) A pulse of 95 beats per minute
e) His age
The answer is: A history of ischemic heart disease.
11) Components of biophysical profile include all of the following, EXCEPT:
a) Fetal movement
b) Placental thickness
c) Fetal tone
d) Fetal breathing movement
e) Amniotic fluid volume assessment
The answer is: Placental thickness
12) A woman in early pregnancy is worried because of some recent discomfort in her left breast. On
examination, her skin appears normal, she has no axillary or clavicular adenopathy, but you palpate a
smooth, nontender, 2-cm mass. Your immediate management should be which of the following?
a) breast ultrasound
b) needle aspiration of the mass
c) excisional biopsies of the mass

4
EMLE Trial Exams December 2020 (1000 Qs) Version (1.0) 22/12/2020

d) mammography
e) warm compresses and antibiotics
The answer is: breast ultrasound
13) A 82 –year- old female with history of hypertension presents with altered mental status for 2 days . She
is feverish , pulse 110 bpm , RR 22 breath per minute , BP 85/45 mmHg , O2 sat 94%on room air . Urine
analysis shows >100 WBCs .Appropriate antibiotics are already started .Which of the following is the
most appropriate next step in her management ?
a) Intravenous fluid bolus 30ml /kg
b) Norepinephrine 0.5 mcg/kg /minute
c) Milrinone 0.5mcg /kg/minute
d) Dobutamine 5mcg /kg /minute
e) Dopamine 5 mcg /kg /minute
The answer is: Intravenous fluid bolus 30ml /kg
14) A 22-year-old patient presents with a hematocrit of 31% at 28 weeks’ gestation. Her mean corpuscular
volume (MCV) is 105, her mean corpuscular hemoglobin (MCH) is 33, and her mean corpuscular
hemoglobin concentration (MCHC) is 36. Serum iron is 100 mg/dL. There is no evidence of abnormal
bleeding.Which of the following is the best diagnosis?
a) normocytic, normochromic anemia
b) normal
c) macrocytic anemia
d) microcytic anemia
e) hemolysis
The answer is: macrocytic anemia
15) A 56-year – old male presents with generalized fatigue , weakness and vomiting . He tells you that he
has taken an overdose of his tricyclic antidepressant medication . His BP 150/90 mmHg , ECG
demonstrates a regular , wide-complex tachycardia . Which of the following is the most appropriate next
step in management?*
a) Cardioversion at 100 J
b) Lidocaine IV
c) Procainamide IV
d) Sodium Bicarbonate infusion
e) Propafenone
The answer is:: Sodium Bicarbonate infusion
16) 55-year-old male presents with severe headache after head trauma. CT showed accumulated blood in
biconvex pattern. Which is the most likely diagnosis?
a) Subdural hematoma
b) Epidural hematoma
c) Subarachnoid hemorrhage
d) Intracerebral hemorrhage
e) diffuse axonal injury
The answer is: Epidural hematoma
17) Which factor is most associated with an increased risk of going on to have a stroke in a patient
presented with TIA?*
a) Old age
b) Blood pressure > 140/90
c) Unilateral weakness
d) >60 minutes duration

5
EMLE Trial Exams December 2020 (1000 Qs) Version (1.0) 22/12/2020

e) Diabetes
The answer is: >60 minutes duration
18) A 3-year-old boy who weighs 16 kg is admitted for elective surgery and needs to be NPO overnight.
What is the appropriate maintenance fluid and maintenance fluid rate?
a) D5 . NS @ 52 mL/hr
b) D5W @ 52 mL/hr
c) D5 . NS @ 400 mL/hr
d) D5W @ 400 mL/hr
e) NS @ 20 cc/kg over 20 minutes
The answer is: D5 . NS @ 52 mL/hr
19) 24-year-old male ingests liquid drain cleaner and immediately presents to the ED. Which of the following
is the most appropriate treatment?
a) Ingestion of a small cup of water
b) Calcium gluconate
c) Dexamethasone
d) Ampicillin
e) Neutralization therapy with hydrochloric acid
The answer is: Ingestion of a small cup of water
20) Which of the following can cause acute seizures?#
a) Hypokalemia
b) Hyperkalemia
c) Hyponatremia
d) hypernatremia
e) C&D
The answer is: C&D
21) A 44-year-old patient is involved in a motor car accident and presented to have bruising all over the
chest. On examination he has a shallow tachypnoea with indrawing of the chest wall on inspiration.
SaO2 is 94 per cent on air, pulse 110/min, BP 100/65 mmHg.#
a) isolated rib fracture
b) aortic dissection
c) simple pneumothorax
d) flail chest
e) aortic dissection
The answer is: flail chest
22) All of the following are side effects of prolonged use of corticosteroids except:
a) Anemia
b) Growth retardation
c) Increased susceptibility to infections
d) Increased blood pressure
e) Muscle weakness
The answer is: Anemia
23) A 24-year-old married white woman was exposed to rubella at 7 to 8 weeks’ gestation. Several days
later she developed a red macular rash and had a rubella antibody titer of 1:160 when seen by you at 11
weeks’ gestation.Which of the following may be anticipated in an infant born to this mother?
a) rhagades
b) hepatosplenomegaly
c) trisomy 21

6
EMLE Trial Exams December 2020 (1000 Qs) Version (1.0) 22/12/2020

d) Hutchinson’s incisors
e) Cri du chat syndrome
The answer is: hepatosplenomegaly
24) Advantage of lower segment caesarean section over the classic incision includes:
a) Ease of repair
b) Decreases blood loss
c) Lower probability of subsequent uterine rupture
d) Decreases danger of intestinal obstruction
e) All of the above
The answer is: All of the above
25) A 53-year-old female with a high-grade non-Hodgkin lymphoma develops tumor lysis syndrome (TLS)
after undergoing induction chemotherapy. Which of the following substances will have a lower
concentration than normal upon laboratory testing?
a) Calcium
b) Uric acid
c) Phosphate
d) Iron
e) Lactate dehydrogenase
The answer is: Calcium
26) Infants who are smaller size and relatively underweight at birth because of maternal malnutrition have
increased rates of all the following EXCEPT
a) coronary heart disease
b) stroke
c) type 1 diabetes mellitus
d) obesity
e) osteoporosis
The answer is: type 1 diabetes mellitus:
27) A 27-year-old woman presented with a 5-year history of abdominal bloating, weight gain and
constipation, as well as recent anal discomfort with no bleeding. The bloating was eased by bowel
opening, which occurred every 5–7 days. Stool consistency varied from pellet-like to semi-formed. She
strained at stool on every occasion.What is the mostly likely finding on rectal examination?*
a) A tear in the anal skin is normal with chronic straining, and unlikely to be a cause of her pain
b) Contraction of the internal anal sphincter is normal on bearing down
c) Low resting anal pressure excludes an anal fissure
d) Rectal prolapse is possible despite her young age
e) The presence of stool in the rectum confirms the diagnosis of faecal impaction
The answer is: Rectal prolapse is possible despite her young age
28) Which of the following globulin chains is present in Hb F?
a) α2 β2
b) α2 γ2
c) α2 εc
d) ε4
e) β2
The answer is: α2 γ2
29) A 79-year- old female with six month history of rectal bleeding admitted to surgical ward to have a large
bowel tumour surgically removed .She appears to be severe ly short of breath and extremely tired
.Respiratory rate 24 bpm , Hb 6.8g/dL , pH 7.49 , Pco2 25mmHg , Po2 89mmHg , Bicarb 22mmol/L , SP02

7
EMLE Trial Exams December 2020 (1000 Qs) Version (1.0) 22/12/2020

99.8% , K 3.8mmol/L , Na 138mmol/L , Cl 96mmo/L , normal glucose , normal lactate and base excess.
Which of the following describe her acid base state ?*
a) Uncompensated respiratory alkalosis
b) Compensated respiratory alkalosis
c) Uncompensated metabolic alkalosis
d) Compensated metabolic alkalosis
e) Normal acid base state
The answer is: Uncompensated respiratory alkalosis
30) A 34-year-old woman (gravida 3, para 2) at 35 weeks’ gestation complains of sharp, excruciating pain in
the right flank radiating into her groin. No chills or fever have been noted. The pain resolved shortly
after the patient was seen. Urinary analysis reveals numerous red blood cells (RBCs), some WBCs, and
no bacteria. WBC and hematocrit are normal.Ureteral stones during pregnancy are rare. Which of the
following best characterizes them?
a) They are more likely to produce pain during pregnancy than in the nonpregnant state.
b) They are usually discovered during workup for vague abdominal pain.
c) They are associated with hyperparathyroidism.
d) They are frequently a cause of acute obstruction.
e) A prophylactic ureteral filter may need to be placed.
The answer is: They are associated with hyperparathyroidism.
31) A 67-year-old man who has a history of type 2 diabetes mellitus and benign prostatic hypertrophy
presents with burning pain in his feet. This has been present for the past few months and is getting
gradually worse. He has tried taking duloxetine but unfortunately has received no benefit. Clinical
examination is unremarkable other than diminished sensation to fine touch on both soles. What is the
most suitable initial management?
a) Carbamazepine
b) Amitriptyline
c) Pregabalin
d) Fluxetin
e) Na valproate
The answer is: Pregabalin
32) After being fire from his job, a 35-year- old man attempts suicide by drinking from a bottle labele
“insecticide.” Three hours later, (EMS) bring him into the (ED). He is extremely diaphoreti , drooling, and
vomiting. He is awake, but confused . His vital signs include a blood pressure (BP) of 170/90 mm Hg,
heart rate (HR) of 55 b/m, respiratory rate (RR) of 20 b/m, temperature of 37°C, an oxygen saturation of
95% on room air. Physical examination pinpoint pupils. You note crackles and wheezing on lung
examination. Which of the following is the most appropriate therapy?
a) Naloxone
b) N-acetylcysteine
c) Atropine
d) Flumazenil
e) Physostigmine
The answer is: Atropine
33) The mother of a minor consents to medication but the father refuses. What is the most appropriate
action to do?**
a) Write the prescription
b) Do not write the prescription
c) Postpone the prescription
d) Convince the daughter

8
EMLE Trial Exams December 2020 (1000 Qs) Version (1.0) 22/12/2020

e) Report the event to the police


The answer is: Write the prescription
34) For gluose-6-phosphate dehydrogenase deficiency, all are true except:
a) Sex-linked inheritance
b) Indication for splenectomy
c) Can manifest in girls
d) Cause fava beans induced hemolysis
e) Cause drug-induced hemolysis
The answer is: Indication for splenectomy
35) 35-year-old male presents with generalized weakness. His labs were as follows Na 136 (135 – 145), k 6.5
(3.5 – 5.3) Cr 1.8 (normal less than 1.2). Which is the most likely next step in management?#
a) IV fluids
b) Hemodialysis
c) Dobutamine
d) Insulin and Dextrose
e) Peritoneal dialysis
The answer is: Insulin and Dextrose
36) 55-year-old male presents with severe headache after head trauma. CT showed accumulated blood in
crescent pattern. Which is the most likely diagnosis?#
a) Subdural hematoma
b) Epidural hematoma
c) Subarachnoid hemorrhage
d) Intracerebral hemorrhage
e) diffuse axonal injury
The answer is: Subdural hematoma
37) The association between hydrocephalus and meningocele is diagnosed as:
a) Dandy Walker malformation
b) Arnold Chiari malformation
c) Obstruction at duct of Sylvius
d) Obliteration of foramina of Lustka and Magendi
e) Angioma of the choroid plexus
The answer is: Arnold Chiari malformation
38) Which of the following is the most common risk factor associated with aortic dissection ?*
a) Smoking
b) Atherosclerosis
c) Marfan syndrome
d) Hypertension
e) Bicuspid aortic valve
The answer is: Hypertension
39) A patient presents with diplopia. The following photo described what happened on asking him to move
both eyes to the left side. Which is the most likely diagnosis?
a) Right abducent palsy
b) Left abducent palsy
c) Right oculomotor palsy
d) Left oculomotor palsy
e) Right trochlear palsy
The answer is: Left abducent palsy

9
EMLE Trial Exams December 2020 (1000 Qs) Version (1.0) 22/12/2020

40) In rheumatic chorea, all are true except:


a) Movements ceases or decrease during sleep
b) It is a self-limited condition
c) It is associated with hypotonia
d) Is always accompanied by signs of active carditis, arthritis and elevate ESR
e) Is accompanied by emotional changes in the patient
The answer is: Is always accompanied by signs of active carditis, arthritis and elevate ESR
41) A 62-year-old man with hypertension, diabetes, and hyperlipidemia complains of symmetric numbness
and burning pain to bilateral feet. Pain is provoked by gentle touch. What is the MOST likely diagnosis?
a) Complex regional pain syndrome
b) Diabetic neuropathy
c) Fibromyalgia
d) Postherpetic neuralgia
e) Sciatica
The answer is: Diabetic neuropathy
42) A 48-year-old woman presents with a painless mass located in her left breast. Physical examination finds
a firm, nontender, 3-cm mass in the upper outer quadrant of her left breast. There was retraction of the
skin overlying this mass, and several enlarged lymph nodes were found in her left axilla. The mass was
resected and histologic sections revealed an invasive ductal carcinoma. Biopsies from her axillary lymph
nodes revealed the presence of metastatic disease to 4 of 18 examined axillary lymph nodes. The
expression of which one of the following by the tumor cells would best predict a response to therapy
with Trastuzumab?
a) BRCA1
b) Estrogen receptors
c) HER2/neu
d) Progesterone receptors
e) Urokinase plasminogen activator
The answer is: HER2/neu
43) A 49-year-old woman presents with progressive muscle weakness and fatigue. Physical examination
finds an obese adult woman with abdominal stria rubra and increased facial hair. The excess adipose
tissue is mainly distributed in her face, neck, and trunk. Laboratory evaluation finds increased plasma
levels of cortisol and glucose. Which of the following is the most likely diagnosis?
a) Addison’s disease
b) Bartter’s syndrome
c) Conn’s syndrome
d) Cushing’s syndrome
e) Schmidt’s syndrome
The answer is: Cushing’s syndrome
44) Previously healthy 5-day-old male who was born at home develops bruising and melena. The pregnancy,
delivery and postnatal course were unremarkable. The infant is breastfeeding vigorously every 2 hours.
Findings on physical examination are unremarkable except for several large bruises. Of the following,
the most likely cause of the bleeding is:#
a) Disseminated intravascular coagulation
b) Factor VIII deficiency hemophilia
c) Liver disease
d) Vitamin K deficiency
e) Von Willebrand disease

10
EMLE Trial Exams December 2020 (1000 Qs) Version (1.0) 22/12/2020

The answer is: Vitamin K deficiency


45) A 30-year-old primigravida presents at 34 weeks gestational age with blood pressure of 170/100 mmHg,
headache, epigastric pain, visual abnormalities and +3 proteinuria. Biophysical profile of the fetus is 8/8.
Which one of the following is the immediate response?#
a) Start magnesium sulfate intravenously.
b) Perform an emergency C-section.
c) Give betaclomethasone to induce fetal lung maturity.
d) Perform an amniocentesis to assess fetal lung maturity.
e) Repeat the biophysical profile daily.
The answer is: Start magnesium sulfate intravenously
46) A 31-year-old woman, gravida 2, para 1 presents with a positive home pregnancy test. She used to have
regular menstrual cycles every 28 days and lasting for 4 days. She did not use any contraception
previously. She remembered that the first day of her last menstrual period (LMP) was April 28th, 2020.
What is THE NEAREST estimate of expected date of delivery (EDD)?
a) January 3rd, 2021.
b) January 6th, 2021.
c) January 29th, 2021.
d) February 5th , 2021.
e) February 8th, 2021.
The answer is: February 5th , 2021.
47) A female child presenting with epistaxis and bleeding gums of a 2-day duration clinical examination
revealed scattered petechiae and ecchymosis over the upper and lower limbs. No pallor No
organomegaly. CBC showed platelet count 50.000/cmm. Hb 11 gm/dl. The most appropriate diagnosis is:
a) Henoch-Schönlein purpura
b) Immune thrombocytopenic purpura
c) Hemophilia A
d) thrombocytopenia
e) Von Willebrand disease
The answer is: Immune thrombocytopenic purpura
48) A 2-month-old infant presented with daily paroxysms of continuous crying for 3-5 hours starting after 6-
8 pm associated with facial grimacing, leg flexion, and passing flatus, on examination no abnormality
was detected apart from mild abdominal distension. Of the following, the MOST likely cause is
a) hunger
b) wet diaper
c) fussiness
d) colic
e) overfeeding
The answer is: colic
49) Thrombocytopenia is characteristic of the following disease except:#
a) Acute immune thrombocytopenic purpura (ITP)
b) Systemic lupus erythematosus (SLE)
c) Acute leukemia
d) Hemophilia
e) Aplastic anemia
The answer is: Hemophilia

11
EMLE Trial Exams December 2020 (1000 Qs) Version (1.0) 22/12/2020

50) Which of the following is the best treatment for a diabetic patient on regular metformin. However, he
came to the clinic complaining of recurrent attacks of nausea and vomiting and UGIT endoscopy reveals
decreased gastric emptying?#^
a) erythromycin
b) Insulin
c) omeprazole
d) pantoprazole
e) pregabalin
The answer is: erythromycin
51) A 10-year-old male presents with fever, diarrhea, pallor, and weakness. Renal function is abnormal and
the patient is anemic. A peripheral blood smear reveals schistocytes. Which of the following is the most
likely diagnosis?
a) Henoch–Schönlein purpura
b) Hemolytic uremic syndrome (HUS)
c) Disseminated intravascular coagulation (DIC)
d) Idiopathic thrombocytopenic purpura
e) Nephrotic syndrome
The answer is: Hemolytic uremic syndrome (HUS)
52) What is the most common cause of nonmaternal or indirect mortality?
a) domestic violence
b) amniotic fluid emboli
c) motor vehicle accidents
d) heart disease
e) asthma
The answer is: domestic violence
53) A 65 year-old diabetic and hypertensive came to the hospital complaining of bloody diarrhea.10 days
ago, she was admitted to the hospital complaining of lung abcess but now she improved. What is the
best initial investigation?
a) Blood film
b) Stool toxin
c) Stool microscopy
d) Urine analysis
e) CBC
The answer is: Stool toxin
54) Incompetent cervix
a) Is a cause for early pregnancy loss
b) Is best diagnosed by history
c) Is a cause for fetal congenital abnormalities
d) Is not encounted with uterine anomities
e) Can be treated with tocolytics
The answer is: Is best diagnosed by history
55) A 52-year-old man presents with vomiting and severe epigastric pain radiating to the back. Abdominal x-
ray shows a dilated proximal small bowel loop.
a) biliary colic
b) gallbladder carcinoma
c) acute pancreatitis
d) gallstone ileus

12
EMLE Trial Exams December 2020 (1000 Qs) Version (1.0) 22/12/2020

e) ascending cholangitis
The answer is: acute pancreatitis
56) A hernia located at the lateral aspect of the rectus abdominis muscle. What type of hernia is being
described?#
a) Direct inguinal hernia
b) Indirect inguinal hernia
c) Femoral hernia
d) Richter’s hernia
e) Spigelian hernia
The answer is: Spigelian hernia
57) A 33-year-old woman comes to the office due to anxiety; she is by her husband,The patient describes
having several panic attacks while driving over a suspension bridge,She suddenly felt faint and shaky
and to stop the car in the middle of the bridge and get assistance. The patient moved to the suburbs 7
months ago, requiring that she now drive to work. Since having the attacks. she has taken an alternate
route to work that requires her to drive miles out of her way to avoid bridges. She avoids traveling to
the city when possible and has anxiety for days beforehand if a planned trip necessitates taking a bridge.
Mental status examination shows an anxious mood and tense affect , Which of the following is the most
likely diagnosis?
a) Anxiety due to a general medical condition
b) Avoidant personality disorder
c) Generalized anxiety disorder
d) Panic disorder with agoraphobia
e) Specific phobia
The answer is: Specific phobia
58) A thin looking girl with gross muscle wasting of all 4 limbs, she is lying in a windswept position
(alternatively, she could be in a wheel chair with support). She is drooling. There is generalized
hypertonia with contractures and increased reflexes with bilateral positive Babinski sign.Differential
diagnosis:
a) Tetanus
b) Tetany
c) Spastic cerebral Palsy
d) Hypokalemia
e) Acute polio
The answer is: Spastic cerebral Palsy
59) Which of the following is the most common error leading to malpractice claim?*
a) Failure to diagnose
b) Improper performance of a procedure
c) Delay in treatment
d) Treatment without consent
e) Improper medical management
The answer is: Failure to diagnose
60) A 4-week-old infant is brought to the emergency department by his parents after he develops an
episode of emesis with an observed period of apnea. Three other infants were seen there earlier this
week with the flu. The infant is discharged home with instructions for flu management, but the parents
return with him later, reporting that he had another episode of apnea. The patient is further evaluated
and subsequently transferred to the children's hospital with the clinical diagnosis of apnea from
gastroesophageal reflux.Which bias caused this adverse event?*

13
EMLE Trial Exams December 2020 (1000 Qs) Version (1.0) 22/12/2020

a) Violation bias
b) Anchoring bias
c) Confirmation bias
d) Premature closure
e) Availability bias
The answer is: Availability bias
61) A 22-year- old female at 36 weeks of gestation is being treated with magnesium sulfate for pre-
eclampsia while awaiting transfer to a nearby hospital for definitive management . On arrival she was
quite somnolent with markedly depressed deep reflexes and decreased respiratory rate . After
managing the airway , the most reliable next step is intravenous ;*
a) dexamethasone
b) Lidocaine
c) Labetalol
d) Calcium gluconate
e) Atropine
The answer is: Calcium gluconate
62) 55-year-old male presents with severe headache after head trauma. CT showed accumulated blood in
biconvex pattern. Which is the source of bleeding for the most likely diagnosis?
a) Anterior cerebral artery
b) posterior cerebral artery
c) bridging veins
d) middle meningeal artery
e) Anterior communicating artery
The answer is: middle meningeal artery
63) Regarding cervical incompetence, all of the following are true, EXCEPT
a) Typically causes painful abortions
b) Typically causes mid-trimester abortions
c) Is treated by Shirodkar suture (cervical cerculage) which is best
d) preformed early in the second trimester
e) May lead to premature rupture of the membrane
The answer is: Typically causes painful abortions
64) Best method used to reduce nosocomial infection in NICU is:#
a) Handwashing
b) Wearing gloves
c) Wearing gown
d) Wearing shoe covers
e) Wearing mask
The answer is: Handwashing:
65) The major cause of the increased risk of morbidity & mortality among twin gestation is:
a) Gestational diabetes.
b) Placenta previa.
c) Malpresentation.
d) Preterm delivery.
e) Congenital anomalies.
The answer is: Preterm delivery.

14
EMLE Trial Exams December 2020 (1000 Qs) Version (1.0) 22/12/2020

66) A 32-year-old woman who is 16 weeks pregnant of BMI 22 kg/m2 . A 75g oral glucose tolerance test
revealed fasting and 2 hrs post prandial blood glucose of 100 and 220 mg/dl respectively. Which of the
following is the most appropriate step in the management of this patient?
a) Glipizide therapy
b) Insulin therapy
c) Low calorie diet
d) Metformin therapy
e) Repeat OGTT after 4 weeks
The answer is: Insulin therapy
67) Which of the following is the best treatment for a diabetic patient on regular metformin. However, he
came to the clinic with HbA1C of 7.7%?
a) Continue metformin only
b) Insulin lispro
c) Regular insulin
d) Glargine insulin
e) Glimepiride
The answer is: Glimepiride
68) What of the following is an absolute contraindication to thrombolytic in stroke?*^#
a) Active pancreatitis
b) Esophageal varices
c) Hemorrhagic diathesis
d) Major surgery in preceding 2 weeks
e) Pregnancy
The answer is: Esophageal varices
69) What is the definition of a ‘single use’ item?*
a) An item that can be used by a single patient, multiple times
b) An item that can only be used once before it needs disinfecting
c) An item that can only be used once before being disposed of
d) An item that can only be used for a single day
e) An item that can be used for only a one week
The answer is: An item that can only be used once before being disposed of
70) Healing by first intention means:
a) Using catgut.
b) Obtaining union between 2 edges of an incision without subsequent breakdown.
c) Immediate use of protective dressing.
d) Using staples.
e) A method whereby an ulcer heals
The answer is: Obtaining union between 2 edges of an incision without subsequent breakdown.
71) All the following organisms are capable of crossing the placenta and infecting the baby except:
a) CMV
b) Hepatitis B
c) Toxoplasmosis
d) Staphylococcal Aureus
e) Syphilis
The answer is: Staphylococcal Aureus

15
EMLE Trial Exams December 2020 (1000 Qs) Version (1.0) 22/12/2020

72) You are called by the radiologist who was performing an obstetric ultrasound. He has found extensive
hydroureter in a woman who is 26 weeks pregnant. She has mild right-sided pain that bothers her
during the day but does not wake her from sleep. What is the best treatment at this point?
a) intermittent bladder catheterization to keep the bladder as empty as possible
b) ureteral stent on the right
c) bed rest on her left side
d) increased fluid intake
e) no treatment
The answer is: no treatment
73) A 70-year-old cigarette smoker presents with a right inguinal mass that has enlarged and has caused
discomfort in recent months. He complains of recent difficulty with micturition and nocturia. 73-The
swelling, which does not extend to the scrotum, reduces when resting. What is the likely diagnosis?#
a) Direct inguinal hernia
b) Hydrocele
c) Aneurysm of the femoral artery
d) Cyst of the cord
e) Strangulated indirect inguinal hernia
The answer is: Direct inguinal hernia
74) A 36-year-old man who was hit by a car presents to the ER with hypotension. On examination, he has
tenderness and bruising over his left lateral chest below the nipple. An ultrasound examination is
performed and reveals free fluid in the abdomen. What is the most likely organ to have been injured in
this patient?
a) Liver
b) Kidney
c) Spleen
d) Intestine
e) Pancreas
The answer is: Spleen
75) The drug of choice for sedation in the setting of acute delirious state in critically ill patients is:*
a) Haloperidol
b) Diazepam
c) Diphenhydramine
d) Morphine
e) Promethazine
The answer is: Haloperidol
76) What is the first line investigation of a thyroid nodule in a patient with high TSH?
a) US thyroid
b) FNAC
c) CT neck
d) MRI pelvis
e) CXR
The answer is: FNAC
77) A 64-year-old male with a history of well-controlled, mild hypertension presents to the ED with a chief
complaint of an abrupt episode of “flashes of light” and floaters in his right eye. He states that he was
watching television when he experienced a brief episode in which many “glittery lights” flashed in his
right eye. The episode resolved, but he has subsequently noted a couple of gray spots floating in his

16
EMLE Trial Exams December 2020 (1000 Qs) Version (1.0) 22/12/2020

vision. He denies decreased visual acuity. Direct ophthalmoscopy is unrevealing. Which of the following
is the most likely cause of his symptoms?
a) Retinal detachment
b) Ocular migraine
c) Occipital lobe ischemia
d) Vitreous detachment
e) Optic neuritis
The answer is: Vitreous detachment
78) Which is the best statistical test for cross-sectional study analysis?#
a) Chi Square
b) Relative risk
c) Odds ratio
d) P value
e) Confidence interval
The answer is: Chi Square
79) In a patient with pneumothorax and circulatory collapse, the first action should be:
a) Immediate X-ray chest.
b) Oxygen inhalation.
c) Insertion of chest drain / needle.
d) Tracheostomy.
e) Putting the patient on ventilator
The answer is: Insertion of chest drain / needle.
80) A 63-year-old woman is admitted to the hospital with severe abdominal pain of 3-hour duration.
Abdominal examination reveals board-like rigidity, guarding, and rebound tenderness. Her blood
pressure is 90/50 mm Hg, pulse 110 bpm (beats per minute), and respiratory rate is 30 breaths per
minute. After a thorough history and physical, and initiation of fluid resuscitation, what diagnostic study
should be performed?*#
a) Supine abdominal x-rays
b) Upright chest x-ray
c) Gastrograffin swallow
d) Computerized axial tomography (CAT) scan of the abdomen
e) Abdominal sonogram
The answer is: Upright chest x-ray
81) A 45-year-old man with a history of gallstones presents in A&E with severeepigastric pain radiating to
the back and vomiting.
a) large bowel obstruction
b) acute pancreatitis
c) perforated viscus
d) appendicitis
e) small bowel obstruction
The answer is: acute pancreatitis
82) A 72-year-old woman presented with sudden severe persistent left lower abdominal pain since 2 days.
She had constipation since several years. In the past year, she had mild episodes of pain accompanied by
mild fever, which resolved spontaneously. Her temperature is 38.5 °C. Tenderness is noted on palpation
of the left lower quadrant of the abdomen with guarding and rebound. Bowel sounds are absent.
Complete blood count shows: 12,000 leukocytes/mm3, with 85% neutrophils. Which of the following is
the most appropriate next step in diagnosis?

17
EMLE Trial Exams December 2020 (1000 Qs) Version (1.0) 22/12/2020

a) Barium enema
b) Colonoscopy
c) Selective angiography
d) Intravenous pyelography
e) Plain abdominal x-ray
The answer is: Plain abdominal x-ray
83) Pathophysiologic consequences of preeclampsia include all of the following EXCEPT*
a) Decreased glomerular filtration rate
b) Increased plasma uric acid
c) Increased intravascular volume
d) Oliguria
e) Decreased placental perfusion
The answer is: Increased intravascular volume
84) A 60-year-old female is the restrained driver in a moderate speed motor vehicle collision. She is
complaining of chest wall pain after her chest hit the steering wheel. Her vital signs are within normal
limits and her only injury is a nondisplaced sternal fracture seen on chest CT scan. Her electrocardiogram
(ECG) is normal sinus rhythm with no abnormalities. Which of the following is the most appropriate
management plan?
a) Admit for 24 hours of observation while on telemetry.
b) Admit for an exercise stress test.
c) Discharge home with pain medication.
d) Perform an echocardiogram.
e) Perform two sets of cardiac enzymes and troponin 6 hours apart
The answer is: Discharge home with pain medication.
85) A 34-year-old woman is brought to the ED from a kitchen fire. Upon arrival, her BP is 110/75 mmHg, HR
is 115 b/m, RR is 18 b/m, and pulse oxygenation is 98% on room air. She weighs 70 kg. She has tender
burns with blisters to her bilateral forearms. What classification of burns does this patient have?
a) Superficial
b) First degree
c) Second degree
d) Full thickness
e) Fourth degree
The answer is: Second degree
86) The danger area of face where from infection can spread directly to cavernous sinus includes:
a) From below the eyes upto chin.
b) Area around the lips.
c) Area around lips including lower part of nose.
d) Whole of the face.
e) From maxillary sinus.
The answer is: Area around lips including lower part of nose.
87) A 65-year-old man with 8 days of progressive abdominal distension, colicky pains, reduced bowel
frequency, and 12 h of vomiting. Choose the single most appropriate first investigation:#
a) Barium enema (double contrast)
b) Colonoscopy
c) CT scan of the abdomen and pelvis
d) Endoanal ultrasound scan
e) Flexible sigmoidoscopy

18
EMLE Trial Exams December 2020 (1000 Qs) Version (1.0) 22/12/2020

The answer is: CT scan of the abdomen and pelvis


88) Which of the following is true regarding concussions?
a) Loss of consciousness is required to meet the technical definition.
b) CT brain is acutely abnormal in half of all cases.
c) MRI brain is almost always acutely abnormal.
d) Post concussive anosmia may be permanent.
e) On the field of play, simple orientation questions are adequate to detect concussion
The answer is: Post concussive anosmia may be permanent.
89) When you arrive at the scene, there is an incident commander (IC) who instructs you that there are
dozens of injured casualties and delegates you to triaging the wounded. You approach a patient who has
several abrasions on his torso, face, and extremi- ties. He has no spontaneous respirations. You perform
a chin lift and note chest wall movement and audible spontaneous respirations. Using the simple triage,
what color triage tag would you assign this patient?
a) Green
b) Yellow
c) Red
d) Blue
e) Black
The answer is: Red
90) Which of the following describes the principal of not doing harm to the patient?
a) Autonomy
b) Beneficence
c) Non-maleficence
d) Justice
e) Maleficence
The answer is: Non-maleficence
91) The following pregnant women are considered potentially diabetic EXCEPT:#^
a) Past history of macrosomia
b) Past history of eclampsia
c) Past history of congenital malformations
d) Past history of unexplained stillbirths
e) Positive family history of overt diabetes mellitus
The answer is: Past history of eclampsia
92) An 18-month-old male infant is noted to assume a squatting position frequently during playtime at the
daycare center. The mother also notices occasional episodes of perioral cyanosis during some squatting
periods. The day of admission, the child becomes restless, hyperpnoea and deeply cyanotic. The most
likely diagnosis is:
a) Cardiomyopathy
b) Tetralogy of Fallot
c) Cystic fibrosis
d) Anomalous coronary artery
e) Aspiration pneumonia
The answer is: Tetralogy of Fallot
93) 65 year old female with history of type 2 diabetes mellitus on insulin presents to the emergency room
with confusion and was dehydrated on examination. Blood glucose was 400 mg/dl (normal less than
200) and urine ketones was positive. Which is the initial step in management?#
a) IV normal Saline

19
EMLE Trial Exams December 2020 (1000 Qs) Version (1.0) 22/12/2020

b) IV dextrose 5%
c) Insulin
d) Potassium
e) Phosphorus
The answer is: IV normal Saline
94) Immediate complications of C-section include all the following, EXCEPT :
a) Complications of anesthesia.
b) Bladder injury.
c) Thromboembolism.
d) Colon injury.
e) Hemorrhage.
The answer is: Colon injury.
95) the natural defense of the vagina to infection includes all of the following EXCEPT:
a) The acidic vaginal pH
b) The presence of Doderlein’s bacilli
c) The physical apposition of the pudendal cleft and the vaginal walls
d) The physical apposition of the pudendal cleft and the vaginal walls
e) The vaginal stratified squamous epithelium
The answer is: The physical apposition of the pudendal cleft and the vaginal walls
96) Which of the following joints is most commonly affected in septic arthritis?
a) Hip
b) Knee
c) Ankle
d) Wrist
e) Shoulder
The answer is: Knee
97) A 65-year-old woman with a past medical history of hypertension and sta-ble angina presents to the GP
practice complaining of episodes of severe central abdominal pain. They occur around half an hour after
meals and can last up to an hour. She is becoming reluctant to eat because of the pain and as a result
she has lost nearly a stone in weight over the past 2 months. Examination is unremarkable. Which of
these investigations would be the most informative?#
a) Barium follow through
b) CT scan
c) Colonoscopy
d) Exercise tolerance test
e) Mesenteric angiography
The answer is: Mesenteric angiography
98) The following organisms cause a sever type of empyema complicated by dense adhesions:
a) Staph
b) Streptococci
c) Pneumococci
d) E.Coli
e) pseudomonus aerogenosa
The answer is: Pneumococci
99) A 63-year-old woman is admitted to the hospital with severe abdominal pain of 3-hour duration.
Abdominal examination reveals board-like rigidity, guarding, and rebound tenderness. Her blood
pressure is 90/50 mm Hg, pulse 110 bpm (beats per minute), and respiratory rate is 30 breaths per

20
EMLE Trial Exams December 2020 (1000 Qs) Version (1.0) 22/12/2020

minute. After a thorough history and physical, and initiation of fluid resuscitation, what diagnostic study
should be performed?*#
a) Supine abdominal x-rays
b) Upright chest x-ray
c) Gastrograffin swallow
d) Computerized axial tomography (CAT) scan of the abdomen
e) Abdominal sonogram
The answer is: Upright chest x-ray
100) A normally fit and healthy 68-year-old man presented to the Emergency Department after a sudden
collapse in the street with sudden onset back pain. On examination he was hypotensive and a palpable
pulsatile expansile epigastric mass was found, and a diagnosis of ruptured abdominal aortic aneurysm
was made. What is the treatment for ruptured AAA?*
a) Do nothing
b) High flow oxygen
c) Open surgical repair
d) IV opioid analgesia
e) IV fluids
The answer is: Open surgical repair

21
EMLE Trial Exams December 2020 (1000 Qs) Version (1.0) 22/12/2020

Exam 2
1) A 68 – year-old male with a history of chronic obstructive pulmonary disease (COPD) presents to the
emergency department with infective exacerbation. He experienced similar condition 6 months ago and
was intubated and added (I do not ever want to be intubated). You discuss the use of non-invasive positive
pressure ventilation (NIPPV) and he is agreeable. With which of the following comorbidities is this type of
ventilation is most appropriate?
a) Excessive secretions
b) Decreased sensorium
c) Hypertension
d) Midfacial trauma
e) Uncooperative patient
The answer is: Hypertension
2) In a patient with refractory hypotension after blunt trauma, which of the following findings is the primary
utility of the focused assessment with sonography in trauma (FAST) scan?*
a) Pelvic fracture
b) Renal injury
c) Diaphragmatic rupture
d) Hemoperitoneum
e) Aortic injury
The answer is: Hemoperitoneum
3) A 21-year-old female patient arrives with her older sister after having hit her head on a night out. The
patient's sister is being openly flirtatious with you and asks for your phone number. You are aware you
feel attracted to her as well. What is the most appropriate action in this scenario?**
a) Excuse yourself for a moment, while you ask a registrar colleague for advice
b) Politely decline to give her your number, explaining it would be inappropriate to do
c) Give her your number as she is not your patient
d) Explain you cannot give her your number whilst you are working, but arrange to meet another time
outside of the hospital
e) Refuse to give her your phone number and ask hospital security to remove her from the hospital
premises
The answer is: Politely decline to give her your number, explaining it would be inappropriate to do
4) A 55-year-old man complains of weight loss and upper abdominal pain of several months ‘duration. The
pain radiates to the mid-back. On physical examination, the man appears emaciated, with mild jaundice.
The liver edge is palpable and smooth. Which of the following tests is the most appropriate next step in
diagnosis?
a) Arteriography
b) Barium enema
c) Colonoscopy
d) CT scan of abdomen
e) Plain X-ray abdomen
The answer is: CT scan of abdomen
5) What is the most common cause of thyrotoxicosis?
a) De Quervian thyroiditis
b) Riedel thyroiditis
c) Hashimoto thyroiditis
d) Graves’ disease
e) Amiodarone

22
EMLE Trial Exams December 2020 (1000 Qs) Version (1.0) 22/12/2020

The answer is: Graves’ disease


6) A 41-year-old lawyer presents with a 3-month history of recurrent headaches which feel like a tight band
around the head. What is the cause of his headache?
a) Giant cell arteritis
b) Sub-arachinoid hemorrhage
c) Tension headache
d) Cluster headache
e) Migraine
The answer is: Tension headache
7) A 75-year-old man presents to the Emergency Department with sudden onset frank haematuria. Five days
earlier he required catheterization following an episode of postoperative urinary retention after an
elective inguinal hernia repair. The catheter was removed on discharge and he has passed urine freely
since. On examination, the urine is frank haematuria with evidence of blood clots. What is the most likely
diagnosis?*
a) Benign prostatic hypertrophy
b) Urethral trauma
c) Urinary tract infection
d) Transitional cell carcinoma
e) Renal calculi
The answer is: Urinary tract infection
8) Which of the following is the most common error leading to malpractice claim?*
a) Failure to diagnose
b) Improper performance of a procedure
c) Delay in treatment
d) Treatment without consent
e) Improper medical management
The answer is: Failure to diagnose
9) A patient at 34 weeks’ gestation develops marked pruritus especially on her palms and soles, and mildly
elevated liver function tests and elevated bile acids. Which of the following diagnostic possibilities is most
consistent with the clinical presentation?
a) pancreatitis
b) hyperthyroidism
c) diabetes insipidus
d) cholestasis of pregnancy
e) progesterone allergy
The answer is: cholestasis of pregnancy
10) Regarding the postoperative period:
a) Preoperative colonic preparation pre disposes to postoperative hypokalemia
b) There is hormone mediated excessive salt and water losses which need extra compensation
c) DVT can occur in the pelvic veins after pelvic surgery
d) A telectasis starts at the apical segments of the lungs
e) Reaction aryhemorrhage after abdominal operation is expected at the third postoperative day
The answer is: DVT can occur in the pelvic veins after pelvic surgery
11) All of these drugs can be used as tocolytic to stop labor, EXCEPT:*^
a) Salbutamol (ventolin)
b) Methotrexate
c) Calcium channel blocker

23
EMLE Trial Exams December 2020 (1000 Qs) Version (1.0) 22/12/2020

d) Indomethacin non-steroidal anti-inflammatory drugs Ritodrine (_- agonist)


e) none of the above
The answer is: Methotrexate
12) Multiple gestations should be suspected in all of the following condition EXCEPT:
a) Maternal weight gain is greater than expected.
b) The uterus is larger than expected.
c) Maternal AFP is elevated.
d) Pregnancy has occurred after induction with Gonadotropins.
e) Maternal perception of fetal movement occurs earlier than expected in gestation.
The answer is: Maternal perception of fetal movement occurs earlier than expected in gestation.
13) Which of these liver diseases results from a copper disorder?
a) Hemochromatosis
b) Gaucher’s disease
c) Biliary cirrhosis
d) Wilson’ s disease
e) Type III glycogen storage disease
The answer is: Wilson’ s disease
14) A 26-year-old woman with a 8-month history of weight loss, anorexia, recurrent abdominal pain after
eating, and bloody diarrhoea presents to A&E. The pain is colicky associated with vomiting, absolute
constipation, and abdominal distension over the past 3 days. Basic observations on arrival are: pulse
122/min, BP 85/58 mmHg, temperature 38.7°C. she is tender in the central abdominal region without
guarding or rebound. Blood tests: Hb 9.1 g/dL, WCC 22.0×10¹°/L, CRP 175, Alb 2.6.
a) adhesions
b) strangulated inguinal hernia
c) small bowel atresia
d) Crohn’s disease
e) irritable bowel syndrome
The answer is: Crohn’s disease
15) A 30-year-old woman is referred to the on-call surgical team with a 1-week history of right iliac fossa pain,
anorexia and fever. On examination, she has a palpable tender mass is in the right iliac fossa. Her
observations are: temperature 38.6°C, heart rate 110 beats/min, BP 125/70 mmHg. Which of the following
is the most likely diagnosis?*
Appendiceal mass
Carcinoma of caecum
Diverticular abscess
Ovarian cyst
Iliac artery aneurysm
The answer is: Appendiceal mass
16) What is the target HbA1c in a 54-year-old patient with a 12-year history of DM?
a) < 6.5
b) < 7
c) < 7.5
d) < 8
e) < 9
The answer is: < 7

24
EMLE Trial Exams December 2020 (1000 Qs) Version (1.0) 22/12/2020

17) A 69-year-old man who is known to have Alzheimer's disease is reviewed in clinic. His latest Mini Mental
State Examination (MMSE) score is 18 out of 30. What is the most appropriate management?#
a) Supportive care + memantine
b) Supportive care + trial of citalopram
c) Supportive care+ donepezil
d) Supportive care+ donepezil+ low dose aspirin
e) None of the above
The answer is: Supportive care+ donepezil
18) A 55-year-old male presents with a seizure. He has never had a seizure before and developed tonic–clonic
movements for 1 minute followed by a postictal confusional state lasting about 10 minutes. Physical
examination is normal, and the patient is amnestic to the event. Blood glucose is normal. Which of the
following is the most appropriate next step in management?
a) MRI brain
b) CT brain
c) Phenobarbital loading dose
d) Lumbar puncture
e) Prolactin level
The answer is: CT brain
19) Transplacental infection occur with all, EXCEPT:
a) Cytomegalovirus.
b) Toxoplasma
c) Rubella
d) Syphilis
e) Gonorrhea
The answer is: Gonorrhea
20) What is the definition of a ‘single use’ item?*
a) An item that can be used by a single patient, multiple times
b) An item that can only be used once before it needs disinfecting
c) An item that can only be used once before being disposed of
d) An item that can only be used for a single day
e) An item that can be used for only a one week
The answer is: An item that can only be used once before being disposed of
21) A 65-year-old woman presents with episodes of lower abdominal pain and intermittent vaginal bleeding.
On examination, there is a palpable mass in the lower abdomen. Which of the following tumour markers
would be associated with this presentation?
a) Alpha fetoprotein
b) Beta-hCG
c) CA 125
d) Calcitonin
e) Carcinoembryonic antigen
The answer is: CA 125
22) A normotensive 28 -year-old G2P 1 0 0 1 completes the third stage of labor 27 minutes after delivery of a
3 ,280-g male infant. The labor was spontaneous and lasted 8 hours. The second stage of labor lasts 3 5
minutes. The nurse reports that the uterus is boggy and 6 em above the umbilicus. When massaged, a clot
of approximately 1 2 5 mL is expressed. In the prevention of postpartum hemorrhage (PPH), the
medication employed as the first -line therapy is:
a) Carboprost tromethamine

25
EMLE Trial Exams December 2020 (1000 Qs) Version (1.0) 22/12/2020

b) Methylergonovine
c) Misoprostol
d) Oxytocin
e) Prostaglandin E 1
The answer is: Oxytocin
23) A 22-years female pregnant at 9 weeks gestation with proven intrauterine pregnancy presents to the ED
with severe nausea and vomiting for 3 days. She has been unable to eat and can drink only minimal fluids.
Which of the following suggests a diagnosis of hyperemesis gravidarum?
a) Bilious vomiting
b) Hypokalemia
c) Metabolic alkalosis
d) Ketonuria
e) White race
The answer is: Ketonuria
24) Which of the following is not a natural barrier to infection?*
a) Intact epithelial surface
b) High gastric pH
c) Antibodies
d) Natural killer cells
e) Macrophages.
The answer is: High gastric pH
25) A 19-year-old man is brought into the trauma room by EMS after a head-on cycling accident. The patient
was not wearing a helmet. Upon presentation his BP is 1 5/75 mm Hg, HR is 105 beats/minute, RR is 19
breaths/minute, and oxygen saturation is 100% on 10 liters of oxygen via facemask. His eyes are closed
but he opens them to verbal command. He also moves his arms and legs on command. When you ask him
questions, he is disoriented but able to converse. What is the patient’s GCS score?
a) 11
b) 12
c) 13
d) 14
e) 15
The answer is: 13
26) An incidental lump is found during inguinal hernia repair. It is benign and left alone.
a) maldescended testis
b) femoral aneurysm
c) hydrocoele
d) lipoma of the cord
e) inguinal hernia
The answer is: lipoma of the cord
27) Obstetrical complications of diabetes mellitus DO NOT include the following:#
a) Congenital fetal malformations
b) Intrauterine fetal death
c) Macrosomia
d) Post-term delivery
e) Preterm labor
The answer is: Post-term delivery
28) Which statement accurately describes the changes in total body water composition of children?

26
EMLE Trial Exams December 2020 (1000 Qs) Version (1.0) 22/12/2020

a) Water accounts for almost all 85% to 90% of body weight for infants and decreases over time (to 60%)
in adolescents and young adults
b) Water accounts for 70% to 75% of body weight for infants and decreases over time (to 50%-60%) in
adolescents and young adults
c) Water accounts for less than half (40%-50%) of body weight for infants and decreases over time (to 35%)
in adolescents and young adults
d) Water accounts for 40% to 50% of body weight for infants and increases over time (to 50%-60%) in
adolescents and young adults
e) Water accounts for 60% of body weight for infants and increases over time (to 85%) in adolescents and
young adults
The answer is: Water accounts for 70% to 75% of body weight for infants and decreases over time (to 50%-60%)
in adolescents and young adults
29) A 35-year- old male presents with severe head trauma. Fundus examination revealed papilledema and
increased intracranial tension and impending herniation is suspected . Mannitol is given and a decision to
hyperventilate the patient till neurosurgical consultation is available. Which of the following is an
appropriate target level of PCO2 for therapeutic hyperventilation?
a) 22mmHg
b) 27mmHg
c) 32mmHg
d) 37mmHg
e) 42mmHg
The answer is: 32mmHg
30) A 30-year-old woman presents with multiple bilateral breast swell-ings which cause her discomfort,
particularly just before her periods. They have been present for several years but appear to be getting
worse. What is the most likely diagnosis?*
a) Duct ectasia
b) Fat necrosis
c) Fibroadenoma
d) Fibrocystic disease
e) Peau d’orange
The answer is: Fibrocystic disease
31) A 33-year-old woman on OCPs presents with progressive jaundice and right upper quadrant abdominal
pain. She has a history of DVT. On examination, there is tender hepatomegaly, ascites, bilateral ankle and
sacral edema. Spleen is not palpable. What is the best investigation to diagnose this condition?*
a) CT angiography
b) Abdominal X-ray
c) HIDA scan
d) ERCP
e) MRCP
The answer is: CT angiography
32) An 18-year-old woman has a yellowish vaginal discharge. On examination, the cervix is erythematous and
the discharge reveals upon microscopic examination clue cells. Which is the most likely etiology?
a) Neisseria gonorrhea.
b) Chlamydia trachomatis.
c) Urea plasma species.
d) Bacterial vaginosis.
e) none of the above

27
EMLE Trial Exams December 2020 (1000 Qs) Version (1.0) 22/12/2020

The answer is: Chlamydia trachomatis.


33) You notice that you are finding it harder to read the notes at work, including when writing prescriptions.
You made a mistake earlier this week that almost resulted in a patient receiving the wrong dose of
gentamicin. There is a history of hereditary macular degeneration in your family, you are concerned you
may be developing symptoms. What should you do?***
a) Ignore it, you may lose your job if you tell someone
b) Buy some reading glasses online and hope for the best
c) Talk to your clinical/educational supervisor about your concerns
d) Do not take any action, hope that nobody will notice
e) Take time off work, phone in with the flu
The answer is: Talk to your clinical/educational supervisor about your concerns
34) A 72- year-old man presented right lower limb pain and numbness started 4 hours ago. He is heavy smoker
and gives a history of diabetes mellitus controlled by insulin & pain in the right leg on walking for 5 minutes
for 2 years that obligate him to get rest and continue walking after 3 minutes. On examination of the right
lower limb, there is pale bluish discoloration of skin, wasting of calf muscle bulk, femoral pulsation is not
felt. The most appropriate provisional diagnosis is:
a) DVT
b) Chronic lower limb ischemia
c) Acute lower limb ischemia
d) Severe diabetic neuropathy
e) Buerger’s disease.
The answer is: Acute lower limb ischemia
35) During early pregnancy, a pelvic examination may reveal that one adnexia is slightly enlarged. This is most
likely due to:
a) A Para ovarian cyst.
b) Fallopian tube hypertrophy.
c) Ovarian neoplasm.
d) Follicular cyst.
e) Corpus luteal cyst.
The answer is: Corpus luteal cyst.
36) A 68-year-old female is involved in a highspeed motor vehicle collision and sustains a left knee dislocation.
She is unable to dorsiflex or evert her foot. What nerve is injured?
a) Obturator
b) Peroneal
c) Posterior tibial
d) Sciatic
e) Tibia
The answer is: Peroneal
37) There is good evidence that a woman who gave birth to an infant with a neural tube defect (NTD) can
substantially reduce the risk of recurrence by taking periconceptional folic acid supplementation. What is
the recommended dose?
a) 0.4 mg
b) 0.8 mg
c) 1.0Mg
d) 4 mg
e) 8 mg
The answer is: 4 mg

28
EMLE Trial Exams December 2020 (1000 Qs) Version (1.0) 22/12/2020

38) A pregnant patient with severe pre-eclampsia is given prophylactic labetalol and magnesium sulfate.
Examination shows absent deep tendon reflexes bilaterally. Which of the following is the most
appropriate next step in the management of this patient?
a) Stop magnesium sulfate and give calcium gluconate
b) Stop labetalol
c) Stop magnesium sulfate and give lorazepam
d) Perform nerve conduction studies
e) Perform emergency cesarean section
The answer is: Stop magnesium sulfate and give calcium gluconate
39) Which of the following investigations is most helpful in the assessment of a child presenting with an acute
asthma attack:
a) White cell count and differential
b) Arterial blood gases
c) Chest x-ray
d) Pulmonary function tests
e) O2 saturation monitoring
The answer is: Arterial blood gases
40) A 9-year-old male presents with progressively worsening right eyelid swelling, pain, and redness, for 3
days. He denies blurry vision. Which of the following is more characteristic of orbital cellulitis than
periorbital cellulitis?
a) Fever
b) Periorbital edema
c) Eye tenderness
d) S. aureus as the etiologic agent
e) Restricted eye movement
The answer is: Restricted eye movement
41) A primigravid patient has been having a long prodromal labor but has finally entered active phase of Stage
1. She is excited and wants to know what she should expect as a typical labor course. Average durations
of active labor without epidural analgesia are best expressed by which of the following?
a) first stage, 750 minutes; second stage, 80 minutes; third stage, 30 minutes
b) first stage, 80 minutes; second stage, 20 minutes; third stage, 5 minutes
c) first stage, 120 minutes; second stage, 80 minutes; third stage, 5 minutes
d) first stage, 80 minutes; second stage, 20 minutes; third stage, 20 minutes
e) first stage, 750 minutes; second stage, 80 minutes; third stage, 5 minutes
The answer is: first stage, 750 minutes; second stage, 80 minutes; third stage, 5 minutes
42) Which of the following drugs is used as a first line in any diabetic patient?
a) Exenatide
b) Rosiglitazone
c) Metformin
d) Sitagliptin
e) Acarbose
The answer is: Metformin
43) Which bone is most often fractured in difficult deliveries?
a) Clavicle
b) Humerus
c) Radius
d) Femur

29
EMLE Trial Exams December 2020 (1000 Qs) Version (1.0) 22/12/2020

e) Tibia
The answer is: Clavicle
44) A 48-year-old-woman presents with a long-standing, painless swelling on her right thigh which disappears
on lying flat. She is otherwise well. On examination, the swelling is bluish and non-pulsatile. It lies below
and lateral to the pubic tubercle. What is the most likely diagnosis?
a) Femoral hernia
b) Inguinal hernia
c) Psoas abscess
d) Saphena varix
e) Sebaceous cyst
The answer is: Saphena varix
45) What is the best treatment of a patient came to the hospital complaining of fever together with pain and
diffuse redness of his lower limb?
a) Penicillin
b) Ampicillin
c) Amox-clav
d) Gentamycin
e) Neomycin
The answer is: Amox-clav
46) A 73-year-old man under goes an elective total gastrectomy via an upper midline abdominal incision. On
the first postoperative day, he is confused and disorientated in time, place and person, and pulling at his
catheter and drains. On examination, his temperature is 38 C, O2 saturation is 87% on 2L O2. What is the
most likely cause of the patient’s confusion?*
a) Chest infection
b) Cerebrovascular accident (CVA)
c) Hypoxia
d) CO2 retention
e) Opiate toxicity
The answer is: Hypoxia
47) A 15-year-old male is brought to ER by his parents with a chief complaint of bloody stools. They describe
the stools as coated with bloody material and report blood on the tissue paper. The patient reports a
history of chronic constipation and straining but has no complaints. He has a benign abdominal
examination, normal vital signs, and a hemoglobin value of 14.5 g/dL. Which of the following is true?
a) Anal fissures are usually painless
b) Anal hemorrhoids are the likely source of bleeding
c) Diverticulosis is a common cause of pediatric lower GI bleeding
d) The patient should be admitted for a 99m technetium pertechnetate infusion
e) The patient should be referred for outpatient GI evaluation for possible juvenile polyps
The answer is: Anal hemorrhoids are the likely source of bleeding
48) A 73-year-old man under goes an elective total gastrectomy via an upper midline abdominal incision. On
the first postoperative day, he is confused and disorientated in time, place and person, and pulling at his
catheter and drains. On examination, his temperature is 38 C, O2 saturation is 87% on 2L O2. What is the
most likely cause of the patient’s confusion?*
a) Chest infection
b) Cerebrovascular accident (CVA)
c) Hypoxia
d) CO2 retention

30
EMLE Trial Exams December 2020 (1000 Qs) Version (1.0) 22/12/2020

e) Opiate toxicity
The answer is: Hypoxia
Repeated Q 46
49) The drug of choice for sedation in the setting of acute delirious state in critically ill patients is:*
a) Haloperidol
b) Diazepam
c) Diphenhydramine
d) Morphine
e) Promethazine
The answer is: Haloperidol
50) A 32-year-old male presents to the outpatient clinic complaining of recurrent attacks of sudden onset of
left-hand contractions that fades on its own. His consciousness in intact during the attacks. Which is the
most likely diagnosis?#
a) Generalized myoclonic seizure
b) Generalized atonic seizure
c) Simple partial seizure
d) Complex partial seizure
e) Generalized tonic-atonic seizure
The answer is: Simple partial seizure
51) You are counselling a mother on nutrition for her children. She is very interested in the roles that vitamins
play in health. Which of the following is appropriate advice?
a) Vitamin C deficiency can lead to mucosal bleeding.
b) Vitamin K is only important in the synthesis of procoagulants.
c) Vitamin E deficiency leads to decreased clot generation.
d) Vitamin B 12 deficiency is common in infants.
e) Folic acid (vitamin B 9) has no role in blood production
The answer is: Vitamin C deficiency can lead to mucosal bleeding.
52) the presence of curd-like discharge consistent with Candida on speculum examination A 32-year-old
woman has a twin pregnancy at 8 weeks’ gestation. During her initial prenatal care visit, you review risks
for multifetal pregnancies. Which of the following statements reflects the most frequent risks in twin
pregnancies?
a) Pregnancy-induced hypertension occurs at a higher rate than in singletons.
b) Cesarean delivery is necessary in greater than 90% of twin deliveries.
c) Shoulder dystocia occurs more in the aftercoming vertex twin, as compared to a singleton.
d) Perinatal death rate is less than that of singletons.
e) Congenital anomalies occur at the same rate as in singletons
The answer is: Pregnancy-induced hypertension occurs at a higher rate than in singletons.
53) A 12-year-old female falls and sustains a right elbow dislocation. She is unable to abduct her fingers and
has numbness to the volar aspect of the little finger. Which nerve is injured?
a) Axillary
b) Median
c) Musculocutaneous
d) Radial
e) Ulnar
The answer is: Ulnar
54) A 56-year- female with long standing hypertension, renal failure on dialysis, and a seizure disorder
presents to the emergency department after a generalized seizure. After control of seizure, there was

31
EMLE Trial Exams December 2020 (1000 Qs) Version (1.0) 22/12/2020

persistent bleeding tongue wound. Platelet count was 162000/uL, hemoglobin was 8.5 g/dL. Which of the
following is the best approach to stop this bleeding?
a) Platelet transfusion
b) Emergent hemodialysis
c) IV desmopressin (dDAVP)
d) Suture repair of the tongue wound
e) Estrogen
The answer is: IV desmopressin (dDAVP)
55) Which is the best treatment of acute flare of systemic lupus arthritis?#
a) Methotrexate
b) Steroids
c) Hydroxychloroquine
d) Infliximab
e) Ibuprofen
The answer is: Steroids
56) A 61-year-old man, with an 8-year history of hepatitis C infection and well-documented cirrhosis and
portal hypertension, presents with a large hematoma on his thigh. On preoperative screening, his
prothrombin time is noted to be 17.4 seconds (N: 11-13.5 seconds). Transfusion of which of the following
is the most appropriate next step in management of this patient prior to his procedure?*
a) Cryoprecipitate
b) Fresh frozen plasma
c) Packed red blood cells
d) Platelets
e) Whole blood
The answer is: Fresh frozen plasma
57) A 33-year-old woman on OCPs presents with progressive jaundice and right upper quadrant abdominal
pain. She has a history of DVT. On examination, there is tender hepatomegaly, ascites, bilateral ankle and
sacral edema. Spleen is not palpable. What is the best investigation to diagnose this condition?*
a) CT angiography
b) Abdominal X-ray
c) HIDA scan
d) ERCP
e) MRCP
The answer is: CT angiography
58) Cause of biphasic hyperbilirubinemia include all except:
a) Hepatitis B virus infection
b) Chronic active hepatitis
c) Wilson’s disease
d) Galactosemia
e) ABO
The answer is: ABO
59) The most common type of cerebral palsy is:
a) Spastic
b) Atonic
c) Mixed
d) Extra pyramidal
e) Ataxic

32
EMLE Trial Exams December 2020 (1000 Qs) Version (1.0) 22/12/2020

The answer is: Spastic


60) After a woman develops a deep venous thrombosis during pregnancy, which of the following agents is
MOST LIKELY to be contraindicated?
a) Medroxyprogesterone acetate depot (Depo-provera) contraception.
b) Intrauterine contraceptive device (IUD).
c) Combination oral contraceptive.
d) Levonorgestrel silastic implants (Norplant).
e) Prostaglandin compounds.
The answer is: Combination oral contraceptive.
61) A 75-year-old woman presents with a 2-hour history of sudden-onset severe abdominal pain and per
rectum bleeding. Other than a stroke 2 years ago she keeps well. She has been on aspirin and ramipril
since her stroke and takes digoxin for an irregular heart rhythm. On examination she is very distressed;
her abdomen is soft with no evidence of peritonism. Bowel sounds are audible. An arterial blood gas
shows a metabolic acidosis. What is the likeliest diagnosis?#
a) Ruptured aortic aneurysm
b) Acute small bowel ischaemia
c) Perforated peptic ulcer
d) Diverticulitis
e) Irritable bowel syndrome
The answer is: Acute small bowel ischaemia
62) By the age of 3 years, the average weight of a child is:
a) 10kg
b) 12kg
c) 14kg
d) 20kg
e) none of the above
The answer is: 14kg
63) The MOST COMMON site of infection of the vulva:
a) The clitoris
b) The vestibular bulbs
c) The labia majora
d) The Bartholin’s glands
e) The Hymen
The answer is: The Bartholin’s glands
64) A 55-year-old male presents with chest pain that is ongoing for the past 30 minutes. Which is the initial
step in management?#
a) Chest x-ray
b) Echocardiography
c) CT chest
d) ECG
e) Cardiac enzymes
The answer is:: ECG
65) The following fractures ore more common in postmenopausal females except:
a) Fracture surgical neck of humerus.
b) Fracture neck femur.
c) Supracondylar fracture humerus.
d) Colle's fracture.

33
EMLE Trial Exams December 2020 (1000 Qs) Version (1.0) 22/12/2020

e) a,b and d
The answer is: Supracondylar fracture humerus.
66) These conditions are always an indication for C-section. Which one of these is an absolute indication for
CS?
a) Twin pregnancy.
b) Breech presentation.
c) Severe PET.
d) Minor degree placenta previa.
e) IUGR.
The answer is:: Severe PET.
67) A40-year-old male is brought to the ED in mild respiratory distress upon arrival he is tachypneic at 28 b/m
with an oxygen saturation of 97% on facemask oxygen. On physical examination, the patient appears to
be in mild distress with supraclavicular retractions. Scattered wheezing is heard throughout bilateral lung
fields. Which of the following medication should be administered first?
a) Corticosteroids
b) Magnesium sulfate
c) Epinephrine
d) β2-agonist nebulizer treatment
e) Anticholinergic nebulizer treatment
The answer is: β2-agonist nebulizer treatment
68) Glucose in oral rehydration solution will be useful in:
a) Improve the taste of the rehydration solution
b) Meet the child’s energy requirements
c) Increase osmolality of ORS to 400 mOsmol
d) Promote the absorption of sodium
e) Improve the smell
The answer is: Promote the absorption of sodium
69) Photosensitivity dermatitis in nutritional deficiency disorders of childhood is usually caused by deficiency
of:
a) niacin
b) zinc
c) vitamin A
d) riboflavin
e) vitamin C
The answer is: niacin
70) A 24 –year- old man comes into the emergency department with partial thickness burn all over his body
from a house fire. He has persistent cough and stridor. There are deep neck burns, carbonaceous sputum,
and soot in the mouth and oedema of the of the oropharynx, which is the most appropriate single
management?
a) Topical antibiotic
b) Fluid resuscitation
c) Immediate burn care and cooling
d) intravenous analgesia
e) Tracheal intubation
The answer is: Tracheal intubation
71) Termination of pregnancy in cases of preterm premature rupture of membranes occur in the following
conditions EXCEPT:#

34
EMLE Trial Exams December 2020 (1000 Qs) Version (1.0) 22/12/2020

a) The patient goes in active labor


b) There are uterine contractions 1/15 minutes
c) Fetal distress occurred
d) Presence of fever after excluding extrauterine causes of fever
e) Cervix is 5 cm dilated
The answer is: There are uterine contractions 1/15 minutes
72) An 85-year-old woman is being transferred to an acute rehabilitation facility following a hospital
admission for hip replacement surgery. Postoperatively during her hospital stay, she is started on deep
vein thrombosis (DVT) prophylaxis medication with plans to continue the medication upon discharge. The
intern and nurse who are discharging the patient fail to convey this new medication to the receiving
treatment team at the rehabilitation center. The patient is not continued on her anticoagulation
medication and sustains a DVT, leading to a fatal pulmonary embolus 3 weeks after transfer. Which of the
following actions will facilitate quality improvement and the prevention of a similar error in the future?**
a) Determine which staff member(s) failed to order the medication
b) Develop a process to increase the use of medication reconciliation
c) Send a memo to all staff about the importance of DVT prophylaxis
d) Educate patients about the dangers of DVT following hip surgery
e) Conduct monthly audits to monitor medication errors at transitions of care
The answer is: Develop a process to increase the use of medication reconciliation
73) Signs of hypertonic dehydration include all except:
a) Lethargy and hypotonia
b) Irritability
c) Dry tongue
d) Only moderate loss of skin turgor
e) Severe thirst
The answer is: Lethargy and hypotonia
74) What is the best initial treatment of a patient who came to the hospital complaining of drowsiness. On
reviewing lab investigations, his PH was 7.21, HCO3 level was 15 and serum blood glucose 400 mg/dl?
a) IV 0.9% NaCl bolus
b) IV 0.45% NaCl bolus
c) IV 20 ml KCL
d) IV HCO3
e) IV insulin infusion
The answer is: IV 0.9% NaCl bolus
75) Which is the best test for calculating statistical significance?#^
a) Chi Square
b) Relative risk
c) Odds ratio
d) Confidence interval
e) Number needed to treat
The answer is: Confidence interval
76) A 32-year-old male presents to the emergency department after he fall to the ground and started shaking
in the form of alternating rigidity and contractions of his limbs. He is unconscious. Which is best initial
treatment?#
a) Lorazepam
b) phenytoin
c) Valproate

35
EMLE Trial Exams December 2020 (1000 Qs) Version (1.0) 22/12/2020

d) Carbamazepine
e) Lamotrigine
The answer is: Lorazepam
77) What is the best treatment of a patient with high LDL after a stroke attack?
a) Atorvastatin 20 mg
b) Atorvastatin 40 mg
c) Atorvastatin 80 mg
d) Oral fenofibrate
e) Omega 3 tablets
The answer is: Atorvastatin 80 mg
78) A 82 –year- old female with history of hypertension presents with altered mental status for 2 days. She is
feverish, pulse 110 bpm, RR 22 breath per minute, BP 85/45 mmHg, O2 sat 94%on room air. Urine analysis
shows >100 WBCs. Appropriate antibiotics are already started. Which of the following is the most
appropriate next step in her management?
a) Intravenous fluid bolus 30ml /kg
b) Norepinephrine 0.5 mcg/kg /minute
c) Milrinone 0.5mcg /kg/minute
d) Dobutamine 5mcg /kg /minute
e) Dopamine 5 mcg /kg /minute
The answer is: Intravenous fluid bolus 30ml /kg
79) A 2-year-old girl appears in clinic for a routine check-up. She does not like to eat at all but she loves potato
chips. The best measure to check her nutritional status:#
a) Head circumference
b) Length
c) Weight
d) Weight for height
e) Skin fold thickness
The answer is: Weight for height
80) Croup syndromes are characterized by all of the following except:#
a) Distinctive brassy cough
b) Hoarseness of voice
c) Expiratory stridor
d) Respiratory distress
e) Being an acute condition
The answer is: Expiratory stridor
81) A 32-year-old male presents to the outpatient clinic complaining of recurrent attacks of sudden onset of
left-hand contractions that fades on its own. He becomes confused but doesn’t lose consciousness during
the attacks. Which is the most likely diagnosis?#
a) Generalized myoclonic seizure
b) Generalized atonic seizure
c) Simple partial seizure
d) Complex partial seizure
e) Generalized tonic-atonic seizure
The answer is: Complex partial seizure
82) Child 3 years old you noticed that he just says mama and dada with no eye contact, mother says he is not
playing with others at the nursery, what do you suspect?
a) ADHD

36
EMLE Trial Exams December 2020 (1000 Qs) Version (1.0) 22/12/2020

b) Cerebral palsy
c) Autism
d) Rickets
e) Down syndrome
The answer is: Autism
83) A 44-year-old man presents to the GP having noticed a painless swelling in the right side of the scrotum.
On examination, the scrotum is swollen and non-tender. The swelling transilluminates and the testis itself
is not palpable. What is the most likely diagnosis?*
a) Epididymo-orchitis
b) Hydrocele
c) Inguinal hernia
d) Testicular torsion
e) Varicocele
The answer is: Hydrocele
84) A 10-month-old female presents to the emergency department with a 2-day history of runny nose and
fever to 102°F. On examination, her temperature is 103°F, HR 140 beats per minute, and a RR 30 times per
minute. She is alert and playful with copious rhinorrhea. After the examination is complete, she becomes
stiff and displays tonic-clonic movements of all four extremities. Which of the following should be your
first task?
a) obtain whole blood glucose
b) administer intravenous lorazepam (Ativan)
c) perform lumbar puncture
d) establish airway patency
e) Administer intramuscular fosphenytoin (Cerebyx)
The answer is: establish airway patency
85) A 69-year-old man who is known to have Alzheimer's disease is reviewed in clinic. His latest Mini Mental
State Examination (MMSE) score is 18 out of 30. What is the best treatment of an end-stage Alzheimer’s
disease?
a) Supportive care + memantine
b) Supportive care + trial of citalopram
c) Supportive care+ donepezil
d) Supportive care+ donepezil+ low dose aspirin
e) None of the above
The answer is: Supportive care + memantine
86) A child is diagnosed with latex allergy in the hospital. He should be consulted that which of the following
foods are likely to care from?
a) Apples
b) Oranges
c) Strawberry
d) Tomatto
e) Cucamber
The answer is: Cucamber
87) The triad of signs of rupture urethra includes the following except:
a) Perineal hematoma
b) Bleeding per urethra
c) Retention of urine
d) Bleeding per rectum

37
EMLE Trial Exams December 2020 (1000 Qs) Version (1.0) 22/12/2020

e) non are true


The answer is: Bleeding per rectum
88) All are definitive diagnostic signs of severe pre-eclampsia EXCEPT:#
a) Renal impairment.
b) Systolic arterial pressure equal or above 160mmHg.
c) Diastolic blood pressure equal or above 90 mmHg
d) Retinalhemorrhage
e) Pulmonary edema
The answer is: Diastolic blood pressure equal or above 90 mmHg
89) Which of the following is a cause of tall stature in childhood?
a) Noonan syndrome
b) Growth hormone deficiency
c) Turner syndrome
d) Precocious puberty
e) Hypothyroidism
The answer is: Precocious puberty
90) A 46-year-old man presents with the sudden onset of fever and dysuria. Rectal examination finds the
prostate gland to be enlarged and very sensitive; examination is quite painful. Microscopic examination
of prostatic secretions reveals the presence of numerous neutrophils. Which of the following organisms
is the most likely cause of this illness?
a) Bacillus Calmette-Guerin
b) Escherichia coli.
c) Proteus mirabillis
d) Staphylococcus aureus
e) Ureaplasma urealyticum
The answer is: Escherichia coli.
91) Which one of the following does not occur as a systemic manifestation of inflammatory bowel disease?*
a) Amyloidosis
b) Pyoderma gangrenosum
c) Scleritis
d) Cardiomyopathy
e) Sclerosing cholangitis
The answer is: Cardiomyopathy
92) A newly born infant is noted to have coughing and cyanosis during feeding with marked gastric dilation
due to “swallowed” air. Radiological workup reveals that this infant has the most common type of
esophageal atresia. Which one of the following statements describes this type of congenital abnormality?
a) Atresia of the esophagus with fistula between both segments and the trachea
b) Atresia of the esophagus with fistula between the trachea and the blind upper segment
c) Atresia of the esophagus with fistula between the trachea and the distal esophageal segment
d) Atresia of the esophagus without tracheoesophageal fistula
e) Fistula between a normal esophagus and the trachea
The answer is: Atresia of the esophagus with fistula between the trachea and the distal esophageal segment
93) In a patient with refractory hypotension after blunt trauma, which of the following findings is the primary
utility of the focused assessment with sonography in trauma (FAST) scan?*
a) Pelvic fracture
b) Renal injury
c) Diaphragmatic rupture

38
EMLE Trial Exams December 2020 (1000 Qs) Version (1.0) 22/12/2020

d) Hemoperitoneum
e) Aortic injury
The answer is: Hemoperitoneum
Repeated Q 2
94) Which is the best statistical test for case control study analysis?#
a) Chi Square
b) Relative risk
c) Odds ratio
d) P value
e) Confidence interval
The answer is: Odds ratio
95) A-40 year –old male presents with delirium, polyuria, and serum calcium level of 15mg/dL. Which of the
following is true?
a) Nifedipine is the agent of choice for initial treatment
b) Thiazide diuretics are more helpful than loop diuretics in promoting urine calcium excretion
c) Calcitonin can be used as monotherapy in the treatment of hypercalcemia
d) Sodium bicarbonate infusion will increase the amount of ionized calcium
e) Glucocorticoids may be helpful if the patient has an underlying hematologic malignancy or
granulomatous disease.
The answer is: Glucocorticoids may be helpful if the patient has an underlying hematologic malignancy or
granulomatous disease.
96) Bronchiectasis is characterized by all except:
a) Productive cough is the most characteristic symptom
b) Best treatment is surgery
c) Chest physiotherapy is an important line of treatment
d) May complicate whopping cough
e) Best investigation is CT
The answer is: Best treatment is surgery
97) An African American couple presents for prenatal care. They are interested in complete screening as
indicated for genetic diseases. They are unaware of their sickle cell status regarding the presence of trait
or not. The mother is found to have sickle trait. If the father has trait, what percentage of their children
will be born with sickle disease?
a) 0
b) 25
c) 50
d) 75
e) 100
The answer is: 25
98) A 41-year-old woman is suspected of having intrauterine adhesions because she has had irregular menses
since a spontaneous abortion 18 months previously. Which historical or laboratory pieces of information
would support this diagnosis?
a) Presence of hot flushes.
b) FSH level too low to be measurable.
c) Normal estradiol levels for a reproductive age woman.
d) Monophasic basal body temperature chart.
e) none of the above
The answer is: Normal estradiol levels for a reproductive age woman.

39
EMLE Trial Exams December 2020 (1000 Qs) Version (1.0) 22/12/2020

99) Precautions to minimize bleeding during myomectomy DO NOT include:#


a) Preoperative LH-RH analogue
b) Timing of the operation in the premenstrual phase
c) Single rather than multiple incisions
d) Midline rather than lateral incisions
e) Preoperative antibiotics
The answer is: Timing of the operation in the premenstrual phase
100) A normal neonate has all the following characters except:
a) Length of 50 cm
b) Skull circumference of 35 cm
c) Moro’s reflex
d) Grasping reflex
e) Hemoglobin of 10g%
The answer is: Hemoglobin of 10g%

40
EMLE Trial Exams December 2020 (1000 Qs) Version (1.0) 22/12/2020

Exam 3
1) A 28-year-old man with Crohn’s disease complains of watery discharge from apuckered area 2 cm from
the anal canal. The most likely cause for the clinical findings is:
a) fissure-in-ano
b) perianal warts
c) proctalgia fugax
d) second-degree haemorrhoids
e) fistula-in-ano
The answer is: fistula-in-ano
2) Child 8 years you saw him in clinic being hyperactive and mother complains about school performance
He cannot focus in class, what do you suspect?
a) ADHD
b) Cerebral palsy
c) Autism
d) Rickets
e) Down syndrome
The answer is: ADHD
3) A pregnant woman with fibroid uterus develops acute pain in abdomen with low-grade fever and mild
leukocytosis at 28 weeks. The MOST LIKELY diagnosis is:#
a) Preterm labor
b) Torsions of fibroid
c) Red degeneration of fibroid
d) Infection in fibroid
e) none of the above
The answer is: Red degeneration of fibroid
4) Which of the following is essential for an informed consent taking?*
a) It could be taken from children
b) The patient should know everything about his disease but not the management
c) If the patient doesn’t speak Arabic, explain to him by your mother language
d) The consent could be oral
e) Competence is the most essential issue
The answer is: Competence is the most essential issue
5) In attempting to minimize complications during cholecystectomy, the surgeon defines the triangle of
Calot during the operation. The boundaries of the triangle of Calot (modified) are the common hepatic
duct medially, the cystic duct inferiorly, and the liver superiorly. Which structure courses through this
triangle?#
a) Left hepatic artery
b) Right renal vein
c) Right hepatic artery
d) Cystic artery
e) Superior mesenteric vein
The answer is: Cystic artery
6) 65-year-old presents with fatigue and dizziness for the past 6 months. On examination, blood pressure
was 90/60. Her skin was hyperpigmented and her electrolytes was as follows. Na 125 (135 – 145), K 6
(3.5 – 5.3). Her blood pressure failed to improve after fluids. Which is the most likely next step in
management?*

41
EMLE Trial Exams December 2020 (1000 Qs) Version (1.0) 22/12/2020

a) IV normal Saline
b) IV corticosteroids
c) IV thyroxine
d) IV insulin
e) IV glucagon
The answer is: IV corticosteroids
7) Regarding testicular tumors, the untrue is:
a) seminomas and teratomas ore more common than non-germ cell tumors.
b) Seminomas send pulmonary metastasis.
c) They have on increased incidence in undescended testis.
d) a-fetoprotein and B-HCG ore tumor markers.
e) The treatment of choice is radical orchidectomy
The answer is: Seminomas send pulmonary metastasis.
8) A 25-year-old female presents with episodes of bizarre behaviour, memory lapse, and unconsciousness.
She also demonstrated previously episodes of extreme hunger, sweating, and tachycardia. During one of
these episodes, her blood sugar was tested and was found to be 40 mg/dL. Which of the following
would most appropriately indicate a diagnosis of insulinoma?#
a) Demonstration of insulin antibodies in blood
b) Abnormal glucagon level
c) CT of the pancreas showing a mass
d) Hypoglycaemia during a symptomatic episode with relief of symptoms by intravenous glucose
e) Decreased circulating C peptide in the blood
The answer is: Hypoglycaemia during a symptomatic episode with relief of symptoms by intravenous glucose
9) The following is a syndrome causing mental retardation:
a) Marasmus
b) Rickets
c) Cerebral palsy
d) Tisomy 18
e) Khwashirkor
The answer is: Tisomy 18
10) Which of the following pediatric heart diseases causes cyanosis?
a) Mitral stenosis
b) Coarctation of the aorta
c) Transposition of the great vessels
d) Atrial septal defect (ASD)
e) Ventricular septal defect (VSD)
The answer is: Transposition of the great vessels
11) A 44-year-old man presents with the sudden onset of severe right upper quadrant (RUQ) abdominal
pain, ascites, tender hepatomegaly, and hematemesis. These symptoms are suggestive of Budd-Chiari
syndrome. Which of the following is the most likely cause of this disorder?
a) Obstruction of the common bile duct
b) Obstruction of the intrahepatic sinusoids
c) Thrombosis of the hepatic artery
d) Thrombosis of the hepatic vein
e) Thrombosis of the portal vein
The answer is: Thrombosis of the hepatic vein
12) Head circumference (HC) increase in the first 3 months of life by

42
EMLE Trial Exams December 2020 (1000 Qs) Version (1.0) 22/12/2020

a) 0.5 cm per month


b) 1 cm per month
c) 1.5cm per month
d) 2cm per month
e) 2.5 cm per month
The answer is: 2cm per month
13) Risk factors of hepatitis A include all of the following except:
a) Recent travel or migration to an endemic area
b) Sewage workers
c) Multiple transfusion of blood and or its products
d) eating street food
e) Daycare employee or children
The answer is: Multiple transfusion of blood and or its products
14) 55year-old man with past medical history of tobacco abuse presents with shortness of breath and
cough. His vitals are as follows Temperature 37, blood pressure 120/80, heart rate 105, O2 saturation
93%. On examination, decrease air entry on the left hemithorax was heard and trachea was midline.
Which is the most likely treatment?#
a) Antibiotics
b) Steroids and bronchodilators
c) Observation
d) Anticoagulants
e) needle thoracostomy
The answer is: Observation
15) A 10-month-old girl presents with abdominal pain and diarrhea which has recently had pink staining.
She was well until 12 hours ago when she had some loose stools and episodes of crying and holding her
abdomen. On examination, she looks intermittently uncomfortable. Her abdomen is slightly distended
and she has a curved firm mass 10 cm long extending from the right iliac fossa towards the hepatic
flexure. What is the most likely diagnosis?
a) Inborn error of metabolism
b) Constipation/encopresis
c) Intussusception
d) Renal anomalies
e) Malignancy e.g. neuroblastoma, Wilms tumor, lymphoma
The answer is: Intussusception
16) A 55-year-old African man is referred to the ENT clinic with a large, smooth goitre that has been
increasing in size for at least 20 years.#
a) follicular carcinoma
b) Hashimoto’s disease
c) anaplastic carcinoma
d) endemic goitre
e) lymphoma
The answer is: endemic goitre
17) As regared progestin-only injectables the followings are true EXCEPT:
a) Menstrual irregularities are more common than for OCs
b) Contraceptive effect and side effects cannot be stopped immediately
c) Return to fertility after discontinuation is usually delayed
d) Weight gain (more common for progestin-only injectables)

43
EMLE Trial Exams December 2020 (1000 Qs) Version (1.0) 22/12/2020

e) Offer protection against sexually transmitted infections


The answer is: Offer protection against sexually transmitted infections
18) Recognized features of Fallot’s tetralogy include all except:#
a) Cyanosis
b) Pulmonary oligemia
c) Murmur of pulmonary stenosis
d) Murmur of VSD
e) Thrombotic complications
The answer is: Murmur of VSD
19) The vaccine given immediately after birth or during the first month of life is
a) DPT
b) Polio
c) Measles
d) BCG
e) MMR
The answer is: MMR
20) 40year-old female presents with dysuria, frequency, suprapubic pain and urinary discharge that is
purulent. Patient denies fever and costovertebral angle tenderness. Which is most best empiric
antibiotic combination to use?#
a) Ceftriaxone and Azithromycin
b) Ceftriaxone, vancomycin and ampicillin
c) Ceftriaxone and vancomycin
d) Trimethoprim-sulfamethazole
e) Penicillin
The answer is: Ceftriaxone and Azithromycin
21) Booking investigations include all the following, EXCEPT:
a) Liver function test.
b) Glucose challenge test.
c) CBC.
d) US.
e) Toxoplasmosis.
The answer is: Liver function test.
22) Pneumococcal pneumonia could be complicated with all except:
a) Empyema
b) Heart failure
c) Lung abscess
d) Respiratory failure
e) Asthma
The answer is: Asthma
23) For the first 6 hours following a long and difficult surgical repair of a 7-cm abdominal aortic aneurysm, a
70-year-old man has a total urinary output of 25 mL since the operation. Which of the following is the
most appropriate diagnostic test to evaluate the cause of his oliguria?
a) Renal scan
b) Aortogram
c) Left heart preload pressures
d) Urinary sodium concentration
e) Creatinine clearance

44
EMLE Trial Exams December 2020 (1000 Qs) Version (1.0) 22/12/2020

The answer is: Left heart preload pressures


24) A 7-year-old female presents with syncope without a prodrome. EKG shows QT prolongation. Family
history is most likely to reveal which of the following?
a) Cystic fibrosis
b) Hirschsprung disease
c) Juvenile rheumatoid arthritis
d) Deafness
e) Short stature
The answer is: Deafness
25) A 24-year-old nulligravid woman, married for 5 months presents with a non-tender cystic mass in her
right vulva that causes some discomfort when walking and during coitus. The mass was at the posterior
part of labium major and was about 3.0X3.0 cm in dimensions. What is the MOST APPROPRIATE initial
decision?
a) Observation.
b) Administration of antibiotics.
c) Incision and drainage.
d) Marsupialization.
e) Surgical excision.
The answer is: Marsupialization.
26) The management in rupture of membranes in 32 weeks’ gestation include all of the following EXCEPT :#
a) Antibiotics for 10 days
b) Tocolysis until 36 weeks gestation
c) Steroids until 36 weeks gestation
d) Induction of labor at 36 weeks gestation
e) Expectant management
The answer is: Tocolysis until 36 weeks gestation
27) 55-year-old male presents with severe headache after head trauma. CT is shown. Which is the source of
bleeding for the most likely source of bleeding for the most likely diagnosis?
a) Anterior cerebral artery
b) posterior cerebral artery
c) bridging veins
d) middle meningeal artery
e) Anterior communicating artery
The answer is: middle meningeal artery
28) Heart failure is treated by:
a) Corticosteroids
b) Digoxin
c) Penicillin
d) Salicylates
e) Halipridol
The answer is: Digoxin
29) Extrinsic (atopic) asthma is characterized by all except:#
a) Intermittent attacks of dyspnea, wheezing and cough
b) Usually precipitated by exposure to an allergen
c) Eosinophilia
d) Elevated serum IgE
e) Depressed serum IgE

45
EMLE Trial Exams December 2020 (1000 Qs) Version (1.0) 22/12/2020

The answer is: Depressed serum IgE


30) A-40 year –old male presents with delirium , polyuria , and serum calcium level of 15mg/dL . Which of
the following is true?
a) Nifedipine is the agent of choice for initial treatment
b) Thiazide diuretics are more helpful than loop diuretics in promoting urine calcium excretion
c) Calcitonin can be used as monotherapy in the treatment of hypercalcemia
d) Sodium bicarbonate infusion will increase the amount of ionized calcium
e) Glucocorticoids may be helpful if the patient has an underlying hematologic malignancy or
granulomatous disease.
The answer is: Glucocorticoids may be helpful if the patient has an underlying hematologic malignancy or
granulomatous disease.
31) A 56-year-old man is brought to hospital by his wife. He has been vomit-ing fresh blood since earlier that
morning. She tells you he has a long his-tory of alcohol abuse and drinks at least a bottle of spirits a day.
Which of the following is the most likely cause of his symptoms?#
a) Dieulafoy lesions
b) Duodenal ulcer
c) Gastric ulcer
d) Oesophageal varices
e) Oesophagitis
The answer is: Oesophageal varices
32) A 24-year-old male is admitted with acute severe asthma. Treatment is initiated with 100% oxygen,
nebulized salbutamol and ipratropium bromide nebulizers and IV hydrocortisone. Despite initial
treatment there is no improvement. What is the next step in management?#
a) IV aminophylline
b) IV magnesium sulphate
c) IV Salbutamol
d) IV adrenaline
e) Non-invasive ventilation
The answer is: IV magnesium sulphate
33) What is the best management of opiate induced nausea in a cancer patient?
a) Give him ondansetron
b) Give him domperidone
c) Give him metoclopramide
d) Give him cyclizine
e) Give him haloperidol
The answer is: Give him haloperidol
34) The pathognomonic sign of measles during the prodromal stage is:
a) Erythema marginatum
b) Koplik’s spots
c) Erythema nodosum
d) Vesicle in the face
e) Strawberry tongue
The answer is: Koplik’s spots
35) A 40-year-old male with no past medical history presents to the outpatient clinic for routine visit. His
Hemoglobin A1C is 7%. What is the most likely next step in treatment?
a) Diet and exercise only
b) Diet and exercise and metformin

46
EMLE Trial Exams December 2020 (1000 Qs) Version (1.0) 22/12/2020

c) metformin and Glipizide


d) Insulin
e) Metformin, Glipizide and exenatide
The answer is: Diet and exercise and metformin
36) What of the following investigations are specific for alcoholic hepatitis?#
a) AST>ALT
b) ALT>AST
c) High Alkaline phosphatase
d) High GGT
e) High creatinine
The answer is: AST>ALT
37) Which of the following is true regarding traumatic iridocyclitis?
a) It is generally painless.
b) Findings include a fixed and dilated pupil.
c) Treatment involves long-acting cycloplegics.
d) Steroids play no role in management.
e) Resolution generally occurs 1 month after the onset of symptoms.
The answer is: Treatment involves long-acting cycloplegics.
38) A 16-year-old woman with CHD is referred at 8 weeks’ gestation by her cardiologist. She has peripheral
cyanosis, a hematocrit of 65%, and is 95 lb and 5 ft. 4 in tall. You counsel her that the most likely fetal
outcome is which of the following?
a) not affected
b) marked prematurity
c) intrauterine growth restriction
d) spontaneous abortion or fetal death
e) postmaturity
The answer is: spontaneous abortion or fetal death
39) 17year-old student is brought to A&E after she had a witnessed fit. History was given by a friend as she
was too drowsy. She suddenly lost consciousness Awhile sitting on a sofa, fell to the floor, then became
rigid and started shaking all limbs vigorously. She bit her tongue and was incontinent of urine.What is
your diagnosis?*#
a) Tonic-clonic seizures
b) Hyponatremia
c) Absence seizures
d) Hypomagnesemia
e) Hypocalcemia
The answer is: Tonic-clonic seizures
40) You see a 24-year-old student with suspected meningitis. The registrar asks you to perform a lumbar
puncture before she goes to the ward. This is not a procedure you have performed before, and you can
recall only limited information about the procedure from medical school teaching. What should you
do?**
a) Explain to the registrar that this is outside of your competence and so you are not comfortable in
performing the procedure
b) Explain to the registrar this is not a skill that should be expected of you
c) Ask an older colleague to perform the procedure whilst you observe and learn for future attempts
d) As the procedure has many potential risks, await the patient's blood test results before deciding if she
really requires the procedure

47
EMLE Trial Exams December 2020 (1000 Qs) Version (1.0) 22/12/2020

e) Do not perform the procedure, but document clearly in the notes that it has been requested and will
need to be performed on the ward
The answer is: Explain to the registrar that this is outside of your competence and so you are not comfortable
in performing the procedure
41) The most common cause of uterine size-date disproportion:
a) Fetal macrosomia
b) Polyhydramnios
c) Inaccurate last menstrual period date
d) Multiple pregnancy
e) Molar pregnancy.
The answer is: Inaccurate last menstrual period date
42) When to expect a normal child to say his name, age and address?
a) 1 year
b) 1.5 year
c) 2years
d) 2.5years
e) 3 years
The answer is: 3 years
43) The best treatment for pediatric patients showing evidence of end-stage renal disease (ESRD) is
a) long-term peritoneal dialysis and a low-protein diet
b) long-term hemodialysis and a low-protein diet
c) a low-protein diet and dihydroxycholecalciferol (vitamin D3)
d) living related donor renal transplantation
e) renal transplantation with a cadaveric kidney
The answer is: living related donor renal transplantation
44) Which of the following is the most appropriate medication to induce rapid reduction in blood pressure
in hypertensive crisis? *
a) Nifedipine
b) Captopril
c) Furosemide diuretic
d) Nitroprusside
e) Clonidine
The answer is: Nitroprusside
45) Which of the following is false about physiological jaundice
a) It rarely presents before the age of 24 hours
b) It may cause kernicterus
c) In premature infants, it may persist for 3-4 weeks
d) It is mainly due to temporarily impaired hepatic clearance if bilirubin
e) It does not always require treatment with phototherapy
The answer is: It may cause kernicterus
46) A 24 –year- old man comes into the emergency department with partial thickness burn all over his body
from a house fire .He has persistent couph and stridor . There are deep neck burns , carbonaceous
sputum , and soot in the mouth and oedema of the of the oropharynx , which is the most appropriate
single management?
a) Topical antibiotic
b) Fluid resuscitation
c) Immediate burn care and cooling

48
EMLE Trial Exams December 2020 (1000 Qs) Version (1.0) 22/12/2020

d) intravenous analgesia
e) Tracheal intubation
The answer is: Tracheal intubation
47) Which of the following should be avoided during hand washing?**
a) Paper towels
b) Liquid soap
c) Nail brushes
d) Washing finger webs
e) Complete drying
The answer is: Nail brushes
48) Which one of the following conditions is least associated with pruritus?*
a) Pemphigus vulgaria
b) Chronic kidney disease
c) Iron deficiency anemia
d) Polycythemia
e) Scabies
The answer is: Pemphigus vulgaria
49) Which of the following is true about neonatal seizures?
a) They are usually absence seizures
b) They are most often due to hyponatremia
c) Treatment with barbiturates is avoided
d) They are usually caused by inadequate breast-feeding
e) They tend to be more subtle than seizures in older children and adults
The answer is: They tend to be more subtle than seizures in older children and adults
50) All the following are possible of premature labor, EXCEPT:
a) Multiple pregnancy
b) Polyhydramnios
c) Bicorrnuate utures
d) Anecephaly
e) Perinatel infection.
The answer is: Anecephaly
51) A 40 year-old patient came to the clinic complaining of fatigue and pallor. CBC reveals HB is 9 g/dl, MCV
is low. Iron studies show low serum iron, low serum ferritin and high TIBC. Past medical history revealed
nothing. What is the best treatment?
a) Oral iron replacement
b) IV iron replacement
c) Blood transfusion
d) FFP transfusion
e) Platelet transfusion
The answer is: Oral iron replacement
52) A 7-year-old is diagnosed with appendicitis after returning to the ED a day after first presenting with
abdominal pain. Missed cases of appendicitis are most often initially diagnosed as:
a) Mesenteric adenitis
b) Intussusception
c) Gastroenteritis
d) Inflammatory bowel disease
e) Pancreatitis

49
EMLE Trial Exams December 2020 (1000 Qs) Version (1.0) 22/12/2020

The answer is: Gastroenteritis


53) In Beta thalassemia major, which statement is false:
a) Boys are affected more than girls
b) Splenectomy may be indicated
c) Urine may be dark with intravascular hemolysis
d) May develop aplastic anemia
e) May develop hyper hemolysis
The answer is: Boys are affected more than girls
54) A premature birth has been defined as
a) Labour before 37 completed weeks' gestation.
b) Prior to the period of viability.
c) Fetus weighing less than 1000 g.
d) Fetus Weighing more than 1000g but less than 2500g.
e) None of the above.
The answer is: Labour before 37 completed weeks' gestation.
55) Advantages of implanon include the followings EXCEPT :
a) Effectiveness: 99% effective in preventing pregnancy
b) No action needs to be taken at the time of intercourse
c) It is long acting, but reversible and rapidly restoring fertility
d) It contains no estrogen, so can be used by women who are breastfeeding
e) Offer protection against sexually transmitted infections
The answer is: Offer protection against sexually transmitted infections
56) A 19-year-old primigravida at term presents to labor and delivery reporting irregular contractions and
rupture of membranes 21 hours prior to arrival. She has not received prenatal care but reports that her
pregnancy was uncomplicated. She is afebrile, and electronic fetal monitoring is reactive with occasional
mild variable decelerations.Cervical examination reveals a dilatation of 3 cm, 50% effacement, -1
station, vertex presentation. Spontaneous rupture of membranes is confirmed. What is the best course
of action at this time?
a) perform an immediate low transverse cesarean section
b) start intravenous (IV) antibiotics for group B streptococcal (GBS)
c) prophylaxis
d) begin an amnioinfusion
e) conduct a contraction stress test
The answer is: start intravenous (IV) antibiotics for group B streptococcal (GBS)
57) You are doing the annual review of a 50-year-old woman who has rheumatoid arthritis. Which one of
the following complications is most likely to occur as a result of her disease?#
a) Chronic lymphoid leukemia
b) Hypertension
c) Colorectal cancer
d) Type 2 DM
e) IHD
The answer is: IHD
58) A patient presents with a positive pregnancy test, the exact date of the start of her last normal menses,
and the date of her luteinizing hormone (LH) surge from a urine kit. Her expected date of delivery can
most correctly be calculated by which of the following?
a) adding 254 to the date of the start of the last menstrual period (LMP)
b) counting 10 lunar months from the time of ovulation

50
EMLE Trial Exams December 2020 (1000 Qs) Version (1.0) 22/12/2020

c) counting 280 from the first day of the LMP


d) counting 40 weeks from the last day of the LMP
e) adding 256 to the date of the elevated urinary LH when detected by home testing
The answer is: counting 280 from the first day of the LMP
59) A 78-year – old female presents with acute onset of right –sided arm and leg weakness 1 hour prior to
arrival .You suspect stroke and immediately order a noncontrast CT brain . The CT of brain is
Unremarkable .Which of the following is the most appropriate conclusion to draw from this CT results? *
a) There is no stroke
b) There is no ischemic stroke
c) There is no meningitis
d) There is no intracranial hemorrhage
e) There is no cervical artery dissection
The answer is: There is no intracranial hemorrhage
60) About etiology of pneumothorax
a) Spontaneous pneumothorax is always secondary to lung disease
b) Accidental pneumothorax necessitates a penetrating injury
c) Mechanical ventilation can result in pneumothorax
d) Central venous line insertion is not a reported cause
e) is not emergency at any degree
The answer is: Mechanical ventilation can result in pneumothorax
61) Rupture of membranes is suspected with all of the followings EXCEPT:
a) Positive Nitrazine test.
b) P ositive Fern test.
c) Pooling of amniotic fluid on speculum examination.
d) Observing amniotic fluid drain through the cervix during speculum examination .
e) Contractions seen on the CTG.
The answer is: Contractions seen on the CTG.
62) Treatment of rupture bladder includes the following except:
a) Resuscitation.
b) Water tight repair without drain.
c) Urethral catheter for 2 weeks.
d) 2ry survey and treatment of associated injuries.
e) ureteric reimplantation
The answer is: Water tight repair without drain.
63) Diagnosis of allergy include all except:#
a) Provocation tests
b) Elevated serum IgE level
c) Neutrophilia
d) Eosinophilia
e) Positive skin test
The answer is: Neutrophilia
64) A 35-year- old male presents with severe head trauma .Fundus examination revealed papilledema and
increased intracranial tension and impending herniation is suspected . Mannitol is given and a decision
to hyperventilate the patient till neurosurgical consultation is available . Which of the following is an
appropriate target level of PCO2 for therapeutic hyperventilation?
a) 22mmHg
b) 27mmHg

51
EMLE Trial Exams December 2020 (1000 Qs) Version (1.0) 22/12/2020

c) 32mmHg
d) 37mmHg
e) 42mmHg
The answer is: 32mmHg
65) Which of the following is the best treatment for a diabetic patient on regular metformin. However, he
came to the clinic complaining of numbness and paresthesia on his feet?#
a) Gabapentin
b) Insulin
c) Paracetamol
d) Ibuprofen
e) Morphine
The answer is: Gabapentin
66) A 15-year-old girl with short stature neck webbing and sexual infantilism is found to have Coarctation of
the aorta. The most likely diagnosis is
a) Marfan’s syndrome
b) Down’s syndrome
c) Turner’s syndrome
d) Ellis-van Creveld
e) An unrelated group of findings
The answer is: Turner’s syndrome
67) A 70 years old man is admitted to the ER with acute upper GI hemorrhage following a bout of repeated
vomiting. fiber optic gastroscopy reveals 3 linears mucosal tears at the gastroesophageal junction , what
is the diagnosis:
a) GERD
b) Barrets esophageus
c) Carcinoma of the esophagus
d) Mallory wise tear
e) Scleroderma
The answer is: Mallory wise tear
68) Which of the following quality measures is used for calculating Percentage of diabetic patients whose
HbAlc was measured in the past 6 months**
a) Structural
b) Process
c) Outcome
d) balancing
e) Non-specific
The answer is: Process
69) A 45-year- old obese woman presents with right upper quadrant abdominal pain for several weeks .Her
pain is worse after she eats .Which of the following clinical features support the diagnosis of biliary colic
?*
a) Worsened pain on eating protein
b) Colicky spasms lasting 10 minutes
c) Radiation of pain to right shoulder
d) Hyperpyrexia
e) Antecedent use of nonsteroidal anti-inflammatory drugs
The answer is: Radiation of pain to right shoulder
70) Which of the following is the treatment of choice for acute cluster headache?

52
EMLE Trial Exams December 2020 (1000 Qs) Version (1.0) 22/12/2020

a) 100 %oxygen
b) Sumatriptan
c) Morphine
d) Lorazepam
e) Dexamethasone
The answer is: 100 %oxygen
71) A 21-year-old student presents with a cramping diffuse abdominal pain associatedwith alternating
constipation and diarrhoea. Colonoscopy and inflammatorymarkers are normal.
a) ulcerative colitis
b) irritable bowel syndrome
c) umbilical hernia
d) diverticulitis
e) perforated duodenal ulcer
The answer is: irritable bowel syndrome
72) A 61 year-old man presents with rapid onset left iliac fossa pain and fever. He is tender in the left iliac
fossa but there is no obvious palpable mass. Temperature is 38.7°C, pulse 94bpm, white cell count 16 ×
109/L, and CRP 60 mg/L. Which is the single most likely diagnosis?#
a) Acute appendicitis
b) Sigmoid cancer
c) Crohn’s disease of the colon
d) Acute sigmoid diverticulitis
e) Ureteric colic
The answer is: Acute sigmoid diverticulitis
73) Which of the following is the best treatment for a newly diagnosed diabetic patient came to the clinic
with HbA1C of 7%
a) Glipizide
b) Metformin
c) Exenatide
d) Sitagliptin
e) Glimepiride
The answer is: Metformin
74) A 26-year-old student presents with bloody diarrhoea, abdominal pain and weight loss. Barium enema
reveals ‘cobblestoning’ and colonic strictures. The most likely cause for the clinical finding is:
a) Crohn’s disease
b) diverticulosis
c) chronic pancreatitis
d) ulcerative colitis
e) gluten-sensitive enteropathy
The answer is: Crohn’s disease
75) A 30-year-old male presents with heart burn for the past few months that is worse after eating food and
lying flat. Patient denies weight loss, decreased appetite and melena. Which is the best next step in
management?
a) Pantoprazole
b) upper gastrointestinal endoscopy
c) barium swallow
d) Esophageal manometry
e) Esophageal pH measurement

53
EMLE Trial Exams December 2020 (1000 Qs) Version (1.0) 22/12/2020

The answer is: Pantoprazole


76) Complications of staph aureus pneumonia include all except:#
a) Osteomyelitis
b) Meningitis
c) Pericarditis
d) Empyema
e) Asthma
The answer is: Asthma
77) The severity of fetal affection in Rh sensitized woman can be detected by all the following EXCEPT :
a) Umbilical cord blood sampling (PUBS)
b) Indirect Coombs test
c) Ultrasound
d) Detailed history
e) Amniocentesis
The answer is: Indirect Coombs test
78) A 65-year-old man presents with sudden-onset severe epigastric pain. He is a smoker and admits to
regular excessive alcohol intake. On examination, he is guarding in the epigastrium. An erect chest X-ray
shows a pneumoperitoneum. What is the most likely diagnosis?#
a) Perforated duodenal ulcer
b) Perforated diverticular disease
c) Acute myocardial infarction
d) Gallbladder empyema
e) Ruptured aortic aneurysm
The answer is: Perforated duodenal ulcer
79) A 65-year-old man reports feeling increasingly tired and short of breath. He had been well until a year
ago, when he started losing weight despite any dietary change. He also complains of pencil thin stools.
His wife has commented that he seems very pale. He is on no medications. Physical examination shows
a pale- appearing man with an elevated heart rate. Rectal examination is positive for fresh blood.
Laboratory studies are remarkable for a hematocrit of 25%. Which of the following is the most
appropriate next step in diagnosis?
a) Colonoscopy
b) UGI endoscopy
c) KUB (x-ray film showing the kidney, ureters and bladder)
d) Open laparotomy
e) Selective angiography
The answer is: Colonoscopy
80) Early signs of premature labor include all of following EXCEPT:
a) Increased vaginal discharge
b) Increased uterine contraction
c) Low back pain
d) Cervical dilatation to 4 cm
e) Worsening pelvic pressure
The answer is: Increased vaginal discharge
81) Ultraviolet radiation is used for the disinfection of which of the following?
a) Orthopedics operating theatre
b) Piped water supplies
c) Anesthesia equipment

54
EMLE Trial Exams December 2020 (1000 Qs) Version (1.0) 22/12/2020

d) Rigid bronchoscopy
e) Prostheses
The answer is: Piped water supplies
82) A teenage boy falls from his bicycle and is run over by a truck. On arrival in the emergency room (ER), he
is awake and alert and appears frightened but in no distress. The chest radiograph suggests an air-fluid
level in the left lower lung field and the nasogastric tube seems to coil upward into the left chest. Which
of the following is the next best step in his management?
a) Placement of a left chest tube
b) Thoracotomy
c) Laparotomy
d) Esophagogastroscopy
e) Diagnostic peritoneal lavage
The answer is: Laparotomy
83) A 72-year-old man has noticed a dull aching pain in the left upper quadrant. On examination, a smooth
moderately enlarged spleen was felt. There is a past medical history of rheumatoid arthritis and
hypertension. Routine blood tests show a normocytic anaemia and low white cell count. The most likely
cause for the clinical findings:
a) cutaneous leishmaniasis
b) Budd–Chiari syndrome
c) myeloma
d) Felty’s syndrome
e) acute lymphoblastic leukaemia
The answer is: Felty’s syndrome
84) A frail 89-year-old man has had a large pressure sore on the sacrum for the past 2 months. Medical
history includes type 2 diabetes mellitus and multi-infarct dementia. He is incontinent of stool and urine.
Which of the following factors is the most important contributor to the development of the pressure
ulcer in this patient?
a) Fecal and urinary incontinence
b) Inadequate cushioning on the wheelchair
c) Malnutrition
d) Poorly controlled diabetes mellitus
e) Prolonged bed rest
The answer is: Prolonged bed rest
85) A 16-year-old patient (gravida 1) is seen for the first time when 16 weeks pregnant. History and
examination are entirely normal except for a large solid mass in the posterior pelvis. It is slightly
lobulated, immobile, and smooth and cannot be completely palpated. There is some question as to
whether or not it will obstruct labor.Which of the following is the most likely diagnosis?*
a) anterior meningomyelocele
b) pelvic kidney
c) carcinoma of the bowel
d) sacculated uterus
e) idiopathic retroperitoneal fibrosis
The answer is: pelvic kidney
86) What is the best treatment of toxic multinodular goiter?
a) Propylthiouracil
b) Carbimazole
c) Radioactive iodine

55
EMLE Trial Exams December 2020 (1000 Qs) Version (1.0) 22/12/2020

d) Thyroidectomy
e) Radical neck dissection
The answer is: Radioactive iodine
87) Which of the following tumour markers are most commonly associated with pancreatic cancer?
a) Carcinoembryonic antigen
b) Cancer antigen (CA)125
c) CA19–9
d) α-fetoprotein (AFP)
e) β-human chorionic gonadotrophin
The answer is: CA19–9
88) Which of these drugs don’t cross the placenta?
a) Heparin
b) Warfarin
c) Tetracycline
d) Degoxin
e) None of the above
The answer is: Heparin
89) Which of the following is the most common risk factor associated with aortic dissection ? *
a) Smoking
b) Atherosclerosis
c) Marfan syndrome
d) Hypertension
e) Bicuspid aortic valve
The answer is: Hypertension
90) Cytokines are endogenous signals that do not stimulate:#
a) Local cell proliferation within the wound.
b) The central nervous system to initiate fever.
c) The production of “acute-phase proteins”.
d) Hypoferremia.
e) Septic shock
The answer is: Septic shock
91) A 42-year-old male presents with a swollen, erythematous, and tender left knee. He is in a long-term
monogamous relationship. A concomitant history of recurrent renal stones in this man suggests a
diagnosis of:
a) Reactive arthritis
b) Systemic lupus erythematosus
c) Rheumatoid arthritis
d) Gout
e) Nongonococcal septic arthritis
The answer is: Gout
92) A 53-year-old male complains of fatigue, dyspnea, and orthopnea. Which one of the following would
have the highest specificity for heart failure?*
a) Ankle edema
b) A third heart sound (S3 gallop)
c) Cardiomegaly on a chest radiograph
d) Decreased B-type natriuretic peptide (BNP)
e) Fine crackles

56
EMLE Trial Exams December 2020 (1000 Qs) Version (1.0) 22/12/2020

The answer is: A third heart sound (S3 gallop)


93) A 29-year-old man presents with a 4-week history of polyuria and extreme thirst. The patient denies
difficulty voiding, hesitancy or haematuria, although the urine is very dilute. The patient does not
believe he has lost any weight and maintains a good diet. No findings are found on urine dipstick. The
most appropriate investigation is:
a) Serum osmolality
b) Fasting plasma glucose
c) Urinary electrolytes
d) Magnetic resonance imaging (MRI) scan of the head
e) Water deprivation test
The answer is: Serum osmolality
94) A 29-year-old man is brought to the ED with a gunshot wound to his left thigh and in pain. Upon arrival,
his BP is 140/80 mmHg, HR is 95 b/m, RR is 18 b/m, and oxygen saturation is 99% on room air. All of his
distal pulses are +, and there is no gross deformity of his left leg. You turn to the nurse and say “Please
give 50 mcg of fentanyl IV.” The nurse repeats back to you, “I am going to give 50 mcg of fentanyl IV.”
You reply, “Yes, thank you.” What type of communication is demonstrated between you and the nurse
in this scenario?
a) Positive-feedback communication
b) Closed-loop communication
c) Redundant communication
d) Situation Background Assessment Recommendation (SBAR) communication
e) Team huddle communication
The answer is: Closed-loop communication
95) A 15-year-old boy was brought to chest clinic with attacks of breathing difficulty with whistling sounds.
These symptoms occurred frequently especially after dust exposure. His younger sister had the same
condition. Clinical examination revealed scattered rhonchi all over the chest. Which of the following
drugs is a risk factor for this condition?#
a) Calcium channel blocker
b) Metoclopramide
c) NSAID
d) Paracetamol
e) Penicillin
The answer is: NSAID
96) You are examining a child in the ER after history of head trauma you assessed his Glasgow coma score
found he is 13 what is the best advice?*
a) Neurological examination
b) Urgent CT
c) Ortho assessment
d) Ventilate
e) Urgent MRI
The answer is: Urgent CT
97) Fundal height, part of the obstetric examination, is taken from the top of the symphysis pubis to the top
of the fundus. How is it measured?
a) by calipers, approximating the week of gestation
b) in inches, approximating the lunar month of gestation
c) in centimeters and divided by 3.5, approximating the lunar months of gestation
d) in centimeters, approximating the weeks of gestation beyond 22 weeks

57
EMLE Trial Exams December 2020 (1000 Qs) Version (1.0) 22/12/2020

e) by calipers in centimeters, prognosticating the fetal weight


The answer is: in centimeters, approximating the weeks of gestation beyond 22 weeks
98) BCG dose to a child is
a) 0.2 ml
b) 0.5 ml
c) 0.1 ml
d) 0.3 ml
e) 0.8 ml
The answer is: 0.1 ml
99) A 27-year-old woman complains of a fishy odor and a vaginal discharge. The speculum examination
reveals an erythematous vagina and punctuations of the cervix. Which is the MOST LIKELY diagnosis?
a) Candial vaginitis.
b) Trichomonal vaginitis.
c) Bacteria vaginosis.
d) Human papilloma virus.
e) Herpes simplex virus.
The answer is: Trichomonal vaginitis.
100) The followings ore complications of fracture except:
a) Neurogenic shock.
b) Stroke.
c) Fat embolism.
d) Renal failure.
e) Sepsis.
The answer is: Stroke

58
EMLE Trial Exams December 2020 (1000 Qs) Version (1.0) 22/12/2020

Exam 4
1) A 64-year-old woman complains of right calf pain and swelling. She recently underwent an
uncomplicated left hemicolectomy for diverticular disease. A duplex ultrasound confirms the presence
of deep vein thrombosis (DVT) of the calf. Which of the following is the most appropriate initial
management of this patient?
a) Pneumatic compression stockings
b) Warfarin
c) Heparin
d) Thrombolysis
e) Inferior vena cava filter
The answer is: Heparin
2) Which of the following medications that he takes may be associated with increased risk of deep vein
thrombosis?*
a) Fenofibrates
b) Atorvastatin
c) Ramipril
d) Bisoprolo
e) Indapamide
The answer is: Fenofibrates
3) Which of the following medications, when given before & during pregnancy may help to protect neural
tube defects?
a) Vitamin B6.
b) Iron.
c) Folic acid.
d) Zinc.
e) Magnesium
The answer is: Folic acid
4) A 61-year-old man presents to the ED with chest wall pain after a high-speed MVC. He is speaking in full
sentences, breath sounds are equal bilaterally, and his extremities are well perfused. His BP is 150/75
mm Hg, HR is 9 beats/minute, and oxygen saturation is 97% on room air. Chest radiography reveals
fractures of the seventh and eighth ribs of the right anterolateral chest. With no other injuries. Which of
the following is the most appropriate treatment for this patient’s rib fractures?
a) Apply adhesive tape to the chest wall perpendicular to the rib fractures
b) Insert a chest tube into the right hemithorax
c) Send the patient to the OR for surgical fixation
d) Provide analgesia and observe
e) Observe the patient in the ED for 6 hours
The answer is: Provide analgesia and observe
5) A 5-year-old male presents with right hip paiA 5-year-old male presents with right hip pain. There is no
history of trauma. Which of the following is the most likely cause? n. There is no history of trauma.
Which of the following
a) Femoral neck fracture
b) Osteogenesis imperfecta
c) Transient synovitis
d) Slipped capital femoral epiphysis (SCFE)
e) Legg–Calve–Perthes disease

59
EMLE Trial Exams December 2020 (1000 Qs) Version (1.0) 22/12/2020

The answer is: Transient synovitis


6) A 35-year-old man who presents with a neck mass is found to have a serum calcium level of 11.8 mg/dL
and periodic elevation of his blood pressure. Extensive workup reveals the presence of a medullary
carcinoma of the thyroid, a pheochromocytoma, and hyperplasia of the parathyroid glands. Which of
the following is the most likely diagnosis?
a) Multiple endocrine neoplasia syndrome type 1
b) Multiple endocrine neoplasia syndrome type 2A
c) Multiple endocrine neoplasia syndrome type 2B
d) Polyglandular syndrome type I
e) Polyglandular syndrome type II
The answer is: Multiple endocrine neoplasia syndrome type 2A
7) The 16-year-old daughter consents to medication, but the mother refuses What is the most appropriate
action to do?**
a) Write the prescription
b) Do not write the prescription
c) Postpone the prescription
d) Convince the daughter
e) Report the event to the police
The answer is: Do not write the prescription
8) Which of the following is most effective in reducing mortality from acute MI?
a) Metoprolol
b) Aspirin
c) Nitroglycerin
d) Abciximab
e) Morphine
The answer is: Aspirin
9) A 6-year old is admitted to the ward for investigation of glomerulonephritis, as he has peripheral
oedema and blood in his urine. He has been treated with antibiotics for tonsillitis three weeks prior.he is
otherwise well, with no rashes, joint pain or abdominal symptoms.What is the most likely cause of his
glomerulonephritis?
a) Post-streptococcal glomerulonephritis (PSGN)
b) Berger's disease (IgA nephropathy)
c) Haemolytic uraemic syndrome (HUS)
d) Henoch-Scholein purpura (HSP)
e) Goodpasture syndrome
The answer is: Post-streptococcal glomerulonephritis (PSGN)
10) A 14-year-old girl is seen for her first prenatal visit at 34 weeks’ gestation by menstrual history. On
examination her BP is 148/96 mm Hg and her fundus measures 33 cm. Her urine dipstick is 1+ positive
for protein. Which of the following is the most likely diagnosis?
a) hypertensive disease with superimposed preeclampsia
b) mild eclampsia
c) third-trimester pregnancy
d) preeclampsia
e) chronic hypertension
The answer is: preeclampsia

60
EMLE Trial Exams December 2020 (1000 Qs) Version (1.0) 22/12/2020

11) A 46 -year-old engineer taking hormone replacement therapy for the past 10 years, who has smoked all
her working life, develops warm, tender swelling of her calf and thigh at the end of her holiday . She
relates this to an insect bite 3 days earlier choose the single most likely diagnosis
a) Cellulitis
b) Deep vein thrombosis
c) Lipodermatosclerosis
d) Lymphoedema
e) Raynaud’s phenomenon
The answer is: Deep vein thrombosis
12) Presentations of urinary stones may be the following except:
a) Renal pelvic stone pain restricted to loin.
b) Upper ureter stone has no radiating pain.
c) Bladder neck stone pain radiating to neck of penis.
d) Urinary stone may be asymptomatic especially triphosphate stone
e) none of the above
The answer is: Upper ureter stone has no radiating pain.
13) In acute ischaemia pain is more prominent in:
a) At the site of obstruction
b) Most proximal part of the limb
c) Most peripheral part of the limb
d) All of the above
e) non of the above
The answer is: Most peripheral part of the limb
14) A newborn has failed to pass meconium in the first 48 hours and is reluctant tofeed. On examination the
abdomen is distended. Plain abdominal radiographyshows distended loops of bowel with absence of air
in the rectum.Themost likely cause for the clinical findings
a) Wilm’s tumour
b) Hirschsprung’s disease
c) pyloric stenosis
d) necrotizing enterocolitis
e) infantile colic
The answer is: Hirschsprung’s disease
15) A 22-year-old female presents to the ED after being ejected from a vehicle following a high-speed MVC.
Upon arrival, BP is 85/55 mm Hg and HR is 141 b/m. IV fluids are being administered. After 3 L of
crystalloid fluid, the patient’s BP is 83/57 mm Hg. Which of the following is most appropriate regarding
management of this hypotensive trauma patient?
a) It is important to wait for fully cross-matched blood prior to transfusion
b) Whole blood should be used rather than packed red blood cells (RBCs)
c) Blood transfusion should begin after 4 L of crystalloid infusion
d) Type O blood that is Rh-negative should be transfused
e) Type O blood that is Rh-positive should be transfused
The answer is: Type O blood that is Rh-negative should be transfused
16) If the patient has hyperparathyroidism, which of the following will the infant possibly be at increased
risk for postpartum?
a) hyaline membrane disease
b) tetany
c) coma

61
EMLE Trial Exams December 2020 (1000 Qs) Version (1.0) 22/12/2020

d) hyperglycemia
e) malabsorption syndrome
The answer is: tetany
17) A tanned 66-year-old man presents with an ulcerated lesion on the nose, with a rolled edge. Select the
type of ulcer that has being described
a) basal cell carcinoma
b) Bowen’s disease
c) squamous cell carcinoma
d) apthous ulcer
e) Marjolin’s ulcer
The answer is: basal cell carcinoma
18) A 19-year-old woman without prenatal care (gravida 1, para 0) in the third trimester of pregnancy
arrives in the emergency department. She has presented because of headache and visual change. While
being examined, she had a convulsionAfter she awakes from her seizure and the postictal state, she
complains of blurry vision. What is the most likely finding upon fundoscopic examination?
a) exudates and hemorrhage
b) loss of corneal curvature
c) retinal edema
d) arteriolar spasm
e) macular degeneration
The answer is: arteriolar spasm
19) Which of the following pathologic stones are most likely to be visible on plain radiographs of the
abdomen?
a) Appendicoliths
b) Phleboliths
c) Gallstones
d) Renal stones
e) Primary common duct stones
The answer is: Renal stones
20) A patient with suspected cholestasis of pregnancy develops a slight hyperbilirubinemia and slight
elevation of SGOT. You obtain serum bile salts that are positive, confirming the diagnosis. Relief of the
pruritus may be obtained by which of the following?
a) amitriptyline
b) bland diet
c) oral H2 blockers
d) cholestyramine.
e) mild diuretic therapy
The answer is: cholestyramine.
21) A 45-year-old female presents with intermittent epigastric pain and nausea for several weeks. She
denies fever, diarrhea, cough, and chest pain. She does not take medications, drink alcohol, or smoke.
She has had an extensive workup including CT scan of the abdomen and pelvis, gall bladder ultrasound,
and cardiac stress testing, all of which were normal. You suspect gastritis. Which of the following is the
most likely cause?
a) Helicobacter pylori infection
b) NSAID use
c) Alcohol use
d) Peppermint gum chewing

62
EMLE Trial Exams December 2020 (1000 Qs) Version (1.0) 22/12/2020

e) Radon exposure
The answer is: Helicobacter pylori infection
22) Features of severe folate deficiency include:
a) Diarrhea
b) Glossitis
c) Microcytic anemia
d) Increased total iron binding capacity
e) Hyper-segmentation of neutrophils
The answer is: Hyper-segmentation of neutrophils
23) What is the best treatment of a newly diagnosed depressed patient came to the outpatient clinic due to
severe impairment of functional status?
a) phenelzine
b) Citalopram
c) ECT
d) Buspiron
e) Bupropion
The answer is: Citalopram
24) In Spaulding classification, which of the following should undergo disinfection?
a) Stethoscopes
b) Respiratory masks
c) Rigid endoscopy
d) Prostheses
e) Surgical instruments
The answer is: Respiratory masks
25) Thin profuse frothy yellowish offensive vaginal discharge is characteristic of:
a) Candidal vaginitis
b) Trichomonas vaginitis
c) Cenile vaginitis
d) Gonorrhea
e) Syphills
The answer is:: Trichomonas vaginitis
26) Which of the following is a false statement?
a) Angioneurotic edema is always a benign condition
b) Atopic persons usually have a family history of atopy even in other anatomic sites
c) Blood eosinophilia >700/mm3 is suggestive of atopy
d) Breastfeeding is recommended for infants born to atopic parents
e) Avoidance of the precipitating allergen is the corner stone of management
The answer is:: Angioneurotic edema is always a benign condition
27) What is the most common cause of lower gastrointestinal bleeding in children?
a) Anal fissure
b) Hemorrhoids
c) Henoch–Schönlein purpura
d) Food allergy
e) Meckel diverticulum
The answer is: Anal fissure
28) Anaphylaxis is manifested by all except:#^
a) Tingling sensation

63
EMLE Trial Exams December 2020 (1000 Qs) Version (1.0) 22/12/2020

b) Inspiratory stridor
c) Chest tightness
d) Hypotension
e) Hypertension
The answer is: Hypertension
29) During a repair of a primary inguinal hernia, you are asked to name the nerve that is located within the
spermatic cord. Which of the following is the nerve that is found within the spermatic cord?#
a) Ilioinguinal nerve
b) Genitofemoral nerve
c) Genital branch of the genitofemoral nerve
d) Iliohypogastric nerve
e) Lateral femoral cutaneous nerve
The answer is: Genital branch of the genitofemoral nerve
30) A 40-year-old male receives a stab wound to the left flank. Which of the following is true regarding his
management?
a) A triple contrast CT scan should be performed.
b) If the diagnostic peritoneal lavage has less than 10 mL of free flowing blood, he is unlikely to have a
significant injury.
c) If the FAST exam is negative, he is unlikely to have a significant injury.
d) Local wound exploration should be performed at the bedside with proper lighting and adequate
anesthesia.
The answer is: A triple contrast CT scan should be performed.
31) The patient should be admitted for serial exams and hematocrits.
a) The INCORRECT statement regarding luteinizing hormone:
b) It stimulates androgen production in males
c) It stimulates luteinization of the granulose cells
d) Its plasma concentrations are decreased in ovarian failure
e) It is a glycoprotein
f) It is best measured on day 3 of the cycle in infertility workup
The answer is: Its plasma concentrations are decreased in ovarian failure
32) Which of the following is the most appropriate medication to induce rapid reduction in blood pressure
in hypertensive crisis ?*
a) Nifedipine
b) Captopril
c) Furosemide diuretic
d) Nitroprusside.
e) Clonidine
The answer is: Nitroprusside.
33) A 22-year-old man presents with a 6-month history of non-bloody diarrhea, malaise, recurrent
abdominal cramps, and temperature to 38.5 oC. At this time, he is afebrile. Palpation causes local
tenderness without guarding in the right lower quadrant of the abdomen. Oral ulcers are also noted. An
upper gastrointestinal series with small bowel follow-through reveals sharply demarcated stenotic
segment in the terminal ileum. Which of the following is the most likely diagnosis?
a) Celiac disease
b) Chronic appendicitis
c) Crohn’s disease
d) Diverticulitis

64
EMLE Trial Exams December 2020 (1000 Qs) Version (1.0) 22/12/2020

e) Irritable bowel syndrome


The answer is: Crohn’s disease
34) A 79-year- old female with six month history of rectal bleeding admitted to surgical ward to have a large
bowel tumour surgically removed .She appears to be severe ly short of breath and extremely tired
.Respiratory rate 24 bpm , Hb 6.8g/dL , pH 7.49 , Pco2 25mmHg , Po2 89mmHg , Bicarb 22mmol/L , SP02
99.8% , K 3.8mmol/L , Na 138mmol/L , Cl 96mmo/L , normal glucose , normal lactate and base excess.
Which of the following describe her acid base state ?*
a) Uncompensated respiratory alkalosis
b) Compensated respiratory alkalosis
c) Uncompensated metabolic alkalosis
d) Compensated metabolic alkalosis
e) Normal acid base state
The answer is: Uncompensated respiratory alkalosis
35) An 85-year-old woman is being transferred to an acute rehabilitation facility following a hospital
admission for hip replacement surgery. Postoperatively during her hospital stay, she is started on deep
vein thrombosis (DVT) prophylaxis medication with plans to continue the medication upon discharge.
The intern and nurse who are discharging the patient fail to convey this new medication to the receiving
treatment team at the rehabilitation center. The patient is not continued on her anticoagulation
medication and sustains a DVT, leading to a fatal pulmonary embolus 3 weeks after transfer. Which of
the following actions will facilitate quality improvement and the prevention of a similar error in the
future?**
a) Determine which staff member(s) failed to order the medication
b) Develop a process to increase the use of medication reconciliation
c) Send a memo to all staff about the importance of DVT prophylaxis
d) Educate patients about the dangers of DVT following hip surgery
e) Conduct monthly audits to monitor medication errors at transitions of care
The answer is: Develop a process to increase the use of medication reconciliation
36) A 55-year-old male patient with no comorbid illness presented with hematemesis. His hemoglobin was 5
gm/dl. He received blood transfusion. Which of the following is the target hemoglobin level required?
a) Hemoglobin 5 – 6 gm/dl
b) Hemoglobin 7 – 8 gm/dl
c) Hemoglobin 9 -10 gm/dl
d) Hemoglobin 11 -12 gm/dl
e) Hemoglobin 13 – 14 gm/dl
The answer is: Hemoglobin 9 -10 gm/dl
37) An 8-month-old child has vomiting and screaming episodes for 12 hours. Physical examination reveals a
sausage-shaped mass in the right upper quadrant. Which of the following would be most useful?
a) passage of nasogastric tube
b) examination of a stool specimen for ova and parasites
c) blood culture
d) abdominal ultrasound
e) barium enema study
The answer is: barium enema study
38) A 76-year-old hypertensive smoker with a 4-month history of backache presents with acute excruciating
epigastric pain radiating to the back. O/E: blood pressure is 85/50mmHg, pulse is 130bpm (right femoral
pulse is absent, while left femoral pulse is weak), marked abdominal tenderness and rigidity.what is the
cause of his abdominal pain?*

65
EMLE Trial Exams December 2020 (1000 Qs) Version (1.0) 22/12/2020

a) Acute appendicitis
b) Acute pancreatitis
c) Ascending cholangitis
d) Acute MI
e) Ruptured abdominal aortic aneurysm
The answer is: Ruptured abdominal aortic aneurysm
39) A 70-year-old man with 5 weeks of intermittent left iliac fossa pain, anorexia, and anergia. He has lost
2kg of weight. He has had Low-grade fevers for the last 3 nights. There is a tender mass in the left lower
quadrant. choose the single most likely abdominal mass
a) Appendix mass
b) Colonic carcinoma
c) Lymphoma
d) Palpable bladder
e) Pelvic abscess
The answer is: Pelvic abscess
40) Causes of first trimester abortion*^
a) Chromosomal abnormalities
b) Cervical incompetence
c) Bicornuate uterus
d) Gestational hypertension
e) Pre-eclampsia
The answer is: Chromosomal abnormalities
41) A 42 years old engineer was presented with epigastric pain. A biopsy of the antrum of the stomach
reveals numerous lymphocytes and plasma cells within the lamina propria, which is of normal thickness.
There are also scattered neutrophils within the glandular epithelial cells. A Steiner silver stain from this
specimen is positive for a small, curved organism. These histologic changes are most consistent with
infection by which one of the following organisms?
a) Enteroinvasive Escherichia coli
b) Enterotoxigenic E. coli
c) Helicobacter pylori
d) Salmonella typhi
e) Shigella species
The answer is: Helicobacter pylori
42) 30-year-old male presents Right knee pain. On examination, his right knee is swollen, red and warm.
Joint aspiration was done and showed increased WBCS and needle shaped crystals. Which is the most
likely diagnosis?#
a) Rheumatoid arthritis
b) Gout
c) Osteoarthritis
d) Septic arthritis
e) Pseudogout
The answer is: Gout
43) A 32-year-old male presents to the outpatient clinic complaining of recurrent attacks of starting that is
noticed by others. EEG shows absence seizures. Which is the best treatment?#
a) Lorazepam
b) phenytoin
c) Valproate

66
EMLE Trial Exams December 2020 (1000 Qs) Version (1.0) 22/12/2020

d) Carbamazepine
e) Ethosuximide
The answer is: Ethosuximide
44) A 30-year-old taxi driver complains of tenderness from an area in the midline ofthe natal cleft about 4
cm above the anus. This problem has been remitting andrecurring for 2 years and has started to
discharge today for the first time. The most likely cause for the clinical findings is :
a) fissure-in-ano
b) perianal warts
c) proctalgia fugax
d) fistula-in-ano
e) pilinoidal abscess
The answer is:: pilinoidal abscess.
45) A patient is experiencing an arrest of decent. During the evaluation one can feel that it is a vertex
presentation with the sagittal suture transverse or oblique but closer to the symphysis than the
promontory. What is this specific condition called?
a) posterior asynclitism
b) anterior asynclitism
c) internal rotation
d) extension
e) restitution
The answer is: anterior asynclitism
46) A 27-year-old woman presented with a 5-year history of abdominal bloating, weight gain and
constipation, as well as recent anal discomfort with no bleeding. The bloating was eased by bowel
opening, which occurred every 5–7 days. Stool consistency varied from pellet-like to semi-formed. She
strained at stool on every occasion.What is the mostly likely finding on rectal examination?*
a) A tear in the anal skin is normal with chronic straining, and unlikely to be a cause of her pain
b) Contraction of the internal anal sphincter is normal on bearing down
c) Low resting anal pressure excludes an anal fissure
d) Rectal prolapse is possible despite her young age
e) The presence of stool in the rectum confirms the diagnosis of faecal impaction
The answer is: Rectal prolapse is possible despite her young age
47) Which of the following listings correctly ranks contraceptive methods in terms of decreasing effec-
tiveness#
a) Oral contraceptives, IUCD, spermicides, diaphragm, rhythm
b) Oral contraceptives, diaphragm, IUCD, spermicides, rhythm
c) IUCD, oral contraceptives, diaphragm, spermicides, rhythm
d) Rhythm, oral contraceptives, IUCD, diaphragm, spermicides
e) Oral contraceptives, IUCD, diaphragm, spermicides, rhythm
The answer is: Oral contraceptives, IUCD, diaphragm, spermicides, rhythm
48) A 34-year-old woman is brought to the ED from a kitchen fire. Upon arrival, her BP is 110/75 mmHg, HR
is 115 b/m, RR is 18 b/m, and pulse oxygenation is 98% on room air. She weighs 70 kg. She has tender
burns with blisters to her bilateral forearms. Approximately what percentage body surface area is
burned in this patient?
a) 0.18
b) 0.045
c) 0.09
d) 0.15

67
EMLE Trial Exams December 2020 (1000 Qs) Version (1.0) 22/12/2020

e) 0.2
The answer is:: 0.09
49) What is the best diagnosis of hard non tender lymphadenopathy?*
a) Lymph node biopsy
b) CT chest , abdomen and pelvis
c) MRI abdomen and pelvis
d) Colonoscopy
e) UGI endoscopy
The answer is: Lymph node biopsy
50) The acid-base disturbance which commonly accompanies shock is:
a) Metabolic acidosis
b) Metabolic alkalosis
c) Respiratory acidosis
d) Respiratory alkalosis
e) mixed alkalosis
The answer is: Metabolic acidosis
51) A 34-year-old woman (gravida 3, para 2) at 35 weeks’ gestation complains of sharp, excruciating pain in
the right flank radiating into her groin. No chills or fever have been noted. The pain resolved shortly
after the patient was seen. Urinary analysis reveals numerous red blood cells (RBCs), some WBCs, and
no bacteria. WBC and hematocrit are normal. Which of the following laboratory tests should be
performed?
a) serum iron
b) serum glutamic oxaloacetic transaminase (SGOT)
c) tine test
d) bilirubin
e) serum calcium
The answer is: serum calcium
52) A 29-year-old woman presents with sharp left loin and left upper quadrant painradiating to the groin.
she is not jaundiced.
a) diverticulosis
b) duodenal ulcer
c) renal colic
d) diverticulitis
e) mesenteric adenitis
The answer is: renal colic
53) From highest to lowest mortality, what are the three most common causes of cancer death in men?*
a) Colorectal, lung, testicular
b) Lung, colorectal, testicular
c) Lung, prostate, colorectal
d) Prostate, colorectal, lung
e) Prostate, lung, colorectal
The answer is: Lung, prostate, colorectal
54) A 3-year-old boy is brought to the emergency room after spilling bleach onto his lower extremities. He is
diagnosed with a chemical burn and all involved clothing are removed. In addition to resuscitation,
which of the following is the most appropriate initial management of this patient?
a) Treatment of the burn wound with antimicrobial agents.
b) Neutralize the burn wound with weak acids.

68
EMLE Trial Exams December 2020 (1000 Qs) Version (1.0) 22/12/2020

c) Lavage of the burn wound with large volumes of water.


d) Wound debridement in the operating room.
e) Treatment of the burn wound with calcium gluconate gel.
The answer is: Lavage of the burn wound with large volumes of water.
55) Which of the following is the most common cause of pediatric cardiopulmonary arrest?
a) Cardiac arrhythmias
b) Metabolic abnormalities
c) Overwhelming infections
d) Trauma
e) Respiratory problems
The answer is: Respiratory problems
56) The following occur in childhood asthma except
a) Cough is predominant symptom
b) Expiratory sibilant rhonchi
c) Increased airway resistance
d) Increase eosinophils
e) Increase blast cells
The answer is: Increase blast cells
57) A 45-year-old woman presents to your office complaining of fatigue and pruritus, which is
predominantly of the palms and soles. The physical examination shows bilateral xanthelasmas and
hepatomegaly. Labs show bilirubin as normal, AST/ALT as normal, cholesterol as elevated, and alkaline
phosphatase also elevated. Sonogram shows hepatomegaly. Which test would be the most appropriate
to guide us to a diagnosis of her disease?
a) Antimitochondrial antibody
b) Antinuclear antibody
c) Anti-smooth muscle antibody
d) Hepatitis B surface antigen
e) Hepatitis C antibody
The answer is: Antimitochondrial antibody
58) The etiologic agent for bronchiolitis is:#
a) Parainfluenza virus
b) Streptococcus viridians
c) Pneumococcus
d) Haemophilus influenza
e) Respiratory syncytial virus
The answer is: Respiratory syncytial virus
59) What is the most common cause of thyroid disease and is a predisposing factor for the development of
lymphoma?
a) De Quervian thyroiditis
b) Riedel thyroiditis
c) Hashimoto thyroiditis
d) Grave’s disease
e) Amiodarone
The answer is: Hashimoto thyroiditis
60) A 32-year-old male presents to the emergency department after he fall to the ground. He was
unconscious for 10 minutes and was confused for 1 hour after regaining consciousness. He had urine and
stool incontinence as well as tongue biting. Which is the most likely diagnosis?

69
EMLE Trial Exams December 2020 (1000 Qs) Version (1.0) 22/12/2020

a) Generalized myoclonic seizure


b) Generalized atonic seizure
c) Simple partial seizure
d) Complex partial seizure
e) Generalized tonic-atonic seizure
The answer is: Generalized atonic seizure
61) Which is the most sensitive test for diagnosing systemic lupus?#
a) Antinuclear antibodies
b) Rheumatoid factor
c) Anti Jo
d) Anti Ro
e) Anti CCP
The answer is: Antinuclear antibodies
62) A 4 years old girl diagnosed as ITP, her platelet count is 30,000/dl with no active bleeding or mucous
membrane purpura. The next step in her management is:
a) Steroids
b) IVIG
c) Anti-D
d) observation and follow up
e) splenectomy
The answer is: observation and follow up
63) Ambulance bring in a 54-year-old man who was found down in his apartment by his wife. He is
successfully intubated in the field and para- medics are currently performing (CPR). In the ED during the
CPR, the cardiac monitors attached (sinus bradycardia). but no pulses are palpable. On examination you
appreciate bilateral breath sounds with BVM. In addition to CPR, which intervention should be
performed next?
a) Administer 1 ampule of sodium bicarbonate
b) Administer epinephrine every 3 to 5
c) Administer 1 ampule of D50 (dextrose)
d) Place a left-sided chest tube
e) Perform pericardiocentesis
The answer is: Administer epinephrine every 3 to 5
64) A rapid correction of fluid in a child with hypernatremic dehydration results in:#
a) IVH
b) Subdural hemorrhage
c) Cerebral cells atrophy
d) Cerebral cells swelling
e) PVL (periventricular leukomalacia
The answer is: Cerebral cells swelling
65) A 38-year-old pregnant woman presents to the ED with vaginal bleeding and abdominal pain radiating to
the back. She is 25 weeks by dates and has had an uncomplicated pregnancy with routine prenatal care.
She admits to smoking one pack of cigarettes per day and otherwise has no significant history. Her
uterus is firm and tender on examination and there is bright red blood oozing from the cervical os. The
most likely diagnosis is:
a) Appendicitis
b) Placenta previa
c) Vasa previa

70
EMLE Trial Exams December 2020 (1000 Qs) Version (1.0) 22/12/2020

d) Fibroid degeneration
e) Abruptio placentae
The answer is: Abruptio placentae
66) Which is the best treatment of acute flare of systemic lupus nephritis?#
a) Methotrexate
b) Steroids and cyclophosphamide
c) Hydroxychloroquine
d) Steroids only
e) Cyclophosphamide only
The answer is: Steroids and cyclophosphamide
67) A 40 year old man, 18 hours after a gastrectomy, suddenly becomes hypotensive with a blood pressure
of 80/60, tachycardia of 110/min, CVP +2 cmsH2O and a temperature of 39C. The likely diagnosis is:
a) Ruptured anastomosis
b) Haemorrhage
c) Acute gastric dilatation
d) Pulmonary embolism
e) Septicaemia
The answer is: Ruptured anastomosis
68) A 31-year-old man is admitted to the hospital because of melena. He takes daily nonsteroidal anti-
inflammatory agents for pain in his knee. He observed copious bright red blood per rectum. On physical
examination, pulse is 110 per minute and peripheral pulses are faint but present. Blood pressure is
90/60 mmHg and extremities are cool. An intravenous line is placed. Which of the following is the most
appropriate next step in management?
a) Begin parenteral administration of large volumes of normal saline solution
b) IV proton pump inhibitors
c) Order an urgent hematocrit level
d) Order an urgent type and cross match for blood
e) Place two additional large bore peripheral IV catheters
The answer is: Begin parenteral administration of large volumes of normal saline solution
69) What is the most common cause of anesthetic death in obstetrics :
a) Failed intubation.
b) Hemorrhage.
c) Stroke.
d) Reaction to medication.
e) Aspiration pneumonitis
The answer is: Aspiration pneumonitis
70) 30-year-old male presents Right knee pain. On examination, his right knee is swollen, red and warm.
Temperature is 38.5. Which is the best next step in management?#
a) Knee Xray
b) Allopurinol
c) Joint aspiration
d) Antibiotics
e) hypercalcemia workup
The answer is: Joint aspiration
71) Subconjunctival bleed with no posterior border indicates fracture of which bone:
a) Maxilla.
b) Mandible.

71
EMLE Trial Exams December 2020 (1000 Qs) Version (1.0) 22/12/2020

c) Zygoma.
d) Nasal.
e) Skull.
The answer is: Zygoma.
72) A 68-year-old female presents with a chief complaint of severe weakness, general malaise, and fever.
The patient is hypotensive, and is found to have pneumonia on chest x-ray. The patient received broad-
spectrum antibiotics but remains hypotensive despite 5 L of fluids. Which of the following is the best
next step?
a) Norepinephrine
b) Vasopressin
c) Dopamine
d) Dobutamine
e) Phenylephrine
The answer is: Norepinephrine
73) A 5-year-old boy presents to the assessment unit with a purpuric rash on his legs and buttocks, with
associated swelling and pain in his right knee, resulting in a limp. He has had a recent viral upper
respiratory-tract infection(URTI) but is otherwise fit and well. A urine dip reveals haematuria only. What
is the most likely diagnosis?
a) Haemophilia
b) Henoch-Scholein purpura (HSP)
c) Septic arthritis
d) Transient synovitis
e) Urinary tract infection
The answer is: Henoch-Scholein purpura (HSP)
74) Which of the following would not be included in the differential diagnosis of acute croup in children?
a) Diphtheria
b) Foreign body inhalation
c) Parainfluenza viral infection
d) Bronchial asthma
e) Laryngeal papilloma
The answer is: Bronchial asthma
75) In cases of pruritus vulva:#
a) Radical vulvectomy is indicated
b) Hydrocortisone is contraindicated
c) Recurrent symptoms are uncommon
d) Common after menopause
e) Antihistaminics are ineffective for treatment
The answer is: Common after menopause
76) Vacuum extraction (ventouse):
a) Causes more maternal birth canal injuries than the forceps.
b) Can be used when the cervix is 7 cm dilated.
c) Can be applied when the vertex is minus 2 station.
d) Can cause Cephalohematoma to the baby.
e) Can be used in face presentation.
The answer is: Can cause Cephalohematoma to the baby.
77) The main causative agent of bronchiolitis is:
a) RSV

72
EMLE Trial Exams December 2020 (1000 Qs) Version (1.0) 22/12/2020

b) Adenovirus
c) Mycoplasma
d) Herpes virus
e) H1N1
The answer is: RSV
78) A 7 year old male with glucose-6-phosphate dehydrogenase deficiency presents with tiredness and is
noticed to be jaundiced. These features have developed since he developed a mild chest infection one
week ago. Which one of the following is the most likely laboratory finding?
a) Haemoglobinuria
b) low mean cell volume
c) Positive direct antiglobulin test
d) Reduced reticulocyte count
e) Spherocytes present on blood film
The answer is: Haemoglobinuria
79) A 61-year-old man, with an 8-year history of hepatitis C infection and well-documented cirrhosis and
portal hypertension, presents with a large hematoma on his thigh. On preoperative screening, his
prothrombin time is noted to be 17.4 seconds (N: 11-13.5 seconds). Transfusion of which of the
following is the most appropriate next step in management of this patient prior to his procedure?*
a) Cryoprecipitate
b) Fresh frozen plasma
c) Packed red blood cells
d) Platelets
e) Whole blood
The answer is: Fresh frozen plasma
80) Breastfeeding mother should be instructed to:
a) Follow a time table to feed her baby
b) Give newborn sterile water in between feeds
c) Feed her baby on demand
d) Dive newborn fruit juice as a source of vitamin C
e) Add cereal at 3 months of age
The answer is: Feed her baby on demand
81) A patient has failed to have any further dilation after achieving 6 cm. As possible reasons for this
secondary arrest of dilation are explored, you place an intrauterine catheter to calculate the intensity of
her contractions which are measured in Montevideo units. Which of the following is a Montevideo unit?
a) number of contractions in 10 minutes
b) number of contractions per minute times their intensity
c) intensity of any 10 contractions times the time it took for them to occur
d) number of contractions over 50 mm Hg in 10 minutes
e) number of contractions in 10 minutes times their average intensity
The answer is: number of contractions in 10 minutes times their average intensity
82) A 27-year-old male is hammering nails at a construction site without eye protection and feels something
strike his right eye. After washing out the eye, he still complains of pain and presents to the ED. Which
of the following is the safest and most accurate modality for locating the potential foreign body?
a) X-ray
b) Ultrasonography
c) MRI
d) CT scan

73
EMLE Trial Exams December 2020 (1000 Qs) Version (1.0) 22/12/2020

e) Nuclear medicine scan


The answer is: CT scan
83) One of the congenital cyanotic heart diseases is:#
a) ASD
b) Fallot’s Tetralogy
c) PDA
d) VSD
e) AVSD
The answer is: Fallot’s Tetralogy
84) A 45-year- old obese woman presents with right upper quadrant abdominal pain for several weeks .Her
pain is worse after she eats .Which of the following clinical features support the diagnosis of biliary colic
?*
a) Worsened pain on eating protein
b) Colicky spasms lasting 10 minutes
c) Radiation of pain to right shoulder
d) Hyperpyrexia
e) Antecedent use of nonsteroidal anti-inflammatory drugs
The answer is: Radiation of pain to right shoulder
85) A 34-year –old female takes phenelzine for depression presents with agitation , severe hypertension ,
mydriasis and hyperthermia .Which of the following foods did she most likely eat before presentation ?
a) Oranges
b) Apples
c) Peanuts
d) Cheese
e) Ice cream
The answer is: Cheese
86) Usual fluid requirement in burnt patient per Kg percent burn during first 24 hours is:
a) 3-4 ml.
b) 5-6 ml.
c) 7-8 ml.
d) < 10 ml.
e) < 20 ml
The answer is: 3-4 ml.
87) A 33-year-old male presents with palpitations. ECG is shown. Blood pressure is 120/80, heart rate is 130,
O2 saturation is 93%. Which is the best initial step?*^
a) Adenosine
b) Metoprolol
c) Verapamil
d) Digoxin
e) Cardioversion
The answer is: Adenosine
88) The most serious complication of preterm premature rupture of membrane (PPROM) at 28 weeks
a) Fetal compression anomaly.
b) Pulmonary hypoplasia.
c) Intrauterine infection.
d) Limb contraction.
e) Abruptio placenta.

74
EMLE Trial Exams December 2020 (1000 Qs) Version (1.0) 22/12/2020

The answer is:: Intrauterine infection.


89) A 65-year-old man presents with constant right upper quadrant pain of 3 days duration associated with
vomiting. He is febrile with temperature 39.50c WCC 18×109
a) biliary colic
b) empyema of the gall bladder.
c) acute pancreatitis
d) gallstone ileus
e) ascending cholangitis
The answer is: empyema of the gall bladder.
90) In cases with premature rupture of membranes, all the following are acceptable in the conservative
management except:
a) Frequent vaginal examination to assess cervical dilatation
b) serial complete blood count to diagnose rising of WBC
c) Close monitoring of maternal vital signs
d) Ultrasound to assess fetal weight and amount of liquor
e) Monitoring of the fetus by doing cardiotocogram
The answer is: Frequent vaginal examination to assess cervical dilatation
91) The following are obstetrics causes for premature labor EXCEPT :
a) Multiple pregnancy
b) Multiparity
c) Premature preterm rupture of the membrane
d) Cervical incompetence
e) Uterine congenital anomalies
The answer is: Multiparity
92) A 21-year-old female patient arrives with her older sister after having hit her head on a night out. The
patient's sister is being openly flirtatious with you and asks for your phone number. You are aware you
feel attracted to her as well. What is the most appropriate action in this scenario?**
a) Excuse yourself for a moment ,while you ask a registrar colleague for advice
b) Politely decline to give her your number, explaining it would be inappropriate to do
c) Give her your number as she is not your patient
d) Explain you cannot give her your number whilst you are working, but arrange to meet another time
outside of the hospital
e) Refuse to give her your phone number and ask hospital security to remove her from the hospital
premises
The answer is:: Politely decline to give her your number, explaining it would be inappropriate to do
93) A 4-year-old girl had a low-grade fever, intermittent crampy, abdominal pain and swollen knees for 3
days, there is purpuric rash and petechial rashes on the lower extremity. The most likely diagnosis is#
a) Meningococcemia
b) Idiopathic thrombocytopenic purpura
c) Henoch-Schӧnlein purpura
d) Rheumatic fever
e) German measles
The answer is: Henoch-Schӧnlein purpura
94) A 40-year-old male presents with palpitations. ECG is shown. Blood pressure is 120/80, heart rate is 130,
O2 saturation is 93%. Which is the best initial step?*^
a) Adenosine
b) Labetalol

75
EMLE Trial Exams December 2020 (1000 Qs) Version (1.0) 22/12/2020

c) Amiodarone
d) Defibrillation
e) Cardioversion
The answer is: Labetalol
95) A patient is found to be Rh negative with a negative antibody screen. Anti-D immune globulin should be
given for which of the following situations?
a) after an abortion, spontaneous or therapeutic, prior to 6 weeks’ gestation in an Rh-negative female
b) a Du positive mother who has an Rh-positive baby
c) an Rh-negative female infant with an Rh-positive mother
d) postpartum to Rh-positive females with Rh-negative husbands
e) after a motor vehicle accident to an Rh-negative mother
The answer is: after a motor vehicle accident to an Rh-negative mother
96) A 65-year-old female presents by out-of-hospital cardiac arrest with ventricular fibrillation. Which of the
following is the most effective management for this condition?
a) Defibrillation
b) Pericardiocentesis
c) Epinephrine IV
d) Amiodarone IV E. Invasive mechanical ventilation
e) non of the above
The answer is: Defibrillation
97) A butcher comes into emergency department after accidently stabbing his groin with knife . He tried to
use a towel to stop the bleeding but has bled so much that the towel is now soaked with blood . His
blood pressure is 75/40mmHg , and pulse is a 120/min . What is the single most appropriate initial
management ?
a) Blood sample for crossmatch and blood transfusion
b) IV fluids
c) Fresh frozen plasma
d) Refer to surgeon
e) Emergency Doppler Ultrasound
The answer is: IV fluids
98) A 39-year-old male is brought to A&E after suffering multiple stab wounds to the chest. On examination
the patient is in respiratory distress with poor expansion on the left side of the chest. There is deviation
of the trachea to the right. Neck veins appear distended. SaO2 is 90 per cent on air, pulse 123/min, BP
85/40 mmHg.#
a) haemothorax
b) diaphragmatic rupture
c) myocardial contusion
d) pleural effusion
e) tension pneumothorax
The answer is: tension pneumothorax
99) A 32-year-old male presents to the emergency department after he fall to the ground and started
shaking in the form of alternating rigidity and contractions of his limbs. He is unconscious. Which is the
most likely diagnosis?
a) Generalized tonic seizure
b) Generalized atonic seizure
c) Simple partial seizure
d) Complex partial seizure

76
EMLE Trial Exams December 2020 (1000 Qs) Version (1.0) 22/12/2020

e) Generalized tonic-clonic seizure


The answer is: Generalized tonic-clonic seizure
100) Total amount of plasma proteins in blood ranges from:
a) 2-4 gm%
b) 9-11 gm%
c) 6-8 gm%
d) 5-12.5 gm%
e) 1-3 gm%
The answer is: 6-8 gm%

77
EMLE Trial Exams December 2020 (1000 Qs) Version (1.0) 22/12/2020

Exam 5
1) When should you dispose of a plastic disposable apron?*
a) At the end of each shift
b) Every hour
c) Between caring for each patient
d) After it’s been washed once
e) none of the above
The answer is: Between caring for each patient
2) Which of the following drugs is most likely to cause impaired glucose tolerance?*
a) Interferon alpha
b) Bromocriptine
c) Imipramine
d) Montelukast
e) Strontium
The answer is: Interferon alpha
3) The following are the treatment options available for the management of the patient with severe head
injury. Which of the following does not have an effect on reducing intracranial pressure?
a) Corticosteroids
b) Mannitol
c) Barbiturates
d) Hyperventilation
e) Furosemide
The answer is: Corticosteroids
4) A 62-year –old male complaining of chest pain and shortness of breath .After placing the patient on
oxygen and giving aspirin and sublingual nitroglycerin ECG was provided .ECG showed ST elevation in
lead aVR with diffuse depression in the remaining leads, picture considered conclusive of left main
disease, which of the following is the most appropriate next step?
a) Prepare the catheterization laboratory for immediate cardiac catheterization
b) Start nitroglycerin drip and give loading dose of low-molecular dose heparin
c) Start nitroglycerin drip , give a loading dose of clopidogrel and a loading dose of low-molecular weight
heparin
d) Start nitroglycerin drip and thrombolytic therapy
e) Place nitroglycerin patch on the chest of the patient and give a loading dose of clopidogrel , oral
metoprolol , and a loading dose of low – molecular – weight heparin.
The answer is: Prepare the catheterization laboratory for immediate cardiac catheterization
5) The effect of pregnancy on insulin and blood glucose:
a) In the first half of pregnancy, there is increased insulin response to glucose load
b) In the second half of pregnancy, there is increased insulin resistance
c) In the second half of pregnancy, there is increased insulin requirements.
d) All of the above
e) None of the above
The answer is: All of the above
6) A 4 years old boy and has recently started school. He presents with fever and this widespread rash that
is itchy. Select the organism that is most likely to be the causative agent .

78
EMLE Trial Exams December 2020 (1000 Qs) Version (1.0) 22/12/2020

a) Chickenpox
b) Herpes simplex
c) Measles
d) Erythema infectiosum
e) Roseola infantum
The answer is: Chickenpox

7) A 55-year-old woman, who recently had been dieting with


a weight loss of 20 lb, presents with a small-bowel
obstruction and pain, which radiates down the inside of her thigh to the knee. She has no past history of
abdominal surgery. Which of the following is the likely diagnosis?**
a) Strangulated obturator hernia
b) Obstructing neoplasm of the ileum
c) Gallstone ileus
d) Strangulated femoral hernia
e) Fracture of the pubic bone
The answer is: Strangulated obturator hernia
8) -65year-old presents with fatigue and dizziness for the past 6 months. On examination, blood pressure
was 90/60. Her skin was hyperpigmented and her electrolytes was as follows. Na 125 (135 – 145), K 6
(3.5 – 5.3). Her blood pressure failed to improve after fluids. Which is the most likely next step in
management?*
a) IV normal Saline
b) IV corticosteroids
c) IV thyroxine
d) IV insulin
e) IV glucagon
The answer is: IV corticosteroids
9) A normally fit and healthy 68-year-old man presented to the Emergency Department after a sudden
collapse in the street with sudden onset back pain. On examination he was hypotensive and a palpable
pulsatile expansile epigastric mass was found, and a diagnosis of ruptured abdominal aortic aneurysm
was made. What is the treatment for ruptured AAA?*
a) Do nothing
b) High flow oxygen
c) Open surgical repair
d) IV opioid analgesia
e) IV fluids
The answer is: Open surgical repair
10) A 65-year-old man has an enterocutaneous fistula originating in the jejunum secondary to inflammatory
bowel disease. Which of the following would be the most appropriate fluid for replacement of his
enteric losses?
a) D5W. Dextrose 5%
b) %3normal saline
c) Ringer lactate solution
d) %0.9sodium chloride
e) 6% sodium bicarbonate solution
The answer is: Ringer lactate solution

79
EMLE Trial Exams December 2020 (1000 Qs) Version (1.0) 22/12/2020

11) You found an accident in the street, what is your first response***^
a) Ask someone to call the umbulance before uo start CPR
b) Try to move the people inside
c) Pass by as you have no license yet
d) Start CPR with no other considerations
e) Call a friend
The answer is: Ask someone to call the umbulance before uo start CPR
12) Signs and symptoms of moderate VSD include all except:#
a) Tachypnea and dyspnea
b) Feeding difficulties
c) Slow growth
d) Higher risk for pulmonary infection
e) Central cyanosis
The answer is: Central cyanosis
13) Which is the best initial step for a patient presenting with chest pain?*
a) Chest Xray
b) Arterial blood gases
c) Cardiac Enzymes
d) ECG
e) Complete blood picture
The answer is: ECG
14) In VSD, all are true except:#
a) Pulmonary hypertension may occur
b) The murmur is soft ejection in character
c) There is palpable systolic thrill
d) With pulmonary hypertension there is palpable 2nd sound
e) Murmur is harsh pansystolic
The answer is: The murmur is soft ejection in character
15) The following is true about atopic dermatitis:
a) The infants with atopic eczema may develop asthma later
b) Never affects the flexures
c) Non-pruritic during infancy
d) Topical agents have no role in the treatment
e) No relation to cow milk allergy
The answer is: The infants with atopic eczema may develop asthma later
16) You are called to see an 85-year-old female patient as the nursing staff is concerned that the patient has
not passed stool for 4 days. The patient has been admitted after family members became increasingly
concerned regarding her general deterioration in health and level of function. She is orientated but frail
and complains of increasing abdominal discomfort. On examination bowel sounds are increased. The
abdomen is distended with generalized tenderness, but no rebound or guarding. There is a firm palpable
mass in the left iliac fossa. Digital rectal examination shows an empty rectum. What diagnosis must be
excluded?#
a) Simple constipation
b) Paralytic ileus
c) Sigmoid volvulus
d) Peritonitis secondary to diverticular disease
e) Neoplasia

80
EMLE Trial Exams December 2020 (1000 Qs) Version (1.0) 22/12/2020

The answer is: Neoplasia


17) A primigravida, 37-week gestation presented with central placenta previa with heavy bleeding per
vaginum. The fetal heart rate was normal at the time of examination. The best management option for
her is:
a) Expectant management.
b) Induction and vaginal delivery.
c) Induction and forceps delivery.
d) Cesarean section.
e) none of the above
The answer is: Cesarean section.
18) Microcephaly is a feature of each of the following except :
a) Congenital rubella
b) Congenital cytomegalovirus infection
c) Aqueductal stenosis
d) Post anoxic CP
e) Phenylketonuria
The answer is: Aqueductal stenosis
19) You were reviewing an infant 1 year old with dilated unilateral pupil what would you expect?
a) Opiates
b) Barbiturates
c) Severe hypoxia
d) Third nerve lesion
e) Hypothermia
The answer is: Third nerve lesion
20) In a patient with peripheral arterial disease, which one of the following features suggests the worst
prognosis?
a) Claudication distance of 50–100 m
b) Night pain
c) Diminished pedal pulses
d) Ankle:brachial pressure index (ABPI) ratio of 0.8
e) Bruit over femoral artery
The answer is: Night pain
21) A patient at 34 weeks’ gestation develops marked pruritus especially on her palms and soles, and mildly
elevated liver function tests and elevated bile acids.Under which of the following conditions are the
pruritus and jaundice from the previous patient likely to recur?
a) menopause
b) after discontinuation of breast-feeding
c) poor diet
d) with another pregnancy
e) with the use of antihypertensive medication
The answer is: menopause
22) In amenorrhea the following are true EXCEPT:
a) In Turner syndrome there is failure of menstruation
b) Kallman syndrome is associated with amenorrhea and anosmia
c) Sheehan syndrome is associated with amenorrhea
d) Asherman syndrome is associated with retinitis pigmentosa
e) In Fitz Hugh Curtis there is perihepatitis

81
EMLE Trial Exams December 2020 (1000 Qs) Version (1.0) 22/12/2020

The answer is: Asherman syndrome is associated with retinitis pigmentosa


23) You found an accident in the street, what is your first response***^
a) Ask someone to call the umbulance before uo start CPR
b) Try to move the people inside
c) Pass by as you have no license yet
d) Start CPR with no other considerations
e) Call a friend
The answer is: Ask someone to call the umbulance before uo start CPR
24) You are meeting parents of a 1.5-year-old boy who has suboptimal growth. He has been delivered
prematurely at the gestational age of 32 weeks. Examination reveals a healthy child with growth
parameters below normal for age. You reassure the parents that their child will catch his normal growth
with time. Of the following, the growth parameter that should be corrected at this age is
a) weight
b) height
c) body mass index
d) head circumference
e) arm circumference
The answer is: head circumference
25) Which of the following does not affect the choice of prophylactic antibiotic?*
a) The expected spectrum of organisms likely to be encountered
b) Cost
c) Personal preference
d) Hospital policies
e) Local resistance strains.
The answer is: Personal preference
26) A 2-year-old is admitted following a road traffic collision. Multiple attempts at peripheral cannulation
have failed. She is tachycardic and hypotensive. What is the most appropriate next step in the patient’s
management?*
a) Endotracheal intubation
b) Laparotomy
c) Intraosseous line
d) Chest drain insertion
e) Mannitol
The answer is: Intraosseous line
27) The following are typical of Fallot’s tetralogy except:
a) Pulmonary and aortic stenosis are combined with VSD
b) Finger clubbing and central cyanosis are present
c) The second heart sound is loud and single
d) The chest x-ray and ECG are typically abnormal
e) Right ventricular hypertrophy
The answer is: Pulmonary and aortic stenosis are combined with VSD
28) A 17-year-old boy presents with bloody diarrhea over the past 2 weeks, accompanied by frequent urges
& abdominal cramping. One hour ago, he saw mucus & fresh blood on his stool. Similar symptoms have
occurred over the past 2 years, except for the blood in his stool. His temperature is 37 °C and normal
pulse and blood pressure. His abdomen is soft, without guarding. Which of the following is the most
likely diagnosis?
a) Celiac disease

82
EMLE Trial Exams December 2020 (1000 Qs) Version (1.0) 22/12/2020

b) Chronic appendicitis
c) Diverticulitis
d) Irritable bowel syndrome
e) Ulcerative colitis
The answer is: Ulcerative colitis
29) A patient is seen in the early third trimester of pregnancy with acute onset of chills and fever, nausea,
and backache. Her temperature is 102°F. The urinary sediment reveals many bacteria and WBCs. Which
of the following is the most likely diagnosis?
a) acute appendicitis
b) ruptured uterus
c) pyelonephritis
d) abruptio placentae
e) labor
The answer is: pyelonephritis
30) Arterial blood gas sampling can be done at a point:*
a) Midway between ASIS and symphysis pubis
b) Midway between ASIS and pubic tubercle
c) Midway between ASIS and pubic crest
d) Lateral to the internal inguinal ring
e) Non of the above
The answer is: Midway between ASIS and pubic tubercle
31) You found an accident in the street, what is your first response***^
a) Call ambulance
b) Try to move the people inside
c) Pass by as you have no license yet
d) Start CPR
e) Call a friend
The answer is: Call ambulance
32) What is the origin of complex partial seizures?*
a) Frontal lobe
b) Parietal lobe
c) Temporal lobe
d) Occipital lobe
e) Putamen
The answer is: Temporal lobe
33) A 30-year-old woman presents with generalized abdominal pain, vomiting, distention, fever,
leukocytosis and an abdominal ultrasound that showed pelvic collection. The most likely diagnosis :
a) Perforated appendicitis.
b) Salpengitis.
c) Complicated Meckel’s diverticulum.
d) Amoebic colitis.
e) Ulcerative colitis.
The answer is: Perforated appendicitis.
34) A 40-year-old woman presents with a history of a slowly growing right-sided lump at the base of the
neck. On examination there is a thyroid nodule with associated lymphadenopathy. She has a recent
history of adrenalectomy.
a) Graves’ disease

83
EMLE Trial Exams December 2020 (1000 Qs) Version (1.0) 22/12/2020

b) follicular adenoma
c) medullary carcinoma
d) De Quervain’s thyroiditis
e) papillary carcinoma
The answer is: medullary carcinoma
35) What is the best management of pain in a patient with CKD?*
a) Give him morphine
b) Give him ibuprofen
c) Give him fentanyl
d) Give him tramadol
e) Give him oxycodone
The answer is: Give him fentanyl
36) A 3-week-old neonate presents with regurgitation and persistent, projectile vomiting. An olive-like
epigastric mass is felt during physical examination. This infant’s mother did not have polyhydramnios
during this pregnancy. A chest x-ray does not reveal the presence of bowel gas in the chest cavity. Which
of the following is the most appropriate treatment for this infant’s condition?
a) Oral medication with omeprazole and clarithromycin
b) Oral medication with vancomycin or metronidazole
c) Surgery to cut a hypertrophied stenotic band at the pylorus
d) Surgery to remove a mass of the adrenal gland
e) Surgery to resect an aganglionic section of the intestines
The answer is: Surgery to cut a hypertrophied stenotic band at the pylorus
37) Which of the following quality measures is used for counting the number of diabetes educators?
a) Structural
b) Process
c) Outcome
d) balancing
e) Non-specific
The answer is: Structural
38) From highest to lowest mortality, what are the three most common causes of cancer death in men?*
a) Colorectal, lung, testicular
b) Lung, colorectal, testicular
c) Lung, prostate, colorectal
d) Prostate, colorectal, lung
e) Prostate, lung, colorectal
The answer is: Lung, prostate, colorectal
39) A 16-year-old daughter refuses medication but her mother consents. What is the most appropriate
action to do?**
a) Write the prescription
b) Do not write the prescription
c) Postpone the prescription
d) Convince the daughter
e) Report the event to the police
The answer is: Write the prescription
40) A 53-year – old male with a history of alcohol abuse presents to the emergency department for
evaluation of diaphoresis , repeated non bloody vomiting and abdominal pain . His last drink was 24
hours ago . Temperature 37.3C , pulse 113bpm , RR 18breath per minute , BP 120/70mmHg , SaO2 97% ,

84
EMLE Trial Exams December 2020 (1000 Qs) Version (1.0) 22/12/2020

HCO3 13 mEq/L , glucose 86 mg/dL , Urine ketones very high ( four plus ), Na 129 mEq/L , K 3.8mEq/L ,
normal serum creatinine and unremarkable abdominal examination . Which is the best next step ? *
a) IV thiamine , bolus of 5% dextrose in normal saline
b) IV insulin infusion , IV thiamine , IV infusion of 5 % dextrose in normal saline
c) IV insulin infusion , IV 5% dextrose infusion with 40 mEq Potassium chloride
d) IV insulin infusion , IV 5% dextrose infusion with 20 mEq sodium bicarbonate
e) Bolus of normal saline over one hour plus antiemetic
The answer is: IV thiamine, bolus of 5% dextrose in normal saline
41) A 27-year-old male is hammering nails at a construction site without eye protection and feels something
strike his right eye. He is presented to the ED complaining of painful eye. Which of the following is the
safest and most accurate for him to locate the potential foreign body?*
a) X-ray
b) Ultrasonography
c) MRI
d) CT scan
e) Nuclear medicine scan
The answer is: CT scan
42) Management of autistic children includes all except:
a) Psychometric evaluation
b) Intensive and early behavior intervention
c) Dietary control
d) Speech therapy
e) Home schooling only
The answer is: Home schooling only
43) the correct statement regarding cancer cervix:
a) Adenocarcinoma occurs in about 30% of cases
b) It is the most common cancer among women
c) Is more common in smokers
d) It does not spread by direct infiltration
e) Is more common in nulliparous women
The answer is: Is more common in smokers
44) A 69-year-old woman with right leg pain is diagnosed with a deep vein thrombosis on Doppler
investigation. On examination there are firm rubbery matted masses in both groins .#
a) varicocoele
b) seminoma
c) scrotal carcinoma
d) saphena varix
e) inguinal lymphadenopathy
The answer is: Inguinal lymphadenopathy
45) Lower motor neuron lesion is caused by all the following except:
a) Congenital myopathy
b) Hereditary sensory moto neuropathy
c) Neuropathy
d) Cerebral palsy
e) Spinal muscle atrophy
The answer is: Cerebral palsy
46) All of the following are associated with acute rheumatic fever except:#

85
EMLE Trial Exams December 2020 (1000 Qs) Version (1.0) 22/12/2020

a) Joint pain
b) Erythema nodosum
c) Fever
d) High ESR
e) Carditis
The answer is: Erythema nodosum
47) Fetal manifestations of erythroblastosis fetalis may include all of the following EXCEPT :
a) Kimictrus
b) Hepatomegaly
c) Placental edema
d) Splenic enlargement
e) Polycythemia
The answer is: Placental edema
48) Which of the following is known to be the commonest presentation in twins?
a) Breech, cephalic
b) Cephalic, breech
c) Cephalic, cephalic
d) Breech, breech
e) Cephalic, transverse
The answer is: Cephalic, cephalic
49) A 24-year –old man is presented to the ER after becoming unwell , drosy and disoriented . He has no
history of medical problems . Over the last few days he has become progressively weak , lethargic ,
drosy and disoriented . The patient appears confused , temperature 36.5 C , with cool extremeties , poor
capillary refill , pulse 120 bpm , BP 75/55mmHg , K 5.6mmol/L , Na 125mmol/L , Cl 101mmol/L , Hb
13g/dL , glucose 65mg/dL . After BLS and fluid resuscitation , what is the most emergent next step ? *
a) IV hydrocortisone
b) IV thyroid hormone
c) IV Noradrenaline
d) IV antibiotics
e) IV glucose
The answer is: IV hydrocortisone
50) What of the following causes microcytic anemia?*
a) Thalassemia
b) Sickle cell anemia
c) Vitamin B12 deficiency anemia
d) G6PD deficiency
e) Autoimmune hemolytic anemia
The answer is: Thalassemia
51) You entered a room and found a child on the ground not responding First thing to do is?*
a) Try to move him
b) Call for help
c) Freak out
d) Find a certified doctor in emergency
e) Non of the above
The answer is: Call for help
52) A 29-year-old G2 P1 woman at 39 weeks gestation had a myomectomy for infertility previously. While
pushing during the second stage of labor, she is noted to have fetal bradycardia associated with some

86
EMLE Trial Exams December 2020 (1000 Qs) Version (1.0) 22/12/2020

vaginal bleeding. The fetal head, which was previously at +2 stations, is now noted to be -3 station.
Which of the following is the MOST LIKELY diagnosis?
a) Submucosal myomata.
b) Umbilical cord prolapsed.
c) Uterine rupture.
d) Placental abruption.
e) Fetal congenital heart block.
The answer is: Uterine rupture.
53) An 82-year-old woman presents with a 24-hour history of a painful swelling in her left groin. The patient
is unwell and dehydrated with a tachycardia of 110beats/min. On examination, the swelling is 3 x 3cm in
size, firm and tender. It is not reducible. What is the most likely diagnosis?#
a) Inguinal lymphadenopathy
b) Femoral hernia
c) Inguinal hernia
d) Saphena varix
e) Femoral artery aneurysm
The answer is: Femoral hernia
54) A 2-year-old is admitted following a road traffic collision. Multiple attempts at peripheral cannulation
have failed. She is tachycardic and hypotensive. What is the most appropriate next step in the patient’s
management?*
a) Endotracheal intubation
b) Laparotomy
c) Intraosseous line
d) Chest drain insertion
e) Mannitol
The answer is: Intraosseous line
55) Anti-D prophylaxis:
a) Should be given to all sensitized Rhesus negative women after delivery
b) Should be given to all Rhesus negative women after amniocentesis.
c) Should be given to all Rhesus positive women who give birth to Rhesus negative babies
d) Should be given to all women who's babies are Rhesus negative
e) Is contra-indicated during pregnancy if the women is Rhesus negative
The answer is: Should be given to all Rhesus negative women after amniocentesis.
56) A 45-year-old female complains of fresh bleeding per rectum and occasionally bloody diarrhea since few
weeks. She also has anorexia, tiredness and significant weight loss. Which of the following is the most
likely diagnosis?
a) Anal fissure
b) Colorectal cancer
c) Infectious diarrhea
d) Irritable bowel syndrome
e) Diverticulitis
The answer is: Colorectal cancer
57) Rheumatic carditis is treated by
a) Corticosteroids
b) Phenobarbitone
c) Analgesics
d) Chloramphenicol

87
EMLE Trial Exams December 2020 (1000 Qs) Version (1.0) 22/12/2020

e) Diuretics
The answer is: Corticosteroids
58) A 60-year-old man is admitted to ER with hematemesis. His pulse is 110/min and blood pressure is
100/60 mm. He has multiple spider angiomata on his back and chest, with bilateral gynecomastia.
Abdominal examination is significant for hepato-splenomegaly, his abdomen is distended and ascites.
His hematocrit is 23%. After placement of a nasogastric tube, 400 mL of bright red blood is evacuated.
After initial fluid resuscitation, which of the following is the most appropriate next step in management?
a) Barium swallow
b) Esophageal balloon tamponade
c) UGI endoscopy
d) Ultrasonography of abdomen
e) Selective angiography
The answer is: UGI endoscopy
59) Gas in the biliary system in a plain X-ray is diagnostic of
a) Choledochoduodenal fistula
b) intestinal obstruction
c) Emphysematous Cholecystitis
d) Viral hepatitis
e) A and C
The answer is: A and C
60) A 40-year-old male with no past medical history presents to the outpatient clinic for routine visit. His
Hemoglobin A1C is 8%. The physician started him on metformin. Which is the most important side effect
of this drug?
a) Lactic acidosis
b) Pancreatitis
c) UTI
d) Vaginal Candida
e) Hypoglycemia
The answer is: Lactic acidosis
61) A 36-year-old woman presents with a history of increasing weight gain, tiredness and neck swelling. On
examination the patient has a firm rubbery goitre. Fineneedle aspiration shows a diffuse lymphocytic
and plasma cell infiltrate. The report also mentions the presence of lymphoid follicles and parenchymal
atrophy.
a) follicular carcinoma
b) Hashimoto’s disease
c) anaplastic carcinoma
d) endemic goitre
e) lymphoma
The answer is: Hashimoto’s disease
62) What is the commonest cause of diarrhea followed by HUS?#
Shigella
Salmonella
Campylobacter
E-coli
None of the above
The answer is: E-coli

88
EMLE Trial Exams December 2020 (1000 Qs) Version (1.0) 22/12/2020

63) A 32-year-old woman (gravida 3, para 1, abortus 1) at term is admitted in labor with an initial cervical
examination of 6-cm dilatation, complete effacement, and the vertex at –1 station. Estimated fetal
weight is 8 lb, and her first pregnancy resulted in an uncomplicated vaginal delivery of an 8-lb infant.
After 2 hours, there is no cervical change. An intrauterine pressure catheter is placed. This shows three
contractions in a 10-minute period, each with a strength of 40 mm Hg.What is the best course of action
at this time?
a) wait 2 more hours and repeat the cervical examination
b) start oxytocin augmentation
c) perform a cesarean section
d) discharge the patient, instructing her to return when contractions become stronger
e) therapeutic rest with analgesia and short-acting anti-anxiety medication
The answer is: start oxytocin augmentation
64) In developed countries (western societies) what is the most common cause of direct maternal mortality?
a) hemorrhage
b) congenital cardiac disease
c) infection
d) hypertension
e) amniotic fluid embolism
The answer is: hypertension
65) All are true about rheumatic fever except:
a) Recurrence is common
b) Is commonest between ages of 5-15 years
c) Follows an upper respiratory tract infection by group A β-hemolytic streptococci
d) Is treated by corticosteroids even if arthritis is the only manifestations
e) Rheumatic arthritis does not result in permeant damage of the affected joint
The answer is: Follows an upper respiratory tract infection by group A β-hemolytic streptococci
66) All of the following are findings in iron deficiency anemia except:*
a) Pallor and easy fatigability
b) Large hepatosplenomegaly
c) Hypochromic microcytic anemia
d) Decreased serum ferritin
e) Increased total iron binding capacity
The answer is: Large hepatosplenomegaly
67) A 62-year-old man admitted 5 hours after left hemiplegia. ECG confirms AF. A CT head is normal. What is
the most appropriate initial management?
a) Aspirin
b) Heparin
c) Clopidogrel
d) Alteplase
e) Streptokinase
The answer is: Aspirin
68) You are at the word round and suddenly a kid collapsed in coma with unilateral dilated pupil what
would you suspect?*
a) Opiates
b) Barbiturates
c) Severe hypoxia
d) Tentorial herniation

89
EMLE Trial Exams December 2020 (1000 Qs) Version (1.0) 22/12/2020

e) Hypothermia
The answer is: Tentorial herniation
69) A 55-year-old male with history of Diabetes and hypertension presents to the emergency department
with chest pain that started 1 hour ago. ECG showed ST elevation. Which is the most definitive
treatment?*
a) Aspirin
b) Percutaneous coronary angiography
c) Warfarin
d) Metoprolol
e) Oxygen
The answer is: Percutaneous coronary angiography
70) A 30-year-old Afro-Carribean woman presents to accident and emergency with a 1-week history of
progressive shortness of breath and fever. On further questioning, she mentions that her hands have
been painful and stiff over the past few months and she has been having recurrent mouth ulcers. Chest
x-ray confirms bilateral pleural effusions and blood tests reveal a raised ESR and a normal CRP. What is
the most likely diagnosis?*
a) Systemic lupus erythematosus
b) Systemic sclerosis
c) Sjögren’s syndrome
d) Discoid lupus
e) Beçhet’s disease
The answer is: Systemic lupus erythematosus
71) The following are complications of multiple pregnancy EXCEPT:
a) Increase incidence of pre-eclamptic toxemia
b) Polyhydramnios
c) Increase incidence of preterm labor
d) Increase incidence of gestational diabetes
e) Increase incidence of placenta previa
The answer is: Increase incidence of gestational diabetes
72) Which of the following statements regarding clostridial wound infections are false?#
a) Clostridia are anaerobic, terminal, spore bearing, Gram-positive bacteria.
b) Thin, brown and sweet-smelling exudate is seen in gas gangrene.
c) Necrotic and foreign material in wounds increase risk.
d) The spores are widely spread in soil and manure.
e) The signs and symptoms are due to the endotoxins.
The answer is: The signs and symptoms are due to the endotoxins.
73) The most common cause of death in recipients of a solid-organ transplant is:
a) Recurrent organ failure
b) Infection
c) Drug toxicity
d) Organ rejection
e) Effects of the primary disease process
The answer is: Infection
74) A 47 –year –old male with hypertension and hyperlipidemia presents to the emergency department with
a history of 10 minutes of right sided weakness that has just resolved . Which of the following should be
included in his treatment in the department before admission for further evaluation?
a) Clopidogrel

90
EMLE Trial Exams December 2020 (1000 Qs) Version (1.0) 22/12/2020

b) Ticlopidine
c) Heparin
d) Aspirin
e) Warfarin
The answer is: Aspirin
75) in congenital diaphragmatic hernia, all are true except:
a) Diagnosis is by auscultation bowel sounds in the thorax
b) Pulmonary hypoplasia is common
c) Surgery is indicated
d) Initial management with O2 by ambu bag and mask
e) X ray will show intestinal lobes in chest
The answer is: Initial management with O2 by ambu bag and mask
76) During pregnancy, blood tests for diabetes are more apt to be abnormal than in the nonpregnant state.
Also nondiabetic women may develop gestational diabetes during the last half of the pregnancy. This is
due in part to which of the following?
a) decreased insulin production
b) increased food absorption from the GI tract
c) increased placental lactogen
d) decreased hepatic secretion of insulin-binding globulin
e) hemoconcentration
The answer is: increased placental lactogen
77) A 62-year-old man presents with rectal bleeding and year‘s history of constipation and left iliac fossa
pain. There is no weight loss. Which of the following is the most likely cause for the clinical findings?
a) Anal fissure
b) Celiac disease
c) Chronic appendicitis
d) Diverticulosis
e) Irritable bowel syndrome
The answer is: Irritable bowel syndrome
78) Which of the following signs is elicited in pleural effusion?#
a) Increased movement on the affected side
b) Diminished intensity of breath sounds on the affected side
c) Shift of the trachea towards the affected side
d) Increased vocal resonance on the affected side
e) Hyper resonance on percussion of the affected side
The answer is: Diminished intensity of breath sounds on the affected side
79) A 13-year-old female presenting with RUQ abdominal pain is suspected of having a choledochal cyst.
Which of the following studies would be least helpful in confirming the diagnosis in this case?
a) Computed tomography (CT) scan
b) Percutaneous transhepatic cholangiography
c) Endoscopic retrograde cholangiopancreatography
d) Magnetic resonance cholangiopancreatography (MRCP)
e) Upper GI series
The answer is: Upper GI series
80) Which of the following is not associated with increased likelihood of infection after major elective
surgery?*
a) Age over 70 years.

91
EMLE Trial Exams December 2020 (1000 Qs) Version (1.0) 22/12/2020

b) Chronic malnutrition.
c) Controlled diabetes mellitus.
d) Long-term steroid use.
e) Infection at a remote body site.
The answer is: Controlled diabetes mellitus.
81) A 25-year-old diabetic man is brought unconscious, he is dehydrated and shocked. Investigations reveal
Blood glucose level 270mg/dl, pH 7.26 (7.35 to 7.45), HCO3 15 mEq/l (22 – 28 mEq/ l), urinalysis shows
ketone bodies positive. Which of the following is most useful in the management of this case?#
a) Basal insulin only
b) Metformin with short acing insulin
c) Mixed short acting and NPH insulin
d) NPH INSULIN
e) Short acting insulin by IV infusion
The answer is: Short acting insulin by IV infusion
82) You see a 24-year-old student with suspected meningitis. The registrar asks you to perform a lumbar
puncture before she goes to the ward. This is not a procedure you have performed before, and you can
recall only limited information about the procedure from medical school teaching. What should you
do?**
a) Explain to the registrar that this is outside of your competence and so you are not comfortable in
performing the procedure
b) Explain to the registrar this is not a skill that should be expected of you
c) Ask an older colleague to perform the procedure whilst you observe and learn for future attempts
d) As the procedure has many potential risks, await the patient's blood test results before deciding if she
really requires the procedure
e) Do not perform the procedure, but document clearly in the notes that it has been requested and will
need to be performed on the ward
The answer is: Explain to the registrar that this is outside of your competence and so you are not comfortable
in performing the procedure
83) During elective major vascular surgery the best way to reduce the risk of acute renal failure is to
maintain a normal#
a) Central venous pressure
b) Mean arterial blood pressure
c) Renal blood flow
d) Systemic vascular resistance
e) Urine output
The answer is: Renal blood flow
84) In febrile convulsions, the following statements are true except:
a) Occur in 2-5% of children
b) Genetic predisposition is common
c) Higher fever ≥39°C is usually present
d) Occurs in children aged between 3 months to 5 years
e) It is treated by oral glucose
The answer is: It is treated by oral glucose
85) Which of the following tocolytics is associated with reversible oligohydramnios?
a) Indomethacin
b) Magnesium sulfate
c) Ritodrine

92
EMLE Trial Exams December 2020 (1000 Qs) Version (1.0) 22/12/2020

d) Terbutaline
e) None of the above
The answer is: Indomethacin
86) Child presented to you with acetone breath, disoriented and looks dehydrated what is your first line of
treatment (his glucose level is giving Error in the strips) *
a) Give fluids
b) Give packed RBC
c) Give plasma
d) Give glucose solution
e) Ask for CBC
The answer is: Give fluids
87) A 30-year-old motorcyclist presents with an open left femoral fracture following a road traffic collision
at 40 mph. On presentation to the ED, the left leg is in a Thomas splint. The left thigh wound is washed
out and covered with an iodine dressing. What is the most appropriate treatment option?*
a) Hanging cast
b) External fixation
c) Manipulation and cast application
d) Plating
e) Dynamic hip screw
The answer is: External fixation
88) The following has a recognized relation with spontaneous preterm labor :
a) Fetus with anencephaly
b) Oligohydromnios
c) Maternal hypothyroidism
d) Bacterial Vaginosis
e) Transverse lie of the fetus
The answer is: Bacterial Vaginosis
89) What is the best drug to give in variceal bleeding?*
a) Octreotide
b) Frusemide
c) Spironolactone
d) Indipamide
e) Propranolol
The answer is: Octreotide
90) What is the best treatment of uremic bleeding in CKD?
a) Intranasal desmopressin
b) Prothrombin complex
c) FFP
d) Coagulation factor concentrates
e) VWF concentrates
The answer is: Intranasal desmopressin
91) Acute ITP is characterized by all except:
a) Presence of antiplatelet antibody
b) Frequency of spontaneous remission
c) Splenomegaly and lymphadenopathy
d) Increased number and megakaryocytes in BM
e) Both sex affected

93
EMLE Trial Exams December 2020 (1000 Qs) Version (1.0) 22/12/2020

The answer is: Splenomegaly and lymphadenopathy


92) Which of the following does not affect the choice of prophylactic antibiotic?*
a) The expected spectrum of organisms likely to be encountered
b) Cost
c) Personal preference
d) Hospital policies
e) Local resistance strains.
The answer is: Personal preference
93) Angular stomatitis may be caused by:
a) Iron deficiency
b) Vitamin D deficiency
c) Riboflavin deficiency
d) Folic acid deficiency
e) Ascorbic acid deficiency
The answer is: Riboflavin deficiency
94) The best uterine scar a patient can have for Caesarian section is
a) Transverse upper segment
b) Longitudinal upper segment
c) Transverse lower segment
d) Longitudinal lower segment
e) A T-shaped incision
The answer is: Transverse lower segment
95) What is the SAAG ratio in ascites secondary to portal hypertension?
a) 5
b) 0.5
c) 0.3
d) 0.6
e) 0.9
The answer is: 5
96) A 24-year-old patient who has signs and symptoms of renal lithiasis is to have an intravenous pyelogram
(IVP) as part of a urologic investigation. Before proceeding with the study, which of the following should
you determine?
a) whether she is using contraception
b) whether she is in the follicular phase of a menstrual cycle
c) whether she is sexually active
d) whether she has a history of children with birth defects
e) whether she may be pregnant
The answer is: whether she may be pregnant
97) One of the complications of I.N.H treatment of TB is:
a) Renal toxicity
b) Peripheral neuritis
c) Blindness
d) Hemolysis
e) Deafness
The answer is: Peripheral neuritis

94
EMLE Trial Exams December 2020 (1000 Qs) Version (1.0) 22/12/2020

98) What is the cause of abdominal pain in a young patient complaining of a 12-hour history of severe right
iliac fossa pain, nausea and vomiting with guarding, rigidity and rebound tenderness in the right iliac
fossa on examination?*
a) Acute appendicitis
b) Acute pancreatitis
c) Ascending cholangitis
d) Acute MI
e) Ruptured abdominal aortic aneurysm
The answer is: Acute appendicitis
99) The increased risk of postpartum hemorrhage during delivery of diabetic woman can be attributed to :#
a) Polyhydramnios
b) Macrsomia
c) Prolongedlabor
d) Obstructed labor
e) All of the above
The answer is: All of the above
100) A 52-year-old male presents with malaise, fever, and epigastric pain that radiates to his back.
Medical history includes repeated bouts of pancreatitis. Physical examination finds a low-grade fever,
and a mass is palpated in the epigastric area. An abdominal CT scan finds a fluid-filled mass in the
pancreas. This mass is removed during laparotomy. It is filled with clear fluid, and histologic sections
reveal a large cystic structure that lacks an epithelial lining. Which of the following is the most likely
diagnosis?
a) Cylindroma
b) Hydrocystoma
c) Pseudocyst
d) Pseudomyxoma
e) Syringoma
The answer is: Pseudocyst

95
EMLE Trial Exams December 2020 (1000 Qs) Version (1.0) 22/12/2020

Exam 6
1) An otherwise healthy, 60-year-old male has been advised to undergo surgical treatment for a left ingunial
hernia. Which of the following are acceptable standards of surgical treatment?
a) Traditional surgical repair under general or local anesthesia
b) Repair of the hernia and ipsilateral orchiectomy, in order to better assure closure of the inguinal canal
and reduce the possibility of recurrence
c) Laparotomy to perform a retroperitoneal repair
d) Surgical exploration of the contralateral groin to search for an occult hernia sac and to remove it before
a hernai develops
e) The patient should be advised to wear a truss postoperatively, in order to reduce the incidence of
recurrence
The answer is: Traditional surgical repair under general or local anesthesia
2) One of the following statements is true about septic arthritis
a) Staph aureus is the most commonly incriminated organism
b) It is more common in males with a peak age >5 years
c) It is polyarticular in over 90% of cases
d) Blood culture and ESR usually reveals no abnormalities
e) Oral antibiotic therapy for 10 days is sufficient treatment
The answer is: Staph aureus is the most commonly incriminated organism
3) Which type of hepatitis was a top priority in a national campaign in Egypt recently?
a) Hepatitis A
b) Hepatitis B
c) Hepatitis C
d) Hepatitis D
e) Hepatitis E
The answer is: Hepatitis C
4) A 62-year-old man presents to the GP with a lump in the left groin which has been present for over 2
months. On examination, the lump is above the inguinal ligament. It is reducible and has a cough impulse,
but does not extend into the scrotum. Which of the following is the most likely diagnosis?*
a) Direct inguinal hernia
b) Femoral hernia
c) Gluteal hernia
d) Indirect inguinal hernia
e) Obturator hernia
The answer is: Direct inguinal hernia
5) Genetic primary hypophosphatemic rickets is characterized by the following except:
a) X-linked dominant inheritance
b) Profound leg deformities
c) Marked dwarfism in untreated patients
d) Hydrocephalus
e) Elevated serum level of alkaline phosphatase
The answer is: Hydrocephalus
6) What is the origin of complex partial seizures?*
a) Frontal lobe
b) Parietal lobe
c) Temporal lobe

96
EMLE Trial Exams December 2020 (1000 Qs) Version (1.0) 22/12/2020

d) Occipital lobe
e) Putamen
The answer is: Temporal lobe
7) All of the following are important in the evaluation of tall patients except:
a) Family history
b) IGF-1
c) Serum homocysteine
d) Chromosomal analysis
e) Morning cortisol
The answer is: Morning cortisol
8) A 55-year-old woman, who recently had been dieting with a weight loss of 20 lb, presents with a small-
bowel obstruction and pain, which radiates down the inside of her thigh to the knee. She has no past
history of abdominal surgery. Which of the following is the likely diagnosis?**
a) Strangulated obturator hernia
b) Obstructing neoplasm of the ileum
c) Gallstone ileus
d) Strangulated femoral hernia
e) Fracture of the pubic bone
The answer is: Strangulated obturator hernia
9) Which of the following should be avoided during hand washing?**
a) Paper towels
b) Liquid soap
c) Nail brushes
d) Washing finger webs
e) Complete drying
The answer is: Nail brushes
10) Which of the following about reactionary hemorrhage are false?
a) This is delayed hemorrhage occurring within 24 h after operation.
b) It is usually caused by dislodgement of clot, normalization of blood pressure or slippage of ligature.
c) It is not associated with infection.
d) It can be significant, requiring re-exploration.
e) It is usually venous.
The answer is: It is usually venous.
11) After a head trauma what is the urgent investigation to ask ( if needed )*
a) CT
b) MRI
c) U/S
d) Angiography
e) CBC
The answer is: CT
12) What is the cause of abdominal pain in a young patient complaining of a 12-hour history of severe right
iliac fossa pain, nausea and vomiting with guarding, rigidity and rebound tenderness in the right iliac fossa
on examination?*
a) Acute appendicitis
b) Acute pancreatitis
c) Ascending cholangitis
d) Acute MI

97
EMLE Trial Exams December 2020 (1000 Qs) Version (1.0) 22/12/2020

e) Ruptured abdominal aortic aneurysm


The answer is: Acute appendicitis
13) A 40-year woman comes in with 4 months of worsening joint pain and swelling of the small joints of the
hands, wrists, feet and ankles, severe fatigue, and 2 hours of morning stiffness which improves as the day
goes on and with movement. You obtain an x-ray of the hands and wrists that demonstrates diffuse
osteopenia and marginal erosions in the metacarpophalangeal and carpal bones of the wrist. Which is the
most specific test recommended for diagnosis of this patient?#
a) ANA (Antinuclear antibody)
b) Anti-CCP antibody (Cyclic Citrullinated Peptide)
c) Anti-Double stranded DNA (dsDNA)
d) HLA-B27
e) Rheumatoid factor (RF)
The answer is: Anti-CCP antibody (Cyclic Citrullinated Peptide)
14) The most common cause of a peptic ulcer is
a) Non-steroidal anti-inflammatory drugs
b) Cigarette smoking
c) Frequent intake of caffeinated drinks
d) Helicobacter pylori
e) High levels of emotional stress
The answer is: Helicobacter pylori
15) A 30-year-old motorcyclist presents with an open left femoral fracture following a road traffic collision at
40 mph. On presentation to the ED, the left leg is in a Thomas splint. The left thigh wound is washed out
and covered with an iodine dressing. What is the most appropriate treatment option?*
a) Hanging cast
b) External fixation
c) Manipulation and cast application
d) Plating
e) Dynamic hip screw
The answer is: External fixation
16) A 28-year-old G2P0 at 39 weeks is in early labor. She is 2 cm dilated and 90% effaced, with contractions
every 4 to 5 minutes. The fetal heart tones are reassuring. Her nurse steps out for a moment and returns
to find her having a seizure.This patient is most at risk for mortality from which of the following
complications?
a) infection
b) uremia
c) congestive heart failure
d) fever
e) cerebral hemorrhage
f) non of the above
The answer is: cerebral hemorrhage
17) A Rh negative women has increase the chances of being immunized during pregnancy when:
a) Performing External cephalic version.
b) Maternal anemia.
c) Premature labor.
d) Maternal Thyrotoxicosis.
e) Multiple pregnancy.
The answer is: Performing External cephalic version.

98
EMLE Trial Exams December 2020 (1000 Qs) Version (1.0) 22/12/2020

18) All of the following statements with regard to the copper IUCD are correct EXCEPT#
a) Modern copper IUCDs are clinically effective and safe for at least five years.
b) It reduces the number of sperm reaching the fallopian tube and their capacity to fertilize the egg
c) It is associated with a slight increased risk of PID
d) Is contraindicated in women with irregular uterine bleeding
e) Levonorgestrel releasing devices are associated with heavy menstrual bleeding (HMB
The answer is: Levonorgestrel releasing devices are associated with heavy menstrual bleeding (HMB
19) Complications of pancreatitis include:*
a) Pseudocyst
b) Retroperitoneal haematoma
c) Tetany
d) Abscess formation
e) All of the above
The answer is: All of the above
20) A 42-year-old man is hospitalized with chest pain. The patient is awake and alert. A woman comes to you
demanding information about the patient, saying that she is his wife. She shows her identification card
verifying this. You tell her about his condition .What rule is violated here?
a) The principle of confidentiality
b) The principle of patient autonomy
c) The principle of social justice
d) The principle of professional competence
e) The principle of honesty with patients
The answer is: The principle of confidentiality
21) Regarding intrauterine growth the following is true:
a) Hormones are the only involved factors
b) Placental insufficiency is a detrimental factor
c) Maternal health doesn’t influence it
d) Bad maternal nutrition is not harmful
e) Folic is a meth and of no use
The answer is: Placental insufficiency is a detrimental factor
22) Which of the following statements regarding Boerhaave’s syndrome is false?*
a) Endoscopic evaluation of the esophageal tear is an integral part of diagnosis
b) The full‐thickness esophageal tear is typically located in the left posterolateral aspect of the lower
esophagus approximately 2–3 cm above the gastroesophageal junction
c) Mackler’s triad of vomiting, lower chest pain, and subcutaneous emphysema is seen in up to two thirds
of patients with Boerhaave’s syndrome
d) The underlying pathophysiology involves uncoordinated esophageal contraction against a closed pylorus
distally and cricopharyngeus muscle proximally
e) Upper GI bleeding frequently accompanies Boerhaave’s syndrome
The answer is: Endoscopic evaluation of the esophageal tear is an integral part of diagnosis
23) A 16 -year-old boy his relatives in west africa after schooling in England for his GCSEs. During his visit ,he
develops a fever and abdominal pain,and is treated for plasmodium ovale malaria. He is commence on a
course of primaquine three days prior to attendence for hypozoite eradication. which of the follwing is
the most likely underlying diagnosis select one answer only
a) pyruvate kinase deficiency
b) triose phosphate isomerase deficiency
c) glucose-6-phoshate dehydrogenase deficiency (G6PD)

99
EMLE Trial Exams December 2020 (1000 Qs) Version (1.0) 22/12/2020

d) phosphoenolpyruvate carboxykinase(PEPCK) deficiency


e) hexokinase deficiency
The answer is: glucose-6-phoshate dehydrogenase deficiency (G6PD)
24) Which of the following statements regarding cytokines is incorrect?
a) Cytokines act directly on target cells and may potentiate the actions of one another.
b) Interleukin 1 (IL-1) is a major proinflammatory mediator with multiple effects, including regulation of
skeletal muscle proteolysis in patients with sepsis or significant injury.
c) Platelet-activating factor (PAF) is a major cytokine that results in platelet aggregation,
bronchoconstriction, and increased vascular permeability.
d) Tumor necrosis factor alpha (TNF-a), despite its short plasma half-life, appears to be a principal mediator
in the evolution of sepsis and the multiple organ dysfunction syndrome because of its multiple actions
and the secondary cascades that it stimulates.
e) All of the above
The answer is: Platelet-activating factor (PAF) is a major cytokine that results in platelet aggregation,
bronchoconstriction, and increased vascular permeability.
25) A 55-year-old man is brought to the emergency department by the police after being found wandering
and incoherent to himself. The initial impression is of an emaciated, jaundiced, and confused. The breath
has a musty, sweet odor. Abdominal examination shows ascites and marked nodularity of the liver edge.
Neurologic examination is notable for asterixis. A toxicology screen is negative. Which of the following is
the most appropriate pharmacotherapy?
a) Amoxacellin
b) Rifuxamin
c) Silymarin
d) Spironolactone
e) Ursodeoxycolic acid
The answer is: Rifuxamin
26) Clue cells are seen in:#
a) Bacterial vaginosis
b) Candidiasis
c) Chlamydiasis
d) Trichomoniasis
e) Non of the above
The answer is: Chlamydiasis
27) What of the following causes macrocytic anemia?*
a) Thalassemia
b) Sickle cell anemia
c) Vitamin B12 deficiency anemia
d) Sideroblastic anemia
e) Anemia of chronic illness
The answer is: Vitamin B12 deficiency anemia
28) What is the SAAG ratio in ascites secondary to ovarian tumors?
a) 1
b) 5
c) 3
d) 4
e) 2
The answer is: 1

100
EMLE Trial Exams December 2020 (1000 Qs) Version (1.0) 22/12/2020

29) Disadvantages of Norplant include the following EXCEPT


a) It cannot be initiated and discontinued without a provider’s help
b) A minor surgical procedure is required for both insertion and removal
c) Provides no protection against STIs, including HIV
d) Side effects as bleeding irregularities are common
e) Effectiveness: 90% effective in preventing pregnancy
The answer is: Effectiveness: 90% effective in preventing pregnancy
30) 35 years old female presented to ER with palpation , cold extremities and dizziness . By examination blood
pressure was 60/40 , HR was 200 b/min . ECG showed wide QRS complex tachycardia . Which of the
following will be your first approach ?*
a) Intravenous beta-blockers
b) Intravenous calcium channel blockers
c) Termination of tachycardia by DC cardioversion
d) Intravenous amiodarone
e) Carotid sinus attack
The answer is: Termination of tachycardia by DC cardioversion
31) A 62-year-old man presents to the GP with a lump in the left groin which has been present for over 2
months. On examination, the lump is above the inguinal ligament. It is reducible and has a cough impulse,
but does not extend into the scrotum. Which of the following is the most likely diagnosis?*
a) Direct inguinal hernia
b) Femoral hernia
c) Gluteal hernia
d) Indirect inguinal hernia
e) Obturator hernia
The answer is: Direct inguinal hernia
32) The commonest presentation of gall stones is:
a) Pancreatitis
b) Biliary colic
c) Charcot’s triad
d) Jaundice
e) Fever
The answer is: Biliary colic
33) What is the best drug antibiotic to give in patients with hepatic encephalopathy?
a) Neomycin
b) Amoxycillin
c) Rifaximin
d) Ceftriaxone
e) Ciprofloxacin
The answer is: Rifaximin
34) Conjugated hyperbilirubinemia is seen in:
a) Gilbert’s syndrome
b) Criggler-Najjar syndrome
c) Dubin-Johnson syndrome
d) Breast milk jaundice
e) ABO
The answer is: Dubin-Johnson syndrome

101
EMLE Trial Exams December 2020 (1000 Qs) Version (1.0) 22/12/2020

35) A 29-year-old female complains of a gradual increase in a painless neck swelling. The history reveals that
she has remained well and she has sought advice for cosmetic reasons. Inspection her neck is shown in
the figure. What features of this swelling would you expect to elicit on clinical examination?
a) Depression of the swelling on swallowing
b) Elevation of the swelling on tongue protrusion
c) Presence of a sinus
d) The swelling gurgles on palpation
e) The swelling is pulsatile on palpation
The answer is: Elevation of the swelling on tongue protrusion
36) A 7-year-old girl is reported by her teachers as developing several ‘funny turns’ during which she becomes
vague and stops whatever she is doing for a few seconds, then continues again as if nothing has
happened.Which of the following epileptic type that is characterized by automatism?
a) Absence
b) Complex partial
c) Both absence and complex partial
d) Myoclonic
e) Atonic
The answer is: Both absence and complex partial
37) A 28-year-old female complaining of bilateral left-sided visual field problems most likely has:
a) Bilateral optic neuritis
b) A lesion in the right occipital lobe
c) A pituitary tumor
d) Multiple sclerosis
e) A lesion affecting the left optic nerve
The answer is: A lesion in the right occipital lobe
38) When to expect a normal child to say mama and dada?
a) 6 months
b) 1 year
c) 2 years
d) 2.5 years
e) 3 years
The answer is: 1 year
39) A 30-year-old Afro-Carribean woman presents to accident and emergency with a 1-week history of
progressive shortness of breath and fever. On further questioning, she mentions that her hands have been
painful and stiff over the past few months and she has been having recurrent mouth ulcers. Chest x-ray
confirms bilateral pleural effusions and blood tests reveal a raised ESR and a normal CRP. What is the most
likely diagnosis?*
a) Systemic lupus erythematosus
b) Systemic sclerosis
c) Sjögren’s syndrome
d) Discoid lupus
e) Beçhet’s disease
The answer is: Systemic lupus erythematosus
40) Oliguria is diagnosed when urine volume is:*
a) Less than 800-1600 ml / 24 hours
b) Less than 400-500 ml / 24 hours
c) Less than 100-400 ml / 24 hours

102
EMLE Trial Exams December 2020 (1000 Qs) Version (1.0) 22/12/2020

d) Less than 50-100 ml / 24 hours


e) None of the above
The answer is: Less than 400-500 ml / 24 hours
41) A 33-year-old woman on OCPs presents with progressive jaundice and right upper quadrant abdominal
pain. She has a history of DVT. On examination, there is tender hepatomegaly, ascites, bilateral ankle and
sacral edema. Spleen is not palpable.What is the most appropriate treatment?
a) Antiplatelet drugs
b) Anticoagulant drugs
c) Antiviral drugs
d) Chemotherapy
e) Antibiotics
The answer is: Anticoagulant drugs
42) A 55-year-old obese woman presents to the clinic for evaluation of multiple symptoms. She notes frequent
episodes of vaginal yeast infections in the past 2 months, recent weight loss in spite of a large appetite,
and waking up frequently at night to urinate. There is no history of fever or chills, and her only past
medical illness is hypertension that is treated with ramipril. Which of the following is the most likely
diagnosis?
a) diabetes mellitus (DM)
b) diabetes incipidus
c) vaginitis and cystitis
d) myxedema
e) pheochromocytoma
The answer is: diabetes mellitus (DM)
43) All of the following may require more than one dose of prophylactic antibiotic except?
a) Prolonged operations
b) Excessive blood loss
c) Gastrointestinal surgery
d) Insertion of prosthesis
e) Unexpected contamination.
The answer is: Gastrointestinal surgery
44) A 25-year-old man is admitted with nausea and frequent vomiting four hours after a meal in a restaurant.
Lastly he vomits a cup full of blood. Which of the following is the cause of his hematemesis?
a) Duodenal ulceration
b) Esophageal varices
c) Esophagitis
d) Hemorrhagic Gastritis
e) Mallory-Weiss tear
The answer is: Mallory-Weiss tear
45) 54 years old male patient presented to ER with sudden onset of chest pain one hour before presentation
, Which of the following conditions you need to exclude as soon as possible :*
a) Acute myocardail infarction
b) Acute pulmonary embolism
c) Acute aortic dissection
d) a, b and c
e) Acute cholecystitis
The answer is: a, b and c
46) Normal or elevated 25-hydroxly D3 with depressed 1,25 VD3 is found in:

103
EMLE Trial Exams December 2020 (1000 Qs) Version (1.0) 22/12/2020

a) Nutritinal rickets
b) Vitamin D dependent rickets- I
c) Vitamin D dependent rickets- II
d) X linked hypophosphatemic rickets
e) Osteopetrosis
The answer is: Vitamin D dependent rickets- I
47) Most common organism of acute bronchiolitis is:
a) Influenza virus
b) RSV
c) Parainfluenza virus
d) Rhinovirus
e) TB
The answer is: RSV
48) Characteristic features of a premature newborn include all of the following, except:
a) The muscle tone is increased
b) Have weak suckling & swallowing
c) Decreased subcutaneous fats
d) Decreased sweating mechanism
e) May have hypothermia
The answer is: The muscle tone is increased
49) In Spaulding classification, which of the following should undergo sterilization?
a) Stethoscopes
b) Respiratory masks
c) Flexible endoscopy
d) Prostheses
e) Blood pressure cuffs
The answer is: Prostheses
50) WHO defines anemia with pregnancy as follow:
a) Hemoglobin level below 11 g/dL.
b) MCV value below 80 fL
c) Hematocrit value less than 25%
d) MCHC below 33%
e) Reticulocytic count less than 2%
The answer is: Hemoglobin level below 11 g/dL.
51) You are the resident at the hospital and a child you are supervising found to be having dilated fixed pupil
what is the possible cause?
a) Opiates
b) Barbiturates
c) Severe hypoxia
d) Third nerve lesion
e) Tentorial herniation
The answer is: Severe hypoxia
52) A true statement about angioedema:
a) It usually causes pleural effusion
b) May cause suffocation
c) Treated by topical steroids
d) Treat with antibiotics

104
EMLE Trial Exams December 2020 (1000 Qs) Version (1.0) 22/12/2020

e) Just reassure
The answer is: May cause suffocation
53) Mark the incorrect statement#
a) Pauciarticular juvenile rheumatoid arthritis affects 4 joints or fewer
b) Henoch-Schӧnlein vasculitis is contagious
c) Neonatal lupus may cause congenital heart block
d) Juvenile dermatomyositis causes proximal muscle weakness
e) Lupus more common in femals
The answer is: Henoch-Schӧnlein vasculitis is contagious
54) Child 5 years old presented to you with severe pallor and tachycardia after assessing him you found that
his Hb is 5 gm what is your recommendation?*
a) Give Plasma
b) Give Packed RBC
c) Give fluids
d) Give glucose
e) Repeat the CBC
The answer is: Give Packed RBC
55) Which of the following quality measures is used for calculating Incidence of hypoglycemia among patients
who tried an intervention to lower HbAlc?#
a) Structural
b) Process
c) Outcome
d) balancing
e) Non-specific
The answer is: balancing
56) A 2-year-old G0 P0 woman has regular menses every 28 days. The semen analysis is normal. The patient
had a postcoital test revealing motile sperm and stretchy watery cervical mucus. She has been treated for
chlamydial infection in the past. Which of the following is the MOST LIKELY etiology of her infertility?
a) Peritoneal factor.
b) Male factor.
c) Cervical factor.
d) Uterine and tubal factor.
e) Ovulatory factor.
The answer is: Uterine and tubal factor.
57) 35 years old female presented to ER with palpation , cold extremities and dizziness . By examination blood
pressure was 60/40 , HR was 200 b/min . ECG showed wide QRS complex tachycardia . Which of the
following will be your first approach ?*
a) Intravenous beta-blockers
b) Intravenous calcium channel blockers
c) Termination of tachycardia by DC cardioversion
d) Intravenous amiodarone
e) Carotid sinus attack
The answer is: Termination of tachycardia by DC cardioversion
58) A 62-year-old man admitted 5 hours after left hemiplegia. ECG confirms AF. A CT head is normal. Which
of the following should be done in unilateral amaurosis fugax?*
a) CT angiography of the neck
b) MR angiography of the neck

105
EMLE Trial Exams December 2020 (1000 Qs) Version (1.0) 22/12/2020

c) Dupplex US of the neck


d) Cervical angiography
e) CT brain
The answer is: Dupplex US of the neck
59) A 53-year-old man presents to the GP with a deep, painful ulcer over the big toe. He gives a 3-month
history of severe calf pain on walking which is only eased on resting. Examination shows cool peripheries
with reduced distal pulses. Which ulcer does the patient most likely have?*
a) Arterial ulcer
b) Curling ulcer
c) Marjolin ulcer
d) Neuropathic ulcer
e) Venous ulcer
The answer is: Arterial ulcer
60) A 19-year-old man suffers a single gunshot wound to the left chest and is brought in by his friends. He is
complaining of chest pain. On examination, BP is 70/40 mm Hg, HR is 140 b/m, RR is 16 b/ m, temperature
is 37°C, and oxygen saturation is 96% on room air. He has distended neck veins, but his trachea is not
deviated. Lungs are clear to auscultation bilaterally. Heart sounds are difficult to appreciate, but you feel
a bounding, regular pulse. Two large-bore IV lines are placed and the patient is given 2L of normal saline.
Chest radiograph shows a globular cardiac silhouette but a normal mediastinum and no pneumothorax.
What is the definitive management of this patient?
a) Intubation
b) Tubethoracostomy
c) Pericardiocentesis
d) Thoracotomy
e) Blood transfusion
The answer is: Thoracotomy
61) A 34-year-old infertile woman is noted to have evidence of distally blocked distended fallopian tubes by
hysterosapingogram. Which of the following is the BEST NEXT step for this patient?
a) Short term trial of FSH therapy
b) Clomiphene citrate therapy
c) Intrauterineinsemination
d) IVF cycle
e) none of the above
The answer is: IVF cycle
62) The CORRECT statement for what happening during normal labor is that:
a) The head undergoes increased flexion to bring the longest longitudinal diameter of head to be presented
first to the maternal pelvis.
b) In occiptoanterior, the head undergoes internal rotation 3/8 of a circle.
c) Internal rotation is due to sloping shape of pelvic floor and rifling action of the bony pelvis.ç
d) Restitution is rotation of the head after delivery 1/8 of a circle in same direction of internal rotation.
e) None of the above is true.
The answer is: Internal rotation is due to sloping shape of pelvic floor and rifling action of the bony pelvis.ç
63) Which of the following is a cause of secondary (or exogenous) infection?*
a) Poor hand-washing technique
b) Hospital acquired infection
c) Perforated diverticular disease
d) Anastomotic leak

106
EMLE Trial Exams December 2020 (1000 Qs) Version (1.0) 22/12/2020

e) Inadequate air filtration in the theatre.


The answer is: Perforated diverticular disease
64) A 67-year-old woman who was conservatively treated in plaster for a Colles fracture re-presents to the
fracture clinic 2 weeks after plaster removal complaining of a burning pain in the whole hand. On
examination, the hand is slightly swollen, red and shiny. There is a reduction in all hand and finger
movements. Which of the following is most likely to be the cause of her symptoms?
a) Carpal tunnel syndrome
b) Compartment syndrome
c) Malunion of the fracture
d) Rupture of the extensor pollicis longus
e) Sudeck atrophy
The answer is: Sudeck atrophy
65) The bacterial infection that IS NOT sexually-transmitted:
a) Neisseria gonococci
b) Gardenerella vaginalis
c) Group B streptococci
d) Treponemapalladium
e) Trichomonas vaginalis
The answer is: Group B streptococci
66) A 16-year-old daughter refuses medication but her mother consents. What is the most appropriate action
to do?**
a) Write the prescription
b) Do not write the prescription
c) Postpone the prescription
d) Convince the daughter
e) Report the event to the police
The answer is: Write the prescription
67) Hemostasis and the cessation of bleeding require which of the following processes?#
a) Adherence of platelets to exposed subendothelial glycoproteins and collagen with subsequent
aggregation of platelets and formation of a hemostatic plug.
b) Interaction of tissue factor with factor VII circulating in the plasma.
c) The production of thrombin via the coagulation cascade with conversion of fibrinogen to fibrin.
d) Cross-linking of fibrin by factor XIII.
e) Non of the above
The answer is: Non of the above
68) Oliguria is diagnosed when urine volume is *
a) Less than 800-1600 ml / 24 hours.
b) Less than 400-500 ml / 24 hours.
c) Less than 100-400 ml / 24 hours.
d) Less than 50-100 ml / 24 hours.
e) None of the above
The answer is: Less than 400-500 ml / 24 hours.
69) A healthy mother delivers a term infant with microcephaly. The mother’s urine was found to contain cells
with inclusion bodies. Which of the following is the most likely diagnosis?
a) chromosomal abnormality
b) cytomegalovirus disease
c) syphilis

107
EMLE Trial Exams December 2020 (1000 Qs) Version (1.0) 22/12/2020

d) poliomyelitis
e) granuloma inguinale
The answer is: cytomegalovirus disease
70) Right sided sensory loss affecting legs more than the arms together with urine incontinence. What area is
the stroke most likely to be affected?#
a) ACA
b) MCA
c) PCA
d) ASA
e) AICA
The answer is: ACA
71) The main source of infection with viral hepatitis C is:
a) Blood and blood derivatives
b) Body fluids
c) Feces
d) Food
e) Contaminated soil
The answer is: Blood and blood derivatives
72) A 35-year-old man complains of chronic vague epigastric pain of several years 'duration. Endoscopy with
gastric antral biopsy demonstrates gastritis but no ulcerative lesions. H. pylori organisms are seen with
special stains on the biopsy fragments. The patient is treated with a 2-weeks course of omeprazole (20
mg bid), plus clarithromycin and metronidazole (500 mg bid each). Which of the following is the most
appropriate noninvasive test to ensure H pylori eradication?
a) Culture of gastric biopsy
b) Qualitative IgA and IgG antibodies against H. pylori
c) Quantitative IgA and IgG antibodies against H. pylori
d) Rapid urease test of gastric biopsy
e) Urea breath test
The answer is: Urea breath test
73) a 22-year- old female presents with anticholinergic crisis .She is delirious , agitated , and requires sedation
. Which of the following medications whould be the most appropriate to sedate this patient ?
a) Chlorpomazine
b) Fluphenazine
c) Lorazepam
d) Etomidate
e) Ketamine
The answer is: Lorazepam
74) Of the proposed etiologies for recurrent pregnancy wastage, the least likely is:
a) Maternal trauma.
b) Maternal balanced translocation.
c) Paternal balanced translocation.
d) Luteal phase deficiency.
e) Autoimmune disease.
The answer is: Maternal trauma.
75) What is the best treatment of trigeminal neuralgia?
a) Carbamazepine
b) Lorazepam

108
EMLE Trial Exams December 2020 (1000 Qs) Version (1.0) 22/12/2020

c) Phenytoin
d) Midazolam
e) Lamotrigine
The answer is: Carbamazepine
76) A 25-year-old male presents with paralysis of the right side of his face on waking this morning. On
examination, you note that the upper half of his face on the right side is functioning normally, but the
lower half is completely paralyzed. He does not have a rash and has not traveled recently. Which of the
following is the most appropriate next step in management?
a) Prednisone PO
b) Valacyclovir PO
c) Prednisone PO + valacyclovir PO
d) Doxycycline PO
e) CT brain
The answer is: CT brain
77) A 30-year-old woman presents with multiple bilateral breast swell-ings which cause her discomfort,
particularly just before her periods. They have been present for several years but appear to be getting
worse. What is the most likely diagnosis?*
a) Duct ectasia
b) Fat necrosis
c) Fibroadenoma
d) Fibrocystic disease
e) Peau d’orange
The answer is: Fibrocystic disease
78) The correct statement regarding norplant#
a) The main mechanism of action is on the ovulatory function
b) It contains estrogen
c) It is irreversible contraceptive
d) Nonserious side effects are rare
e) is composed of the progestin, levonorgestrel
The answer is: is composed of the progestin, levonorgestrel
79) A 60-year-old man complains of tiredness and significant weight loss. He notes episodes of rectal bleeding
with blood mixed with stool over the last few weeks. There is no diarrhea. Which of the following is the
most likely cause for the clinical findings?
a) Anal fissure
b) Colorectal carcinoma
c) Infectious diarrhea
d) Diverticulitis
e) Irritable bowel syndrome
The answer is: Colorectal carcinoma
80) A 55-year-old male with history of Diabetes and hypertension presents to the emergency department
with chest pain. ECG showed ST elevation. Which is the best initial step in management?
a) Aspirin
b) Percutaneous coronary angiography
c) Warfarin
d) Metoprolol
e) Xygen
The answer is: Aspirin

109
EMLE Trial Exams December 2020 (1000 Qs) Version (1.0) 22/12/2020

81) You see a 24-year-old student with suspected meningitis. The registrar asks you to perform a lumbar
puncture before she goes to the ward. This is not a procedure you have performed before, and you can
recall only limited information about the procedure from medical school teaching. What should you do?**
a) Explain to the registrar that this is outside of your competence and so you are not comfortable in
performing the procedure
b) Explain to the registrar this is not a skill that should be expected of you
c) Ask an older colleague to perform the procedure whilst you observe and learn for future attempts
d) As the procedure has many potential risks, await the patient's blood test results before deciding if she
really requires the procedure
e) Do not perform the procedure, but document clearly in the notes that it has been requested and will
need to be performed on the ward
The answer is: Explain to the registrar that this is outside of your competence and so you are not comfortable
in performing the procedure
82) Child 5 years old presented to you with severe pallor and tachycardia after assessing him you found that
his Hb is 5 gm what is your recommendation? *
a) Give Plasma
b) Give Packed RBC
c) Give fluids
d) Give glucose
e) Repeat the CBC
The answer is: Give Packed RBC
83) A 27-year-old man presents with pain in his left testis that has been present for the past 2 days. The pain
has been getting gradually worse and there is now also noticeable swelling of the testis. On examination
he is febrile with a temperature of 38.3°C and the scrotum appears red and oedematous on the affected
side. The testis is tender on palpation. The most likely diagnosis is:
a) Scrotal haematoma
b) Testicular torsion
c) epididymo-orchitis
d) torsion of hydatid of Morgagni
e) irreducible inguinoscrotal hernia
The answer is: epididymo-orchitis
84) A 74-year-old woman presents with a 3-month history of worsening constipationand rectal bleeding. The
most appropriate investigation is:
a) barium swallow
b) flexible sigmoidoscopy
c) watch and wait
d) faecal occult blood
e) CT abdomen
The answer is: flexible sigmoidoscopy
85) A man has the risks and potential benefits of his inguinal hernia explained to him. Which ethical principle
is best described here?#
a) Gillick competence
b) Assent
c) Consent
d) Autonomy
e) Capacity
The answer is: Consent

110
EMLE Trial Exams December 2020 (1000 Qs) Version (1.0) 22/12/2020

86) When is the age of weaning:#


a) 3 months
b) 6 months
c) 9 months
d) 12 months
e) 15 months
The answer is: 6 months
87) How is symmetrical growth restriction characterized?
a) Reduction in head size
b) Reduction in body size
c) Reduction in both body size and head size
d) Reduction in body and femur length
e) None of the above
The answer is: Reduction in both body size and head size
88) The following is true regarding hepatitis A:
a) Airborne infection
b) Always symptomatic
c) Incubation period 46 days
d) Infections more severe in adults
e) Can progress into liver cirrhosis
The answer is: Incubation period 46 days
89) A 10 year-old girl moves into your catachment area and presents to your clinic saying she has been
monitoring in the past for her chronic renal failure from reflux nephropathy you advice her of sypmtoms
that can be the result of chronic renal disease and arrange approciate monitoring of her which of the
following is not seen in chronic renal disease ?
a) anaemia
b) hypertension
c) hypercalcamia
d) hypokalaemia
e) short stature
The answer is: hypokalaemia
90) Oliguria is diagnosed when urine volume is *
a) Less than 800-1600 ml / 24 hours.
b) Less than 400-500 ml / 24 hours.
c) Less than 100-400 ml / 24 hours.
d) Less than 50-100 ml / 24 hours.
e) None of the above
The answer is: Less than 400-500 ml / 24 hours.
91) Which of the following is TRUE regarding the use of imaging in the evaluation of blunt cerebrovascular
injuries (BCVI)
a) CTA is the most accurate test for the evaluation of BCVI
b) MRA has been shown to be as accurate as CTA for the evaluation of BCVI
c) Injury patterns that place patients at relatively high risk for BCVI include: C1/C2 fractures, basal skull
fractures, cervical spine fractures with involvement of the transverse foramen, Le Fort II or III fractures.
d) The incidence of injuries in most screening series is 5–10%
e) If an injury is identified on imaging, anticoagulation with aspirin for 6 months is adequate therapy.

111
EMLE Trial Exams December 2020 (1000 Qs) Version (1.0) 22/12/2020

The answer is: Injury patterns that place patients at relatively high risk for BCVI include: C1/C2 fractures, basal
skull fractures, cervical spine fractures with involvement of the transverse foramen, Le Fort II or III fractures.
92) Most common cause of nephritic syndrome in children:
a) Acute post streptococcal glomerulonephritis
b) Hemolytic uremic syndrome
c) Systemic lupus
d) Malaria
e) Gold therapy
The answer is: Acute post streptococcal glomerulonephritis
93) Glucose tolerance test is indicated in all, EXCEPT:*
a) Previous Congenital Anomaly.
b) Previous eclampsia.
c) Polyhydramnios.
d) Previous unexpected fetal death.
e) BMI of 32 kg/m .
The answer is: Previous eclampsia.
94) At eight weeks’ gestation, a woman has an intrauterine device (IUCD) and the strings are visible at the
external os. The BEST management is:
a) Antibiotics as prophylactic measure against septic abortion
b) Progesterone as prophylactic measure against abortion
c) Dilation and curettage
d) Removal of IUCD
e) Bed rest and observation
The answer is: Bed rest and observation
95) Regarding Compartment Syndrome which bone fracture is MOST commonly implicated?
a) Femur
b) Radius
c) Tibia
d) Humerus
e) cervical spine
The answer is: Tibia
96) What of the following is seen in the blood film of G6PD deficiency?#
a) Schistocytes
b) Heinz bodies
c) Target cells
d) Anisocytes
e) Poikilocytes
The answer is: Heinz bodies
97) You are seeing for the first prenatal visit a 19-year-old woman with an artificial porcine valve placed 6
months ago for CHD. She is 10 weeks pregnant, tired, and does not sleep particularly well. The fetus is
size-date appropriate. Of the following choices, which is the best first step in management?
a) evaluate for valve replacement due to assumed cardiac enlargement
b) anticoagulate with aspirin and a platelet inhibitor
c) recommend termination of the pregnancy as she had not postponed conception for the mandatory 24
months after valve replacement
d) anticoagulate with heparin
e) keep on low-dose oral antibiotics

112
EMLE Trial Exams December 2020 (1000 Qs) Version (1.0) 22/12/2020

The answer is: anticoagulate with heparin


98) A 34-year-old woman (gravida 3, para 2) at 35 weeks’ gestation complains of sharp, excruciating pain in
the right flank radiating into her groin. No chills or fever have been noted. The pain resolved shortly after
the patient was seen. Urinary analysis reveals numerous red blood cells (RBCs), some WBCs, and no
bacteria. WBC and hematocrit are normal. Which of the following options is the most likely diagnosis?
a) appendicitis
b) pyelonephritis
c) round ligament pain
d) ureteral lithiasis
e) Meckel’s diverticulum
The answer is: ureteral lithiasis
99) A 28-year-old professional football player has sudden pain and swelling in the right groin when attempting
to intercept a pass. He is admitted to the local emergency department. On examination, there is a tender
swelling in the right groin. The scrotum and penis show no abnormality. What is the next step in
management?*
a) Needle aspiration to exclude hematoma
b) Forceful manual reduction
c) Preoperative preparation and exploration of the groin with hernia repair
d) Laparotomy within 20 minutes
e) Morphine and re-evaluation within 12 hours
The answer is: Preoperative preparation and exploration of the groin with hernia repair
100) Which of the following is the most common cause of thrombocytopenia in children?
a) Drug reaction
b) Infection
c) Henoch-Schӧnlein purpura
d) Idiopathic thrombocytopenic purpura
e) Hemophilia
The answer is: Idiopathic thrombocytopenic purpura

113
EMLE Trial Exams December 2020 (1000 Qs) Version (1.0) 22/12/2020

Exam 7
1) Which of the following is uncommon finding in RDS (hyaline membrane disease)?
a) Tachypnea
b) Wheezing
c) Grunting
d) Retractions
e) Cyanosis
The answer is: Wheezing
2) What of the following causes normocytic anemia?*
a) Thalassemia
b) Sideroblastic anemia
c) Iron deficiency anemia
d) Sickle cell anemia
e) Vitamin B12 deficiency anemia
The answer is: Sickle cell anemia
3) A 50-year-old man presents with a complaint of a 1-cm moderately painful, tender mass situated one-
third of the way between the xiphisternum and the umbilicus (Fig. 8–2). What is the most likely
diagnosis?
a) Fat necrosis
b) Spigelian hernia
c) Omphalocele
d) Fibrosarcoma of the abdominal wall
e) Epigastric hernia
The answer is: Epigastric hernia
4) An infant presented with repeated bouts of loose stools, his mother reported that he is always thirsty.
On examination, his tongue was very dry, his skin was warm to touch but with some loss of skin turgor.
2 hours later, he started to convulse. Which results of the following can explain features in this case?
a) Serum sodium level of 160 mEq/L
b) Serum potassium level of 6 mEq/L
c) Serum calcium level of 5 mg/dL
d) Serum potassium level of 2.5 mEq/L
e) Serum sodium level of 125 mEq/L
The answer is: Serum sodium level of 160 mEq/L
5) Pathophysiologic consequences of preeclampsia include all of the following EXCEPT:*
a) Decreased placental perfusion
b) Increased plasma uric acid
c) Decreased glomerular filtration rate
d) Increased intravascular volume
e) Oliguria
The answer is: Increased intravascular volume
6) The INCORRECT statement for trichomnas vaginalis:
a) Symptoms exacerbate after menstrual flow
b) The main method of transmission is at sexual intercourse
c) It may be a cause of postcoital bleeding
d) Decreased vaginal acidity is a predisposing factor
e) Asymptomatic carriers are rare

114
EMLE Trial Exams December 2020 (1000 Qs) Version (1.0) 22/12/2020

The answer is: Asymptomatic carriers are rare


7) A 36-year-old woman presents with a 2-month history of recurrent severe right-side headaches
associated with vomiting and blurring of vision. She feels generally unwell, lethargic and nauseated
about 12-24 hours before each episode. What is the cause of this headache?*
a) Subarachinoid hemorrhage
b) Cluster headache
c) Migraine
d) Tension headache
e) Giant cell arteritis
The answer is: Migraine
8) A 22-year-old primigravida at 34 weeks’ gestation presents with moderate vaginal bleeding and no
uterine contractions. The following sequence of examinations is the most appropriate:#
a) Ultrasound examination, speculum examination, digital examination.
b) Digital examination, speculum examination, ultrasound examination.
c) Speculum examination, ultrasound examination, digital examination.
d) Digital examination, ultrasound examination, speculum examination.
e) Ultrasound examination, digital examination, speculum examination.
f) Ultrasound examination, speculum examination, digital examination.
9) Commonest cause of vomiting in infants include the following:
a) Gastroenteritis
b) Systemic infection
c) Overfeeding
d) Malignancy
e) Gastroesophageal Reflux
The answer is: Malignancy
10) A 24-year-old man is brought in following a road traffic collision. He is suspected to have intra-
abdominal injuries and there is bruising over his trunk. On arrival, he has a pulse rate of 138/min and his
blood pressure is 80/50 mmHg. Which of the following fluids would raise the patient’s blood pressure
rapidly?
a) Mannitol
b) 5% dextrose
c) Packed red cells
d) Normal saline
e) Fresh frozen plasma
The answer is: Packed red cells
11) A known diabetic kid in your apartment you found him complaining of abdominal pain, you feel he is
drowsy and you smelled acetone from his breath what is you recommendation?*
a) Give anti spasmodic
b) Ask him to drink some juice
c) Reassure
d) Prescribe antibiotic
e) Advice to parents to go urgently to hospital
The answer is: Advice to parents to go urgently to hospital
12) You found an accident in the street, what is your first response***^
a) Start CPR
b) Pass by as you have no license yet
c) Call ambulance

115
EMLE Trial Exams December 2020 (1000 Qs) Version (1.0) 22/12/2020

d) Call a friend
e) Try to move the people inside
The answer is: Call ambulance
13) A 29-year-old female is noted to have an elevated bilirubin during a viral illness. Gilbert's syndrome is
suspected. Which one of the following tests may confirm the diagnosis?*
a) CBC
b) Urine analysis
c) Fecal fat excretion
d) Nicotinic acid test
e) HIDA scan
The answer is: Nicotinic acid test
14) A 34-year-old man presents to the emergency department with a 10-hour history of abdominal pain
associated with nausea and vomiting. On examination, the patient is lying still and has tenderness with
guarding in the right iliac fossa. His temperature is 37.6°C. He has no significant past medical history.
What is the most likely diagnosis?*
a) Mesenteric adenitis
b) Renal colic
c) Crohn disease
d) Appendicitis
e) Meckel diverticulitis
The answer is: Appendicitis
15) Posterior fontanel is usually completely closed by:
a) 12 months of age
b) 10 months of age
c) Shortly after birth
d) 24 months of age
e) 18 months of age
The answer is: Shortly after birth
16) Which one of the following does not occur as a systemic manifestation of inflammatory bowel disease?*
a) Scleritis
b) Amyloidosis
c) Sclerosing cholangitis
d) Cardiomyopathy
e) Pyoderma gangrenosum
The answer is: Cardiomyopathy
17) Which of the following should be avoided during hand washing?**
a) Washing finger webs
b) Nail brushes
c) Paper towels
d) Liquid soap
e) Complete drying
The answer is: Nail brushes
18) A 24-year –old man is presented to the ER after becoming unwell , drosy and disoriented . He has no
history of medical problems . Over the last few days he has become progressively weak , lethargic ,
drosy and disoriented . The patient appears confused , temperature 36.5 C , with cool extremeties , poor
capillary refill , pulse 120 bpm , BP 75/55mmHg , K 5.6mmol/L , Na 125mmol/L , Cl 101mmol/L , Hb
13g/dL , glucose 65mg/dL . After BLS and fluid resuscitation , what is the most emergent next step ?*

116
EMLE Trial Exams December 2020 (1000 Qs) Version (1.0) 22/12/2020

a) IV hydrocortisone
b) IV thyroid hormone
c) IV antibiotics
d) IV glucose
e) IV Noradrenaline
The answer is: IV hydrocortisone
19) You found a child convulsing in front of you ,what is your first response*
a) Support in best position to avoid that he might hurt himself
b) None of the above
c) Try to move him
d) Try to awake him
e) Do CPR
The answer is: Support in best position to avoid that he might hurt himself
20) How many breaths and compressions you do in CPR ( compressions- breath)*
a) 43956
b) 30-30
c) 30-2
d) 30-15
e) 5-2
The answer is: 30-2
21) A 30-year-old male with presents with dyspnea on exertion for the past 1 year. His liver enzymes are
abnormal. Pulmonary function tests were done and were as follows: FEV1/FVC 60% (normal >80%) DLCO
decreased Which is the most likely cause of the most likely diagnosis?
a) Tumor
b) Smoking
c) Infection
d) Alpha 1 antitrypsin deficiency
e) Type I hypersensitivity
The answer is: Alpha 1 antitrypsin deficiency
22) A 17-year- old football player suddenly collapsed and died while playing a game without trauma. Which
of the following is the most likely to be the cause ?
a) Subarachnoid hemorrhage
b) Spontaneous pneumothorax
c) Coronary artery disease
d) Hypertrophic cardiomyopathy
e) Pulmonary embolism
The answer is: Hypertrophic cardiomyopathy
23) 17-year-old student is brought to A&E after she had a witnessed fit. History was given by a friend as she
was too drowsy. She suddenly lost consciousness Awhile sitting on a sofa, fell to the floor, then became
rigid and started shaking all limbs vigorously. She bit her tongue and was incontinent of urine.What is
your diagnosis?*#
a) Hyponatremia
b) Hypomagnesemia
c) Absence seizures
d) Tonic-clonic seizures
e) Hypocalcemia
The answer is: Tonic-clonic seizures

117
EMLE Trial Exams December 2020 (1000 Qs) Version (1.0) 22/12/2020

24) A 22-year-old married female comes to the emergency department with a 5-day history of severe
abdominal pain, cramping, nausea, and vomiting. She also has pain with urination. She is sexually active
with her husband, and they use condoms inconsistently. She experienced a burning pain when she last
had sexual intercourse 3 days ago. Menses occur at regular 28-day intervals and last 5 days. Her last
menstrual period was 3 weeks ago. Her temperature is 38.5°C (101.3°F), pulse is 83/min, and blood
pressure is 110/70 mm Hg. Physical examination shows abdominal tenderness in the lower quadrants.
Pelvic examination shows cervical motion tenderness and purulent cervical discharge. Laboratory
studies show a leukocyte count of 15,000/mm3 and an erythrocyte sedimentation rate of 100 mm/h.
Which of the following is the most likely diagnosis?
a) Pyelonephritis
b) Pelvic inflammatory disease
c) Cystitis
d) Ectopic pregnancy
e) Ovarian cyst rupture
The answer is: Pelvic inflammatory disease
25) A woman with advanced metastatic breast cancer is receiving palliative care treatment. She has a
subcutaneous morphine and midazolam infusion in situ, but despite this remains in considerable pain
and distress. You prescribe a bolus dose of morphine to treat her pain, and she quickly suffers a
respiratory arrest and dies. Which of the ethical principles is best described in this question?
a) Non-maleficence
b) Doctrine of double effect
c) Informed consent
d) Beneficence
e) Autonomy
The answer is: Doctrine of double effect
26) A 20-year-old man presents to his GP with a testicular swelling. His serum lactate dehydrogenase (LDH)
is raised with a mildly raised human chorionic gonadotrophin (HCG) and a normal alpha-fetoprotein.
What malignancy does he have?
a) Teratoma
b) Adenocarcinoma
c) Squamous cancer
d) Anaplastic cancer
e) Seminoma
The answer is: Seminoma
27) What of the following causes macrocytic anemia?*
a) Anemia of chronic illness
b) Sickle cell anemia
c) Sideroblastic anemia
d) Thalassemia
e) Vitamin B12 deficiency anemia
The answer is: Vitamin B12 deficiency anemia
28) A 34-year-old woman is noted to have significant uterine bleeding after a vaginal delivery complicated
by placenta abruption. She is noted to be bleeding from multiple venipuncture sites. Which of the
following is the best therapy?
a) Hypogastric artery ligation.
b) Immediate hysterectomy.
c) Packing of the uterus.
d) Ligation of utero-ovarian ligaments.

118
EMLE Trial Exams December 2020 (1000 Qs) Version (1.0) 22/12/2020

e) Correction of coagulopathy.
The answer is: Correction of coagulopathy.
29) A 24-year-old primigravida, in labor for 16 hours and the cervix is arrested at 9 cm for 1 hour, position is
ROP, station 0 and moulded. There are fetal late decelerations over the last 30 minutes. Delivery is BEST
managed by:
a) Lower segment cesarean section (LSCS).
b) Forceps rotation and then traction.
c) Ventouse rotation (vacuum) rotation and then extraction.
d) Augmentation of labor by oxytocin.
e) Neither of the above.
The answer is: Lower segment cesarean section (LSCS).
30) Advising a 34-year-old woman at 12 weeks’ gestation about the risk of chromosomal defects in the
fetus, you can correctly state which of the following?
a) Efficacy of screening for Down syndrome is improved by adding estriol, inhibin A, and hCG
concentration to the MSAFP (quadruple screen).
b) There is little worry regarding Down syndrome before the age of 35.
c) Screening for Down syndrome
d) Maternal serum alpha-fetoprotein (MSAFP) is a very specific test for Down syndrome.
e) Paternal age is very important in the etiology of Down syndrome.
The answer is: Efficacy of screening for Down syndrome is improved by adding estriol, inhibin A, and hCG
concentration to the MSAFP (quadruple screen).
31) A 14-year-old schoolboy presents to the Emergency Department after accidentally pouring a cup of hot
coffee over his right hand. He is in obvious discomfort and as part of his on site first aid, his mother ran
his hand under cold running water for 20 minutes. You are called to examine his burns and notice that
most of the dorsum of his right hand is affected. In order to discuss his case with the regional burns
centre you wish to distinguish whether this is a partial thickness or full thickness burn. From the
following list, which features, on examination would suggest a full thickness burn over a partial
thickness injury?*
a) Painless and not sensitive to light touch
b) Blistering
c) Discharging fluid and weeping
d) Painful and sensitive to light touch
e) Swollen and erythematous
The answer is: Painless and not sensitive to light touch
32) An infant is born with hydrocephalus. CT studies demonstrate herniation of the cerebellum into the
spinal canal. This is an example of which of the following?
a) CP
b) Holoprosencephaly
c) Porencephaly
d) Lissencephaly
e) Arnold Chiari malformation
The answer is: Arnold Chiari malformation
33) A known diabetic kid in your apartment you found him complaining of abdominal pain, you feel he is
drowsy and you smelled acetone from his breath what is you recommendation?*
a) Ask him to drink some juice
b) Reassure
c) Prescribe antibiotic

119
EMLE Trial Exams December 2020 (1000 Qs) Version (1.0) 22/12/2020

d) Advice to parents to go urgently to hospital


e) Give anti spasmodic
The answer is: Advice to parents to go urgently to hospital
34) A 6-year-girl is noted to have breast development and vaginal spotting. No abnormal hair growth is
noted. A 10-cm ovarian mass is palpated on rectal examination. Which of the following is the MOST
LIKELY diagnosis?
a) Congenital adrenal hyperplasia.
b) Granulosa-theca cell tumor.
c) Sertoli-Leydig cell tumor.
d) Idiopathic precocious puberty.
e) Benign cystic tumor (dermoid).
The answer is: Granulosa-theca cell tumor.
35) A 28-year-old man presents to the emergency department complaining of abdominal discomfort, nausea
and vomiting of 3 days duration. He has no significant past medical history and denies any alcohol or
drug abuse. On examination, he has scleral icterus and mildly enlarged soft liver. Investigations revealed
total bilirubin 5.2 mg/dL (n= 0.1 to 1.2 mg/dL), direct bilirubin 4 mg/dL. SGPT 800 U/L (n = 0-49) and
SGOT 620 U/L (n = 0-34). Alkaline phosphatase 80 U/L (n up to 90). Other liver function tests are normal.
Abdominal ultrasonography is normal. Which of the following is the most appropriate diagnosis?
a) Chronic hepatitis
b) Hemolytic jaundice
c) Liver cirrhosis
d) Cholestatic jaundice
e) Acute hepatitis
The answer is: Acute hepatitis
36) The standard initial therapy for acute sigmoid volvulus is:
a) IV neostigmine
b) Laparotomy to reduce the volvulus and replace the sigmoid colon to its normal position
c) Rigid sigmoidoscopy
d) Ileostomy
e) Colonoscopy
The answer is: Rigid sigmoidoscopy
37) A 74-year old man presents with frontal headache, pain on jaw movement and weight loss. He had had a
similar problem 3 years earlier which had responded to prednisolone. Temporal artery biopsy is normal.
What is the diagnosis?
a) Migraine
b) Giant cell arteritis
c) Tension headache
d) Cluster headache
e) Subarachinoid hemorrhage
The answer is: Giant cell arteritis
38) .What is the most confirmatory test for the detection of H-pylori?
a) Urea breath test
b) UGIT endoscopy
c) Stool antigen
d) CT abdomen
e) Serology
The answer is: UGIT endoscopy

120
EMLE Trial Exams December 2020 (1000 Qs) Version (1.0) 22/12/2020

39) You see a 72-year-old man in your Pre-operative Assessment clinic prior to an elective open left inguinal
hernia repair. He is currently taking Warfarin for atrial fibrillation and he asks you if he needs to stop it
before his operation. What do you advise him?
a) Stop 14 days before
b) Stop 10 days before
c) Omit morning of surgery
d) No need to stop
e) Stop 5 days before
The answer is: Stop 5 days before
40) The most common cause of acute hepatitis with transaminase above 1000 IU/L is:
a) HBV
b) CMV
c) HCV
d) HAV
e) EBV
The answer is: HAV
41) In renal rickets (Glomerular) there is:
a) Early onset of rickets
b) High serum calcium
c) Good response to vitamin D
d) High serum phosphorus
e) Low serum alkaline phosphatase
The answer is: High serum phosphorus
42) RH disease :
a) Occurs when the father is RHC.
b) Occurs when the mother is Rh+
c) Occurs when the mother is Rh -ve & fetus is Rh +ve
d) Antibodies are formed against maternal RBCs
e) Can never occurs in the 1st pregnancy
The answer is: Occurs when the mother is Rh -ve & fetus is Rh +ve
43) ‘Sign’ that refers to a spontaneous cessation of inspiration on palpation of the hypochondrium just
below the right costal margin. What is the eponymous sign or law being described?*
a) Battle’s
b) Rovsing’s
c) Courvoisier’s
d) Murphy’s
e) Grey Turner’s
The answer is: Murphy’s
44) Which immunoglobulin will be elevated in bronchial asthma?
a) IgM
b) All of the immunoglobulines
c) IgG
d) IgE
e) IgA
The answer is: IgE
45) Which sign of following is most alarming for acute aortic dissection in a 51 years old male presenting
with acute chest pain ?*

121
EMLE Trial Exams December 2020 (1000 Qs) Version (1.0) 22/12/2020

a) Tachycardia
b) Pallor
c) Unequal pulse bilaterally
d) Pulmonary venous congestion
e) Hypotension
The answer is: Unequal pulse bilaterally
46) Patient presents with carcinoid syndrome and developes hypotension intraoperatively. Best drug to
treat it is:
a) Octreotide
b) Noradrenaline
c) Adrenaline
d) Ephedrine
e) Metaraminol
The answer is: Octreotide
47) Because of concerns of a maternal renal stone, a renal ultrasound is ordered in a 35-year-old G4P3003 at
34 weeks’ estimated gestational age (EGA). Renal imaging during the eighth month of gestation
normally reveals what?
a) hydroureter bilaterally
b) the same findings as those of a normal, nonpregnant woman
c) mild to moderate ureteral reflux
d) ureteral spasm above the pelvic brim
e) nephroptosis
The answer is: hydroureter bilaterally
48) Each one of the following is a recognised side-effect of erythropoietin, except:
a) Long bone fractures
b) Pure red cell aplasia
c) Hypertension
d) Bone aches
e) Urticarial
The answer is: Long bone fractures
49) A 72-year-old woman with 3 months of change in bowel habit including frequent diarrhoea and urgency
but no blood in the stool. Her weight is stable and there are no masses palpable. choose the single most
appropriate first investigation:#
a) Barium enema (double contrast)
b) CT scan of the abdomen and pelvis
c) Colonoscopy
d) Flexible sigmoidoscopy
e) Endoanal ultrasound scan
The answer is: Colonoscopy
50) A 29-year-old female is noted to have an elevated bilirubin during a viral illness. Gilbert's syndrome is
suspected .Which of the following is elevated in Gilbert’s syndrome?
a) Unconjugated bilirubin
b) Conjugated bilirubin
c) Alkaline phosphatase
d) Reticulocytes
e) GGT
The answer is: Unconjugated bilirubin

122
EMLE Trial Exams December 2020 (1000 Qs) Version (1.0) 22/12/2020

51) Glucose tolerance test is indicated in all, EXCEPT:*


a) Polyhydramnios
b) Previouscongenitalanomaly
c) Previous unexpected fetal death
d) BMI of 32 kg/m
e) Previous eclampsia
The answer is: Previous eclampsia
52) The causes of wheezy child include the following except:
a) Acute bronchiolitis
b) Bronchial asthma
c) Allergic bronchopulmonary aspergillosis
d) Pneumonia
e) Foreign body in the airway
The answer is: Pneumonia
53) The mother of a minor consents to medication but the father refuses. What is the most appropriate
action to do?**
a) Convince the daughter
b) Postpone the prescription
c) Write the prescription
d) Do not write the prescription
e) Report the event to the police
The answer is: Write the prescription
54) In Kawasaki disease, all are true except
a) In acute phase, C-reactive protein is usually normal
b) Cervical LN
c) Pharyngeal injection, dry fissured lips, injected lips and strawberry tongue
d) Bilateral non-suppurtive conjunctivitis
e) Unexplained fever of more than 5 days duration
The answer is: In acute phase, C-reactive protein is usually normal
55) A 32-year-old woman comes to the physician because of sudden onset of palpitations and fatigue. She
says that she has been in good health until a couple of months ago, when she began to lose weight
despite an apparent increase in appetite. She also has insomnia and increasing anxiety. She is 168 cm
tall and weighs 50 kg. Her temperature is 37.9 oC, blood pressure is 140/65 mm Hg, and pulse is 120/min
and irregular. An ECG reveals atrial fibrillation. Which of the following is the most appropriate next step
in diagnosis?
a) Echocardiogram
b) Neuropsychiatric referral
c) Blood culture and sensitivity
d) Complete blood count
e) Thyroid function tests
The answer is: Thyroid function tests
56) You are at the word round and suddenly a kid collapsed in coma with unilateral dilated pupil what
would you suspect?*
a) Opiates
b) Severe hypoxia
c) Tentorial herniation
d) Barbiturates

123
EMLE Trial Exams December 2020 (1000 Qs) Version (1.0) 22/12/2020

e) Hypothermia
The answer is: Tentorial herniation
57) What of the following is an absolute contraindication to thrombolytic in stroke?*^#
a) Major surgery in preceding 2 weeks
b) Active pancreatitis
c) BP >200/120 mmHg
d) Hemorrhagic diathesis
e) pregnancy
The answer is: BP >200/120 mmHg
58) Clinical manifestations of anaphylaxis include which of the following:
a) Hypotension
b) cough
c) hypertension
d) sore throat
e) cold skin
The answer is: Hypotension
59) A 22-year- old female at 36 weeks of gestation is being treated with magnesium sulfate for pre-
eclampsia while awaiting transfer to a nearby hospital for definitive management . On arrival she was
quite somnolent with markedly depressed deep reflexes and decreased respiratory rate . After
managing the airway , the most reliable next step is intravenous ;*
a) Atropine
b) Labetalol
c) Lidocaine
d) dexamethasone
e) Calcium gluconate
The answer is: Calcium gluconate
60) A missed abortion is :
a) Bleeding and cramps but the fetus still viable
b) Death of the fetus at 36 weeks of gestation
c) In which the products of conception are partially expelled
d) Death of the fetus before 20 weeks gestation
e) In which the products of conception are expelled completely
The answer is: Death of the fetus before 20 weeks gestation
61) A 53-year-old male complains of fatigue, dyspnea, and orthopnea. Which one of the following would
have the highest specificity for heart failure?*
a) A third heart sound (S3 gallop)
b) Cardiomegaly on a chest radiograph
c) Fine crackles
d) Ankle edema
e) Decreased B-type natriuretic peptide (BNP)
The answer is: A third heart sound (S3 gallop)
62) You are shadowing the Breast Surgery Consultant during a clinical consultation. He begins to explain
that he has a patient who is due for a right-sided axillary lymph node clearance the following day. He
tells you that he will perform a Level 3 axillary lymph node clearance. From the list below, please select
the answer that best describes the location of Level 3 axillary lymph nodes.
a) Posterior to pectoralis minor
b) Posterior to pectoralis major

124
EMLE Trial Exams December 2020 (1000 Qs) Version (1.0) 22/12/2020

c) Lateral to pectoralis minor


d) Superomedial to pectoralis minor
e) Anterior to pectoralis major
The answer is: Superomedial to pectoralis minor
63) Shock can best be defined as:#
a) Hypoperfusion of tissues.
b) Hypotension.
c) All of the above.
d) Hypoxemia.
e) Non of the above
The answer is: Hypoperfusion of tissues.
64) Anaphylaxis could result from all of the following, except?
a) Latex
b) Insect sting
c) peanuts
d) Intravenous immunoglobulin therapy
e) breast milk
The answer is: breast milk
65) A primigravida, 35 weeks’ gestation presents with first episode of painless bleeding yesterday. O/E: BP
120/70 mmHg, uterus relaxed, and cephalic floating fetal head, FHS regular, Hb 10g/dl. Next line of
management is#
a) Blood transfusion
b) Cesarean section
c) Wait and watch
d) Induction of labor
e) none of the above
The answer is: Wait and watch
66) An 18-year-old male student presents to the emergency department with fever, headache and
photophobia. On examination, he is pyrexial at 39ºC and has observable neck stiffness. There is no
evidence of rash and other than a recent ear infection, there is no other relevant past medical history .
Which of the following is the most appropriate intravenous treatment option?#
a) Cephalexin
b) Ceftriaxone
c) Ceftriaxone and dexamethasone
d) Benzylpenicillin
e) Amoxicillin
The answer is: Ceftriaxone and dexamethasone
67) A major hazard of a late missed abortion :
a) Toxemia
b) Systemic allergies
c) Bone marrow depression
d) Coagulopathy
e) A positive human chorionic gonadotropin (hCG) titer
The answer is: Coagulopathy
68) 54 years old male patient presented to ER with sudden onset of chest pain one hour before presentation
, Which of the following conditions you need to exclude as soon as possible :*
a) Acute cholecystitis

125
EMLE Trial Exams December 2020 (1000 Qs) Version (1.0) 22/12/2020

b) Acute myocardail infarction


c) Acute pulmonary embolism
d) Acute aortic dissection
e) a , b and c
The answer is: a , b and c
69) A 75-year-old man presents to the Emergency Department with sudden onset frank haematuria. Five
days earlier he required catheterization following an episode of postoperative urinary retention after an
elective inguinal hernia repair. The catheter was removed on discharge and he has passed urine freely
since. On examination, the urine is frank haematuria with evidence of blood clots. What is the most
likely diagnosis?*
a) Transitional cell carcinoma
b) Renal calculi
c) Urinary tract infection
d) Benign prostatic hypertrophy
e) Urethral trauma
The answer is: Urinary tract infection
70) The most reliable investigation to diagnose cholecystitis is:
a) MRI
b) lV cholangiography
c) CT scan
d) Pelviabdominal ultrasonography
e) PET scan
The answer is: Pelviabdominal ultrasonography
71) The INCORRECT statement regarding the prognosis of cervical cancer:
a) The majority of recurrence occurs after 3 years of treatment
b) Recurrence occurs in 35% of cases
c) The 5 years- survival for stage II is 50%.
d) In stages IB & II A there is little difference between results of surgery & radiotherapy
e) Surgical management of local recurrence following radical surgery is disappointing
The answer is: The majority of recurrence occurs after 3 years of treatment
72) A 19-year-old man presents to the ENT outpatient clinic with a gradually increasing midline neck
swelling. On examination, the swelling is 1 x 1 cm, non-tender and immobile. It moves on protrusion of
the tongue. What is the most appropriate diagnosis?*
a) Thyroglossal cyst
b) Pharyngeal pouch
c) Pleomorphic adenoma
d) Goitre
e) Cervical rib
The answer is: Thyroglossal cyst
73) Parotid duct opens into mouth opposite
a) at the anterior palate
b) Behind third molar
c) Upper third molar
d) Near to midline of hard palate
e) Upper second molar
The answer is: Upper second molar
74) A one-year old infant can

126
EMLE Trial Exams December 2020 (1000 Qs) Version (1.0) 22/12/2020

a) Know his name


b) Run
c) Throw ball above head
d) Says full sentence
e) Jump
The answer is: Know his name
75) A 4-week-old infant is brought to the emergency department by his parents after he develops an
episode of emesis with an observed period of apnea. Three other infants were seen there earlier this
week with the flu. The infant is discharged home with instructions for flu management, but the parents
return with him later, reporting that he had another episode of apnea. The patient is further evaluated
and subsequently transferred to the children's hospital with the clinical diagnosis of apnea from
gastroesophageal reflux.Which bias caused this adverse event?*
a) Availability bias
b) Anchoring bias
c) Confirmation bias
d) Premature closure
e) Violation bias
The answer is: Availability bias
76) A 53-year-old man presents to the GP with a deep, painful ulcer over the big toe. He gives a 3-month
history of severe calf pain on walking which is only eased on resting. Examination shows cool
peripheries with reduced distal pulses. Which ulcer does the patient most likely have?*
a) Neuropathic ulcer
b) Curling ulcer
c) Marjolin ulcer
d) Arterial ulcer
e) Venous ulcer
The answer is: Arterial ulcer
77) The mother of a minor consents to medication but the father refuses. What is the most appropriate
action to do?**
a) Report the event to the police
b) Do not write the prescription
c) Postpone the prescription
d) Convince the daughter
e) Write the prescription
The answer is: Write the prescription
78) Treatment of acute bronchiolitis include all of the following except:
a) Humidified oxygen
b) Bronchodilators at least single dose trial
c) Hospitalization if distressed
d) Ribavirin
e) Antibiotics in all patients
The answer is: Antibiotics in all patients
79) A 62 –year- old man presented to the ICU with hypertension and cerebral stroke . Which of the following
is true regarding hypertension and stroke?
a) Emergency medications that directly increase the cerebral blood flow are of choice in all
cerebrovascular accidents

127
EMLE Trial Exams December 2020 (1000 Qs) Version (1.0) 22/12/2020

b) Only patients with intracerebral hemorrhages should have their BP reduced to a target of systolic PB of
140 to 160 mmHg or prestroke level
c) BP should be controlled to prestroke level in all stroke patients within 15 minutes
d) Patients with both ischemic and hemorrhagic strokes should have their systolic BP reduced to 140 to
160 mm Hg or prestroke level .
e) Only patients with ischemic brain infarcts should have their BP reduced to target systolic BP of 140 to
160mmHg or prestroke level
The answer is: Only patients with intracerebral hemorrhages should have their BP reduced to a target of
systolic PB of 140 to 160 mmHg or prestroke level
80) A 60-year-old woman had unexpected slight blood on her underwear. It was bright red without clots and
there was no pain or discomfort. The bleeding recurred twice since in similar fashion. Her last period
was at the age of 48 years with no other intervening bleeding episodes. She had hot flushes and night
sweats around the time of her menopause. She has normal cervical smear 4 months previ- ously. She
had two vaginal deliveries then used intrauterine devices as contraception. She did not use hormone-
replacement therapy. She is slightly overweight. Abdominal examination is normal. The vulva, vagina
and cervix appear normal for her age. The uterus felt small and anteverted on clinical palpation. There
were no palpable adenxal masses. Transvaginal ultrasound scan showed the endo- metrium 12-mm
thickness. What is the NEXT STEP in management?
a) estimation of the tumor markers
b) Estrogenreplacementtherapy
c) Cyclical hormonal therapy
d) endometrial biopsy
e) hysterectomy
The answer is: endometrial biopsy
81) indications for splenectomy include all except:
a) Sequestration crisis
b) All cases of Beta thalassemia major
c) Chronic immune thrombocytopenic purpura
d) Congenital spherocytosis
e) Traumatic splenic injury
The answer is: All cases of Beta thalassemia major
82) The most common single cause of infantile gastroenteritis is:#
a) Rota virus
b) Campylobacter jejune
c) Parvovirus B-19.
d) E coli.
e) Yersinia enterocolitica
The answer is: Rota virus
83) What is not True of Fournier’s gangrene:
a) Testes and scrotum slough away
b) Many patients die despite active treatment
c) Obliterative arteritis causes skin gangrene
d) Can follow minor injuries to perineum
e) Haemolytic streptococci are responsible
The answer is: Testes and scrotum slough away
84) A 64-year-old woman presents with several episodes of profuse dark per rectal bleeding. She has
experienced intermittent symptoms of central abdominal discomfort and bloating for the past 2 years,

128
EMLE Trial Exams December 2020 (1000 Qs) Version (1.0) 22/12/2020

and admits to being prone to constipation. She undergoes a single contrast barium enema, which is
shown below. What is the diagnosis?*
a) Familial adenomatous polyposis coli
b) Adenocarcinoma
c) Crohn’s disease
d) Diverticular disease
e) Peutz–Jeghers syndrome
The answer is: Diverticular disease
85) The 16-year-old daughter consents to medication, but the mother refuses What is the most appropriate
action to do?**
a) Postpone the prescription
b) Report the event to the police
c) Write the prescription
d) Do not write the prescription
e) Convince the daughter
The answer is: Do not write the prescription
86) A 14-year-old schoolboy presents to the Emergency Department after accidentally pouring a cup of hot
coffee over his right hand. He is in obvious discomfort and as part of his on site first aid, his mother ran
his hand under cold running water for 20 minutes. You are called to examine his burns and notice that
most of the dorsum of his right hand is affected. In order to discuss his case with the regional burns
centre you wish to distinguish whether this is a partial thickness or full thickness burn. From the
following list, which features, on examination would suggest a full thickness burn over a partial
thickness injury?*
a) Blistering
b) Painless and not sensitive to light touch
c) Painful and sensitive to light touch
d) Swollen and erythematous
e) Discharging fluid and weeping
The answer is: Painless and not sensitive to light touch
87) A 70-year-old man was admitted with pallor, light-headedness and loss of energy. On the day prior to
admission he had reported loose dark stools. Examination revealed a pulse of 120 per minute and a
blood pressure of 85/55 mmHg. Investigations revealed: Hemoglobin 5.2 g/dL, MCV 72 fL, white cell
count 11.3 x 103/dL, platelet count 480 x 103/dL. Which of the following is the most appropriate first
step in his management?
a) Endoscopy
b) H2 antagonist IV therapy
c) Proton pump inhibitor IV therapy
d) Blood transfusion
e) Selective angiography
The answer is: Blood transfusion
88) You found a child convulsing in front of you ,what is your first response*
a) Try to awake him
b) Do CPR
c) Support in best position to avoid that he might hurt himself
d) None of the above
e) Try to move him
The answer is: Support in best position to avoid that he might hurt himself

129
EMLE Trial Exams December 2020 (1000 Qs) Version (1.0) 22/12/2020

89) A child with history of enuresis has been found besides his medications in coma with fixed dilated pupil
what do you suspect?
a) Opiates
b) Anticholinergic
c) Third nerve lesion
d) Tentorial herniation
e) Barbiturates
The answer is: Anticholinergic
90) A 20-year-old primigrávida, who is 24 weeks pregnant, expresses concern about the normality of her
fetus after learning that a close friend has just delivered an infant with hydrocephalus. Which of the
following details about hydrocephalus should be included in her counseling?
a) is usually an isolated defect
b) can be identified as early as 10 weeks’ gestation
c) can be cured by intrauterine placement of shunts
d) occurs spontaneously in 1 in 500 regnancies
e) has a multifactorial etiology
The answer is: has a multifactorial etiology
91) Common triggers of asthma in children include all of the following except:
a) Cold air
b) Ozone
c) Perfume
d) Gelatin
e) Tobacco smoke
The answer is: Gelatin
92) Feeding difficulties in the neonatal period may be due to all of the following except:
a) Prematurity
b) Physiological jaundice
c) Cerebral birth trauma
d) Congenital heart disease
e) Hiatus hernia
The answer is: Physiological jaundice
93) Prenatal care is a structured approach to obstetric care to assess for increase risk of complications or the
actual development of problems. Which of the following would most predispose the patient to
obstetrical complications?
a) a clinically measured pelvic diagonal conjugate of 12 cm
b) maternal age 17, with menarche at age 13
c) history of ovarian dermoid cyst removed 4 years ago
d) maternal age 39
e) history of four normal deliveries
The answer is: maternal age 39
94) If both parents have type 2 DM, risk of developing diabetes is ?
a) ~ 30 %
b) ~ 50%
c) ~ 40 %
d) ~ 10 %
e) ~ 20 %
The answer is: ~ 40 %

130
EMLE Trial Exams December 2020 (1000 Qs) Version (1.0) 22/12/2020

95) Reticulocytosis is present in all of the following except:


a) Thalassemia
b) Bleeding
c) Iron therapy in iron deficiency anemia
d) Chronic hemolytic anemia
e) Hypoplastic anemia
The answer is: Hypoplastic anemia
96) Leaders’ walk rounds in an effective opportunity to:#
a) Focus front-line staff on safety issues
b) Identify wrong doers
c) Inspect the different departments in an informal way
d) Discuss issues of concern to staff members
e) All of the above
The answer is: Focus front-line staff on safety issues
97) After a head trauma what is the urgent investigation to ask ( if needed )*
a) U/S
b) CT
c) MRI
d) Angiography
e) CBC
The answer is: CT
98) Which of the following statements about the process of wound healing are true?
a) The inflammatory phase begins 2–3 days after the injury.
b) Non of the above
c) The proliferative phase lasts from 3 days to 3 weeks following the injury.
d) The remodeling phase involves fibroblast activity and production of collagen and ground substance.
e) The white cells stick to the damaged endothelium and release adenosine diphosphate (ADP) and
cytokines.
The answer is: The proliferative phase lasts from 3 days to 3 weeks following the injury.
99) A 49-year-old man presents with gastric pain and is found to have a 6-cm mass located in the anterior
wall of his stomach. This mass is resected and histologic examination reveals a tumor composed of cells
having elongated, spindle-shaped nuclei. The tumor does not connect to the overlying gastric epithelium
and is instead found only in the wall of the stomach. The tumor cells stain positively with CD117, but
negatively with both desmin and S-100. Special studies find that these tumor cells have abnormalities of
the KIT gene. Which of the following is the most likely diagnosis?
a) Ectopic islet cell adenoma (VIPoma)
b) Submucosal leiomyoma “fibroid tumor”
c) Gastrointestinal stromal tumor (GIST)
d) Lymphoma of mucosa-associated lymphoid tissue (MALToma)
e) Nonchromaffin paraganglioma (chemodectoma)
The answer is: Gastrointestinal stromal tumor (GIST)
100) Which IS NOT suitable for treatment for mastalgia?
a) Bromocriptine
b) Surgery
c) Gammalenolenic acid (GLA)
d) Antiprostaglandins
e) Danazol

131
EMLE Trial Exams December 2020 (1000 Qs) Version (1.0) 22/12/2020

The answer is: Surgery

Exam 8
1) Symptoms of Anaphylaxis is manifested by the following except:
a) Inspiratory stridor
b) Emphysema
c) Hypertension
d) Hematemesis
e) Melena
The answer is: Inspiratory stridor
2) What of the following causes microcytic anemia?*
a) Thalassemia
b) Sickle cell anemia
c) Vitamin B12 deficiency anemia
d) G6PD deficiency
e) Autoimmune hemolytic anemia
The answer is: Thalassemia
3) A 22-year-old G0 P0 woman complains of irregular menses every 30 to 65 days. The semen analysis is
normal. The hysterosalpingogram is normal. Which of the following is the most likely treatment for this
patient?
a) Laparoscopy.
b) Intrauterine insemination.
c) In vitro fertilization.
d) Clomiphene citrate.
e) none of the above
The answer is: Clomiphene citrate.
4) An 18-year-old patient presents to you for evaluation because she has not yet started her period. On
physical examination, she is 170 cm in tall. She has minimal breast development and no axillary or pubic
hair. On pelvic examination, she has a normally developed vagina. A cervix is visible. The uterus is
palpable, as are normal ovaries. Which of the following is the BEST NEXT STEP in the evaluation of this
patient?
a) Draw her blood for a karyotype
b) Test her sense of smell.
c) Draw her blood for TSH, FSH, and LH levels
d) Order an MRI of the brain to evaluate the pituitary gland
e) Prescribe a progesterone challenge to see if she will have a withdrawal bleed
The answer is: Test her sense of smell.
5) Which of the following statements regarding Boerhaave’s syndrome is false?*
a) Endoscopic evaluation of the esophageal tear is an integral part of diagnosis
b) The full‐thickness esophageal tear is typically located in the left posterolateral aspect of the
loweresophagus approximately 2–3 cm above the gastroesophageal junction
c) Mackler’s triad of vomiting, lower chest pain, and subcutaneous emphysema is seen in up to two thirds
of patients with Boerhaave’s syndrome
d) The underlying pathophysiology involves uncoordinated esophageal contraction against a closed
pylorus distally and cricopharyngeus muscle proximally
e) Upper GI bleeding frequently accompanies Boerhaave’s syndrome

132
EMLE Trial Exams December 2020 (1000 Qs) Version (1.0) 22/12/2020

The answer is: Endoscopic evaluation of the esophageal tear is an integral part of diagnosis
6) A 39-year-old motorcyclist presents following a road traffic collision. He arrives at the Emergency
Department with a pelvic splint in place. He is assessed with a primary survey, which discovers a likely
pelvic fracture. He is tachycardic at 115 beats/min despite giving an immediate 2 L of Hartmann’s
solution. He is hypotensive on reassessment. What is the most appropriate management option?#
a) Internal iliac artery emobilization
b) Laparotomy
c) CT head
d) Chest drain
e) CT neck
The answer is: Internal iliac artery emobilization
7) Which of the following anesthetic technique will produce the greatest uterine relaxation?
a) Spinal block.
b) Caudal.
c) Nitrous oxide.
d) Halothane.
e) Paracervical
The answer is: Halothane
8) Advance directive is designed to protect which of the following patient’s rights?
a) Autonomy
b) Beneficence
c) Non-maleficence
d) Justice
e) Maleficence
The answer is: Autonomy
9) A case of pregestational DM is usually controlled during pregnancy by:#
a) Diet only.
b) Diet and oral hypoglycaemics.
c) Oral hypoglycaemics only.
d) Diet and insulin therapy.
e) Stoppage of treatment once pregnancy occurs
The answer is: Diet and insulin therapy.
10) A 13- year-old girl is referred to the paediatric assessment unit with a widespread macular rash. She has
recently seen her GP with a flu like illness and sore throat for which she was prescriped Amoxicillin with
a diagnosis of tonsillitis. On examination, she has widespread cervical lymphadenopathy and petechiae
on her palate
a) Coronavirus
b) Epstenin Barr Virus
c) Group A Streptococcus
d) Parainfluenza
e) Rhino virus
The answer is: Epstenin Barr Virus
11) Which of the following is not associated with increased likelihood of infection after major elective
surgery?*
a) Age over 70 years.
b) Chronic malnutrition.
c) Controlled diabetes mellitus.

133
EMLE Trial Exams December 2020 (1000 Qs) Version (1.0) 22/12/2020

d) Long-term steroid use.


e) Infection at a remote body site
The answer is: Controlled diabetes mellitus.
12) A 34-year-old woman is brought to the ED from a kitchen fire. Upon arrival, her BP is 110/75 mmHg, HR
is 115 b/m, RR is 18 b/m, and pulse oxygenation is 98% on room air. She weighs 70 kg. She has tender
burns with blisters to her bilateral forearms. Using the Parkland Formula, approximately how much IV
fluid should this patient receive in the first 4 hours following her injury?
a) 500mL
b) 1000mL
c) 2000mL
d) 2500mL
e) 3000mL
The answer is: 2500mL
13) A friend mentions to you she just had a positive pregnancy test and wonders if you can tell her when
she is likely due. The LMP was June 30. Her expected date of confinement (EDC) is approximately which
of the following?
a) 43913
b) April 7
c) 43918
d) 43944
e) 43897
The answer is: April 7
14) You are seeing patients in clinic when two men come in and show' you badges marking them as
members of a police department. The identification is legitimate. They inform you that they are making
a “minor investigation” of one of your patients. They ask to look at the patient’s chart for a few minutes,
saying, “You wouldn’t want to interfere with a police investigation, would you?” What should you do?
a) Ask them to sign a release for the chart so you are absolved of responsibility.
b) Don’t give them the chart but read the relevant information to them.
c) Give them the chart but watch what they do with it.
d) Tell them that you can give copies but not the original record.
e) Tell them you cannot show them the chart unless there is a signed release from the patient.
The answer is : Tell them you cannot show them the chart unless there is a signed release from the patient
15) You review a 48-year-old woman who is taking methotrexate for rheumatoid arthritis. Concurrent
prescription of which other medication should be avoided?
a) Trimethoprim
b) Sumatriptan
c) Lansoprazole
d) Sodium valproate
e) Erythromycin
The answer is: Trimethoprim
16) A 53-year – old male with a history of alcohol abuse presents to the emergency department for
evaluation of diaphoresis, repeated none bloody vomiting and abdominal pain. His last drink was 24
hours ago. Temperature 37.3C , pulse 113bpm , RR 18breath per minute , BP 120/70mmHg , SaO2 97% ,
HCO3 13 mEq/L , glucose 86 mg/dL , Urine ketones very high ( four plus ), Na 129 mEq/L , K 3.8mEq/L ,
normal serum creatinine and unremarkable abdominal examination . Which is the best next step ? *
a) IV thiamine , bolus of 5% dextrose in normal saline
b) IV insulin infusion , IV thiamine , IV infusion of 5 % dextrose in normal saline

134
EMLE Trial Exams December 2020 (1000 Qs) Version (1.0) 22/12/2020

c) IV insulin infusion , IV 5% dextrose infusion with 40 mEq Potassium chloride


d) IV insulin infusion , IV 5% dextrose infusion with 20 mEq sodium bicarbonate
e) Bolus of normal saline over one hour plus antiemetic
The answer is: IV thiamine, bolus of 5% dextrose in normal saline
17) About Allergic rhinitis, which is true?
a) not related to seasonal variation
b) Never affects toddlers
c) Commonly associated with mouth breathing
d) affects males only
e) affects females only
The answer is: Commonly associated with mouth breathing
18) Arterial embolism may result from:
a) Atherosclerosis
b) Bone fracture
c) Parasites
d) All of the above
e) none of the above
The answer is: All of the above
19) Glucose in Oral rehydration solution is added to#
a) Improve the taste of Oral rehydration
b) Meet the child’s energy requirements
c) Increase osmolality of ors to 400 mOsmol
d) Promote the absorption of sodium
e) Prevent malnutrition
f) May be treated with Co-Trimoxazole
The answer is: Promote the absorption of sodium
20) A false statement about acute post-streptococcal glomerulonephritis is:
a) There is oliguria
b) Hypertension is rare
c) Congestive heart failure may occur
d) Renal function may be impaired
e) RBC Casts
The answer is: Hypertension is rare
21) A 47-year-old female without any past medical history presents with several days of progressively
worsening left eye pain, blurry vision, and redness. Visual acuity in the left eye is slightly reduced. The
patient notes that exposure of the right eye to light causes increased pain in her left eye. Her left pupil is
constricted and minimally reactive to light with perilimbic conjunctival injection. There is no discharge.
Which of the following is the most appropriate treatment?
a) Topical antibiotics
b) Hypertonic eye drops
c) IV mannitol
d) Ocular massage
e) Topical cycloplegic mydriatics
The answer is: Topical cycloplegic mydriatics
22) The lateral boundary of the callot triangle is#
a) Common bile duct
b) Common hepatic duct

135
EMLE Trial Exams December 2020 (1000 Qs) Version (1.0) 22/12/2020

c) Cystic duct
d) Left hepatic duct
e) Right hepatic duct
The answer is: Cystic duct
23) A 30 years old pregnant woman presented to the antenatal clinic with normal GTT, however she had a
4.5 kg neonatal weight in her last pregnancy. This patient is considered:
a) Latent diabetic.
b) Potential diabetic.
c) Chemicaldiabetic.
d) Overt diabetic.
e) none of the above
The answer is: Potential diabetic.
24) Immediate therapy for infants with suspected meconium should routinely include:
a) Corticosteroid
b) Antibiotics
c) Sodium bicarbonate
d) Clearing of the airway
e) Giving O2 under positive pressure
The answer is: Clearing of the airway
25) A 5-year-old male is brought for evaluation of penile erythema. He is uncircumcised and has erythema,
edema, and a semisolid discharge around the glans without accompanying phimosis or paraphimosis. He
has had several similar episodes in the past with identification of yeast. Which of the following is the
most appropriate diagnostic test?
a) Liver function tests
b) Postvoid residual
c) Intravenous pyelogram
d) Serum glucose
e) Retrograde urethrogram
The answer is: Serum glucose
26) Specific immunotherapy is resorted to in cases of:
a) Pollen allergy
b) Fish allergy
c) Cow milk allergy
d) Strawberry allergy
e) banana allergy
The answer is: Pollen allergy
27) A 55-year-old woman, who recently had been dieting with a weight loss of 20 lb, presents with a small-
bowel obstruction and pain, which radiates down the inside of her thigh to the knee. She has no past
history of abdominal surgery. Which of the following is the likely diagnosis?**
a) Strangulated obturator hernia
b) Obstructing neoplasm of the ileum
c) Gallstone ileus
d) Strangulated femoral hernia
e) Fracture of the pubic bone
The answer is: Strangulated obturator hernia
28) The most common cause of perinatal death in mono-amniotic twin is:
a) Cord entrapment.

136
EMLE Trial Exams December 2020 (1000 Qs) Version (1.0) 22/12/2020

b) Cord prolapse.
c) Twin-twin transfusion syndrome.
d) Lethal congenital anomalies.
e) Placental abruption
The answer is: Cord entrapment.
29) Patient presented with acute abdomenal pain, leucocytosis, BP 90/60, bl. Sugar 55mg/dl, Na:
119mEq/dl, K: 6.3mEq/dl. What is the definitive treatment?
a) Hypertonic saline
b) %10dextrose infusion
c) Laparotomy
d) Corticosteroids
e) Vasopressors
The answer is: Corticosteroids
30) Fetal RBCs can be distinguished from maternal RBCs by their:
a) Shape.
b) Resistance to acid elution.
c) Lack of Rh factor.
d) Lower amount of hemoglobin.
e) All of the above
The answer is: Resistance to acid elution.
31) In severe hyponatremic dehydration, deficit fluid therapy should be given over :
a) 4hours
b) 8hours
c) 16hours
d) 24hours
e) 48hours
The answer is: 24 hours
32) A 24-year-old patient now 17 weeks’ pregnant is found to have a positive VDRL (Venereal Disease
Research Laboratory) of 1:16 titer. She gives no history of syphilis. A fluorescent treponemal antibody
test (FTA) is drawn but will require 1 to 2 weeks to be returned. Cerebrospinal fluid (CSF) tests are
negative. The patient denies allergies.Which of the following is the best choice of therapy for this
patient?
a) phenothiazines
b) hypnosis
c) IV hydration
d) psychiatric referral
e) outpatient antiemetic therapy
The answer is: IV hydration
33) A 76-year-old hypertensive smoker with a 4-month history of backache presents with acute excruciating
epigastric pain radiating to the back. O/E: blood pressure is 85/50mmHg, pulse is 130bpm (right femoral
pulse is absent, while left femoral pulse is weak), marked abdominal tenderness and rigidity.what is the
cause of his abdominal pain?*
a) Acute appendicitis
b) Acute pancreatitis
c) Ascending cholangitis
d) Acute MI
e) Ruptured abdominal aortic aneurysm

137
EMLE Trial Exams December 2020 (1000 Qs) Version (1.0) 22/12/2020

The answer is: Ruptured abdominal aortic aneurysm


34) Child presented to you with acetone breath, disoriented and looks dehydrated what is your first line of
treatment (his glucose level is giving Error in the strips) *
a) Give fluids
b) Give packed RBC
c) Give plasma
d) Give glucose solution
e) Ask for CBC
The answer is: Give fluids
35) A 30-year-old woman is referred to the on-call surgical team with a 1-week history of right iliac fossa
pain, anorexia and fever. On examination, she has a palpable tender mass is in the right iliac fossa. Her
observations are: temperature 38.6°C, heart rate 110 beats/min, BP 125/70 mmHg. Which of the
following is the most likely diagnosis?*
a) Appendiceal mass
b) Carcinoma of caecum
c) Diverticular abscess
d) Ovarian cyst
e) Iliac artery aneurysm
The answer is: Appendiceal mass
36) What is the target level of Hb in patients with CKD?*
a) 10G /dl
b) 8g/dl
c) 12.5g/dl
d) 13g/dl
e) 14g /dl
The answer is: 10 G /dl
37) Which of the following is a cause of secondary (or exogenous) infection?*
a) Poor hand-washing technique
b) Hospital acquired infection
c) Perforated diverticular disease
d) Anastomotic leak
e) Inadequate air filtration in the theatre
The answer is: Perforated diverticular disease
38) Disease of urinary bladder course:
a) Initial hematuria.
b) Terminal hematuria.
c) Total hematuria.
d) Con not cause hematuria.
e) all of the above
The answer is: Terminal hematuria.
39) Epinephrine used in treatment of severe urticaria is?
a) Of 1:1 concentration
b) Given in doses of 0.25 ml/kg body weight
c) Given only in ICU
d) can be given by any dose
e) can be given IM
The answer is: can be given IM

138
EMLE Trial Exams December 2020 (1000 Qs) Version (1.0) 22/12/2020

40) An African American couple presents for prenatal care. They are interested in complete screening as
indicated for genetic diseases. They are unaware of their sickle cell status regarding the presence of trait
or not.Since sulfa drugs are commonly prescribed for urinary tract infections (UTIs) during pregnancy,
you include as part of counseling that G6PD homozygous deficiency is present in what percentage of
African American women?
a) less than 1
b) 2
c) 5
d) 10
e) 33
The answer is: 2
41) A 19-year-old student is brought to the Emergency Department by friends due to a severe headache and
drowsiness. On examination he has a widespread purpuric rash. Meningococcal infection is strongly
suspected. What is the antibiotic of choice?#
a) Iv meropenem
b) Iv benzyl penicillin
c) Iv gentamycin
d) Iv amikacin
e) Iv ciprofloxacin
The answer is: IV benzyl penicillin
42) A 12-year-old presents with sneezing clear rhinorrhea and nasal itching. Physical examination reveals
boggy, pale nasal edema with a clear discharge. The most likely diagnosis is:
a) Foreign body
b) Vasomotor rhinitis
c) Neutrophilic rhinitis
d) Nasal mastocytosis
e) Allergic rhinitis
The answer is: Allergic rhinitis
43) A 43-year-old woman presents with increasing “heartburn,” especially after eating. Endoscopic
examination finds a red velvety plaque located at the distal oesophagus. Biopsies from this area, taken
approximately 3 cm proximal to the gastroesophageal junction reveal metaplastic columnar epithelium.
Which of the following is the most likely diagnosis?
a) Acquired achalasia
b) Barrett’s esophagus
c) Hamartomatous polyp
d) Metastatic adenocarcinoma
e) Reflux esophagitis
The answer is: Barrett’s esophagus
44) Jaundice appearing on the first 24 hours of life maybe diagnosed as :
a) Physiological jaundice
b) Thalassemia
c) Rh incompatibility
d) All of the above
e) none of the above
The answer is: Rh incompatibility
45) A 50-year-old woman underwent wide excision of a 2.5-cm infiltrating ductal carcinoma of the breast
with axillary lymph node dissection followed by radiation and chemotherapy 2 years ago. The patient

139
EMLE Trial Exams December 2020 (1000 Qs) Version (1.0) 22/12/2020

now complains of RUQ abdominal pain. ACAT scan reveals two masses in the right lobe of the liver.
Select the most likely diagnosis.*
a) Adenoma
b) Focal nodular hyperplasia
c) Haemangioma
d) Hepatocellular carcinoma
e) Metastatic carcinoma
The answer is: Metastatic carcinoma
46) How many breaths and compressions you do in CPR ( compressions- breath)*
a) 2-5
b) 5-30
c) 2-30
d) 15-30
e) 30-30
The answer is: 2:30
47) Severely distressed neonate is detected at 1 minute by an Apgar score of
a) 10-8
b) 7-4
c) 3-0
d) 8-5
e) 9-7
The answer is:3-0
48) Which of the following regarding blood pressure in shock are false?
a) Elderly patients who are normally hypertensive may present with a ‘normal’ blood pressure.
b) Children and fit young adults are able to maintain blood pressure until the final stages of shock.
c) Hypotension is one of the first signs of shock.
d) Beta-blockers may prevent a tachycardic response.
e) Blood pressure may be initially increased by increase in stroke volume and peripheral vasoconstriction
The answer is: Hypotension is one of the first signs of shock.
49) What is the target level of Hb in patients with CKD?*
a) 10G /dl
b) 8g/dl
c) 12.5g/dl
d) 13g/dl
e) 14g /dl
The answer is: 10 G /dl
50) A 60-year-old woman presents with an ulcer on her shin. The ulcer developed two weeks ago and is
enlarging and getting deeper. There is mild surrounding erythema but she does not feel systemically
unwell. She has Type 2 diabetes and has a BMI of 32.8kg/m2. Her HbA1c is 110 mmol/mol.On
examination, she is afebrile. The wound is exuding a bluish-green fluid with a fruity odour. A swab taken
from the ulcer grows Gram-negative bacilli.Which of the following is the most effective antibiotic for this
case?
a) Flucloxacillin
b) Metronidazole
c) Vancomycin
d) Ceftazidime
e) Trimethoprim

140
EMLE Trial Exams December 2020 (1000 Qs) Version (1.0) 22/12/2020

The answer is: Ceftazidime


51) A 73-year-old man presents with several episodes of hematemesis. Examination shows signs of
orthostatic hypotension and melena. Which of the following is the first priority in caring for this patient?
a) Intravenous infusion of H2-receptor antagonists to stop the bleeding
b) Nasogastric tube placement and gastric lavage.
c) Resuscitation with adequate IV access and appropriate fluid and blood transfusion.
d) Urgent surgical consultation.
e) Urgent upper endoscopy
The answer is: Resuscitation with adequate IV access and appropriate fluid and blood transfusion.
52) A newborn is noted to have a darkened swelling of the scalp that does not cross the midline. This is
most likely a:
a) Caput succedaneum.
b) Subdural hemorrhage.
c) Cephalhematoma.
d) Subarachnoid hemorrhage.
e) Tentorial tear
The answer is: Cephalhematoma.
53) An African American couple presents for prenatal care. They are interested in complete screening as
indicated for genetic diseases. They are unaware of their sickle cell status regarding the presence of trait
or not.What is the percentage of African Americans with sickle cell anemia disease?
a) less than 1
b) 5
c) 10
d) 25
e) 50
The answer is: less than 1
54) A 35 –year –old female presents to the ER with palpitation, chest pain and lightheadedness. Her BP
70/40mmHg, pulse regular, 175bpm, ECG shows narrow complex atrioventricular nodal re-entrant
tachycardia (AVRNT) .Which of the following next step in management?
a) A denosine 6mg IV
b) Diltiazem 20 mg IV
c) Amiodarone 300mg IV
d) Synchronized cardioversion at 50 J
e) Defibrillation at 200 J
The answer is: Synchronized cardioversion at 50 J
55) In patients with three consecutive spontaneous abortion in the second trimester the most useful
investigation is:
a) Chromosomal analysis
b) Hysterosalpingogram
c) Endometrial biopsy
d) Post coital test
e) Prolactin level
The answer is: Hysterosalpingogram
56) A 50-year-old Asian man is referred to the diabetes clinic after presenting with polyuria and polydipsia.
He has a BMI of 30, a blood pressure measurement of 137/88 and a fasting plasma glucose of 7.7
mmol/L. The most appropriate first-line treatment is:
a) Dietary advice and exercise

141
EMLE Trial Exams December 2020 (1000 Qs) Version (1.0) 22/12/2020

b) Sulphonylurea
c) Exenatide
d) Thiazolidinediones
e) Metformin
The answer is: Dietary advice and exercise
57) In hemorrahgic shock perepheral vascular resistance:
a) Increase.
b) Decrease.
c) Remain constant.
d) Any of the above
e) none of the above
The answer is: increase.
58) Investigators studying cardiovascular disease discover a new serum protein marker that is correlated
with the presence of ruptured atherosclerotic plaques. It is hoped that this serum marker could be used
as a screening test to identify whether a person has had a recent myocardial infarction (MI). In a case-
control study of 1400 subjects, the investigators find that of the 500 subjects who had an MI, 400 tested
positive for the serum marker, whereas 850 subjects who did not have an MI tested negative for the
marker. If this marker were used to screen patients for recent MI, what is the probability that a person
will have had an MI given a positive serum protein analysis?#
a) )50 + 400( / 400
b) )100+ 400( / 400
c) )900 + 500( / 400
d) )850 + 50( / 850
e) )850 + 100( / 850
The answer is:)50 + 400( / 400
59) ‘ Sign’ that refers to a spontaneous cessation of inspiration on palpation of the hypochondrium just
below the right costal margin. What is the eponymous sign or law being described?*
a) Courvoisier’s
b) Rovsing’s
c) Murphy’s
d) Grey Turner’s
e) Battle’s
The answer is: Murphy’s
60) Which of the following statements about bacterial endocarditis is false?#
a) A congenital heart disease is a predisposing factor
b) Children usually present acutely with high fever
c) The spleen may be enlarged and tender
d) Clubbing of fingers develops after 5-6 weeks
e) Treatment requires 4-6 weeks of intravenous antibiotic therapy
The answer is: Children usually present acutely with high fever
61) Which of the following quality measures is used for calculating Percentage of diabetic patients whose
HbAlc was measured in the past 6 months**
a) Structural
b) Process
c) Outcome
d) balancing
e) Non-specific

142
EMLE Trial Exams December 2020 (1000 Qs) Version (1.0) 22/12/2020

The answer is: Process


62) A 40-year-old male with no past medical history presents to the outpatient clinic for routine visit. His
Hemoglobin A1C is 6.3%. What is the most likely diagnosis?
a) Type 1 Diabetes
b) Type 2 Diabetes
c) Pre-Diabetes
d) Monogenic diabetes
e) Normal
The answer is: Pre-Diabetes
63) A 31 year old women presents to you with recurrent abdominal pain and frequent bloody diarrhea. You
suspect inflammatory bowel disease and arrange for a colonoscopy with biopsy. Which one of the
following features on biopsy would suggest ulcerative colitis over a diagnosis of crohn's disease?
a) Skip lesions
b) Rosethorn ulceration
c) Presence of granulomas
d) Transmural (full thickness) involvement
e) Presence of Crypt abscess
The answer is: Presence of Crypt abscess
64) You entered a room and found a child on the ground not responding First thing to do is?*
a) Try to move him
b) Call for help
c) Freak out
d) Find a certified doctor in emergency
e) None of the above
The answer is: Call for help
65) A 33-year-old woman has a Pap smear showing moderately severe cervical dysplasia (high-grade
squamous intraepithelial neoplasia). Which of the following is the best next step?
a) Repeat Pap smear in 3 months.
b) Conization of the cervix.
c) Colposcopic directed biopsies.
d) Radical hysterectomy.
e) CT scan of the abdominal and pelvis
The answer is: Colposcopic directed biopsies
66) A 70-year-old ex-smoker presents to his GP with a 3-month history of weight loss and worsening
dysphagia, where he is now unable to swallow soft foods and liquids. He is also experiencing nausea and
vomiting on several occasions. What is the most likely cause of his vomiting?*
a) Gastric carcinoma .
b) Oesophageal carcinoma
c) Opiate analgesia
d) Pancreatitis
e) Gastritis
The answer is: Oesophageal carcinoma
67) Child 7 years presents with a complain that after head trauma he had amnesia for 2 min after you have
examined him you found that he is good and active what is you recommendation?#
a) Reassure and send home
b) CT
c) MRI

143
EMLE Trial Exams December 2020 (1000 Qs) Version (1.0) 22/12/2020

d) US
e) X ray
The answer is: Reassure and send home
68) A 45-year – patient with pneumonia admitted to hospital because of parapneumonic pleural effusion,
which of the following is a reasonable indication to perform tube thoracostomy ?
a) Pleural fluid : serum LDH ratio <o.6
b) Serum – Pleural fluid albumin gradient >1.1
c) Pleural fluid PH < 7.0
d) Pleural fluid glucose > 100mg/dL
e) Fever > 38.5 C after first dose of appropriate antibiotic
The answer is: Pleural fluid PH < 7.0
69) All are characteristics of G6PD deficiency except:#
a) Sex linked recessive inheritance
b) Persistence of low-grade anemia in patients
c) Increased reticulocyte count during attack
d) Heinz bodies are present in hemolytic episodes
e) Diagnosed by enzyme titer between attacks
The answer is: Persistence of low-grade anemia in patients
70) A butcher comes into emergency department after accidently stabbing his groin with knife. He tried to
use a towel to stop the bleeding but has bled so much that the towel is now soaked with blood. His
blood pressure is 75/40mmHg, and pulse is a 120/min. What is the single most appropriate initial
management?
a) Blood sample for crossmatch and blood transfusion
b) IV fluids
c) Fresh frozen plasma
d) Refer to surgeon
e) Emergency Doppler Ultrasound
The answer is: IV fluids
71) Which of the following medications that he takes may be associated with increased risk of deep vein
thrombosis?*
a) Fenofibrates
b) Atorvastatin
c) Ramipril
d) Bisoprolo
e) Indapamide
The answer is: Fenofibrates
72) Etiological factors in spontaneous abortion include:
a) Chromosomal abnormalities
b) Placental abnormalities
c) Maternal disease
d) Uterine abnormalities
e) All of the above
The answer is: All of the above
73) What is the best diagnosis of hard non tender lymphadenopathy?*
a) Lymph node biopsy
b) CT chest, abdomen and pelvis
c) MRI abdomen and pelvis

144
EMLE Trial Exams December 2020 (1000 Qs) Version (1.0) 22/12/2020

d) Colonoscopy
e) UGI endoscopy
The answer is: Lymph node biopsy
74) Disinfection is a process in addition to cleaning. When is it necessary?*
a) Autonomy
b) Beneficence
c) Non-maleficence
d) Justice
e) Maleficence
The answer is: Non-maleficence
75) Which sign of following is most alarming for acute aortic dissection in a 51 years old male presenting
with acute chest pain?
a) Hypotension
b) Pallor
c) Pulmonary venous congestion
d) Unequal pulse bilaterally
e) Tachycardia
The answer is: Unequal pulse bilaterally
76) If you found a child in coma with dilated fixed pupil what will you suspect?
a) Opiates
b) Barbiturates
c) Hypothermia
d) Third nerve lesion
e) Tentorial herniation
The answer is: Hypothermia
77) RH incompatibility occurs with:
a) Rh –ve father & Rh+ve mother
b) Rh –ve mother & Rh –ve father
c) Rh –ve mother & Rh +ve father
d) Rh +ve mother & Rh +ve father
e) none of the above
The answer is: Rh –ve mother & Rh +ve father
78) The effect of pregnancy on DM may be all the following EXCEPT
a) DM may appear for the first time.
b) DM may improve with pregnancy.
c) DM may become difficult to control
d) There is liability to hypoglycaemia
e) None of the above
The answer is: DM may improve with pregnancy.
79) Preterm labor is defined as labor which starts:
a) Before 24 weeks of gestation.
b) Before 37 completed weeks.
c) Before the viability of the fetus.
d) When the fetus weighing < 1000 gm.
e) Prior to 40 weeks gestation
The answer is: Before 37 completed weeks
80) The following ultrasonic measurements may be used to confirm or establish gestational age :

145
EMLE Trial Exams December 2020 (1000 Qs) Version (1.0) 22/12/2020

a) Crown rump length


b) Nuchal pad thickening
c) Amniotic fluid volume
d) Yolk sac volume
e) Biophysical profile
The answer is: Crown rump length
81) A 6-year old is admitted to the ward for investigation of glomerulonephritis,as he has peripheral
oedema and blood in his urine , he had been treated with antibiotics for tonsilites three weeks prior .
Heis otherwise well. With no rashes, joint pain or abdominal sympotoms. What is the most likely cause
of his glomerulonephtotis? select one answer only
a) Post-streptococcal glomerulonephritis (PSGN)
b) Berger's disease (IgA)
c) Haemoltic uraemic syndrom (HUS)
d) Henoch-Schohelein purpura (HUP)
e) Goodpasture syndrome
The answer is: Post-streptococcal glomerulonephritis (PSGN)
82) Which of the following ischaemic stroke patients who have undergone carotid duplex scanning would
be most likely to benefit from left carotid endarterectomy?
a) Right middle cerebral artery territory infarct with good functional recovery, 45% left carotid artery
stenosis
b) Asymptomatic patient with 80% left carotid artery stenosis
c) Left middle cerebral artery territory infarct with good functional recovery, 80% left carotid artery
stenosis
d) Left middle cerebral artery territory infarct with persistent dense right hemiparesis and profound
dysphasia, 90% left carotid artery stenosis
e) Left cerebellar hemisphere infarct with good functional recovery, 55% left carotid artery stenosis
The answer is: Left middle cerebral artery territory infarct with good functional recovery, 80% left carotid
artery stenosis
83) A 22- year-old previously healthy female is admitted to ED with severe dyspnea, palpitation and
extreme fatigue. The patient had been diagnosed with viral infection 7 days before .Upon arrival she is
pale and ill. Pulse 143 bpm, RR 28 breathe per minute, BP 105/70mmHg, SaO2 90%. X-ray showed fluffy
bilateral infiltrates, lung examination revealed tachypnea and bilateral fine basal crepitation without
wheeze .Which of the following is the most likely diagnosis?*
a) Supraventricular tachycardia
b) Acute coronary syndrome
c) Acute myocarditis and acute heart failure
d) Infective endocarditis
e) Previously undiagnosed congenital heart disease
The answer is: Acute myocarditis and acute heart failure
84) An 8-month-old infant should be fed as follows:
a) Pure breastfeeding
b) Breastfeeding supplemented by various semisolids
c) Stop breastfeeding and receive only semisolids
d) Start cow’s milk as the only food
e) Artificial formula only
The answer is: Breastfeeding supplemented by various semisolids

146
EMLE Trial Exams December 2020 (1000 Qs) Version (1.0) 22/12/2020

85) A 40-year-old male with no past medical history presents to the outpatient clinic for routine visit. His
Hemoglobin A1C is 7%. What is the most likely diagnosis?
a) Type 1 Diabetes
b) Type 2 Diabetes
c) Pre-Diabetes
d) Monogenic diabetes
e) Normal
The answer is: Type 2 Diabetes
86) You are seeing a 28-year-old woman (gravida 3, para 2) with suspected UTI. To obtain a urine specimen,
which of the following should you order?#
a) clean-void midstream urine
b) catheterization
c) suprapubic tap
d) 24 hour urine
e) first morning void
The answer is: clean-void midstream urine
87) Which indicator in an infant with hypoxic ischemic encephalopathy (HIE) is associated with the worst
prognosis ?
a) Mild hypotonia
b) Hyperalert state
c) Hyperactive reflexes
d) Decerebrate posturing
e) One seizure attack
The answer is: Decerebrate posturing
88) A 16-year-old patient (gravida 1) is seen for the first time when 16 weeks pregnant. History and
examination are entirely normal except for a large solid mass in the posterior pelvis. It is slightly
lobulated, immobile, and smooth and cannot be completely palpated. There is some question as to
whether or not it will obstruct labor.Which of the following procedures should be carried out?*
a) one shot” intravenous pyelogram (IVP(
b) barium enema
c) exploratory laparotomy
d) abortion
e) pelvic/abdominal ultrasound
The answer is: pelvic/abdominal ultrasound
89) A 36-year-old man is involved in a high road traffic accident. On arrival at the resuscitation room he
complains of chest pain and his repiratory rate is 33/min and pulse 105 bpm. He has an area of
paradoxical movement of the chest wall. Which of the following would be the most appropriate
management measure?
a) Analgesia and respiratory support
b) Chest drain insertion
c) Needle thoracocentesis
d) Pericardiocentesis
e) Thoracotomy
The answer is: Analgesia and respiratory support
90) In Rhesus Iso-immunization, the following test may be helpful:
a) Rhesus antibody titer in liquor
b) Maternal serum bilirubin level

147
EMLE Trial Exams December 2020 (1000 Qs) Version (1.0) 22/12/2020

c) Liquor bilirubin level


d) Maternal hemoglobin
e) Baby gender
The answer is: Liquor bilirubin level
91) Predisposing risk factors for gastroenteritis include:
a) Exclusive breast feeding
b) Well-nourished infant
c) None vaccinated by pertussis
d) Lack of hand washing
e) Passive smoking
The answer is: Lack of hand washing
92) A woman with Class II cardiac disease is pregnant. She has no other health problems. What should be
included in the management plans for her pregnancy?
a) A 2200 cal ADA diet to obtain adequate weight gain and prophylaxis against gestational diabetes
b) limit weight gain to 20–25 lb
c) prophylactic low-dose aspirin (81 mg)
d) vaginal delivery unless obstetric indications for C-section
e) deliver at the verification of lung maturity around 37 weeks
The answer is: vaginal delivery unless obstetric indications for C-section
93) What are the typical constituents of Hartmann's solution? Please select from the list below the most
appropriate option.
a) Sodium 30 mmol/L, chloride 30 mmol/L, glucose 40 g/L
b) Glucose 50 g/L
c) Sodium 131 mmol/L, chloride 111 mmol/L, potassium 5 mmol/L, Lactate 29 mmol/L, calcium 2 mmol/L
d) Sodium 154 mmol/L, chloride 154 mmol/L
e) Sodium 154 mmol/L, chloride 154 mmol/L, potassium 40 mmol/L
The answer is: Sodium 131 mmol/L, chloride 111 mmol/L, potassium 5 mmol/L, Lactate 29 mmol/L, calcium 2
mmol/L
94) A 50-year-old woman underwent wide excision of a 2.5-cm infiltrating ductal carcinoma of the breast
with axillary lymph node dissection followed by radiation and chemotherapy 2 years ago. The patient
now complains of RUQ abdominal pain. ACAT scan reveals two masses in the right lobe of the liver.
Select the most likely diagnosis.*
a) Adenoma
b) Focal nodular hyperplasia
c) Haemangioma
d) Hepatocellular carcinoma
e) Metastatic carcinoma
The answer is: Metastatic carcinoma
95) Goitre A 16-year-old girl presents to her GP complaining of a swelling in her neck which she has noticed
in the last 2 weeks. She has felt more irritable although this is often transient. On examination, a diffuse
swelling is palpated with no bruit on auscultation. The most likely diagnosis is :
a) Hyperthyroidism
b) Simple goitre
c) Riedel’s thyroiditis
d) Thyroid carcinoma
e) Thyroid cyst
The answer is: Simple goitre

148
EMLE Trial Exams December 2020 (1000 Qs) Version (1.0) 22/12/2020

96) A 33-year – old male with a history of type 1 diabetes mellitus presents with nausea, vomiting, and
altered mental status. His chemistry panel shows: Na 125 mEq /L, K 3.8 mEq/L, Cl 90mEq /L, Cr 1.4mg/dl,
Glucose 720mg/dl. In addition to insulin and IV fluids, which of the following is the next most important
in management?
a) Sodium bicarbonate
b) Potassium
c) Fomepizole
d) Glucagon
e) Ceftriaxone plus azithromycin
The answer is: Potassium
97) Which of the following describes treating patients fairly and equitably?
a) Autonomy
b) Beneficence
c) Non-maleficence
d) Justice
e) maleficence
The answer is: Justice
98) Megaloblastic anemia occurs due to:
a) Folic acid deficiency
b) Zinc deficiency
c) Copper deficiency
d) B thalassemia
e) Iron deficiency anemia
The answer is: Folic acid deficiency
99) A 60-year-old man who is obese and heavy smoker presents with heartburn after heavy meal and
occasionally awakens him from sleep since several years. The heartburn is relieved by over the counter
antacids. Recently he complained of dysphagia to solids which is relieved by drinking water. Which of
the following is the appropriate line of management?
a) Avoid smoking
b) H2 antagonists
c) Proton pump inhibitors
d) UGI endoscopy
e) Weight reduction
The answer is: UGI endoscopy
100) Heel prick test used to diagnose
a) Hepatitis
b) Congenital hypothyroidism
c) Down Syndrome
d) Cow milk allergy
e) Lactose intolerance
The answer is: Congenital hypothyroidism

149
EMLE Trial Exams December 2020 (1000 Qs) Version (1.0) 22/12/2020

Exam 9
1) A 45-year-old woman presents for a routine physical examination and is found to have several small
masses within the right lobe of her thyroid gland. No enlarged lymph nodes are found. Her thyroid gland
is resected surgically and histologic sections from the tumor masses reveal multiple papillary structures
and scattered small, round, laminated calcifications. Which of the following histologic changes is most
likely to be present within these tumor masses?
a) Amyloid stromal invasion by malignant C cells
b) Blood vessel and capsular invasion by malignant follicles
c) Optically clear nuclei with longitudinal nuclear grooves“Orphan Annie eyes”
d) Sheets of small round cells with cytoplasmic glycogen
e) Undifferentiated anaplastic cells with giant cell formation
The answer is: Optically clear nuclei with longitudinal nuclear grooves“Orphan Annie eyes”
2) A 62-year-old man admitted 5 hours after left hemiplegia. ECG confirms AF. A CT head is normal. Which
of the following should be done in unilateral amaurosis fugax?*
a) CT angiography of the neck
b) MR angiography of the neck
c) Dupplex US of the neck
d) Cervical angiography
e) CT brain
The answer is: Dupplex US of the neck
3) Which part of the immunity is related to asthma?
a) Mast cell
b) Macrophage
c) Neutrophils
d) Lymphocytes
e) IgG
The answer is: Mast cell
4) The new obstetrical visit typically includes a general well woman assessment as well as pregnancy
evaluation. Which of the following is true about Pap smears taken from the uterine cervix during a
normal pregnancy?
a) They should be part of routine obstetric care, if needed, based on pap frequency for the individual
patient.
b) They are indicated only in patients with clinically assessed risks.
c) They are difficult to interpret because of gestational changes.
d) They are a cost-effective replacement for cultures for sexually transmitted diseases (STDs).
e) They are likely to induce uterine irritability.
The answer is: They should be part of routine obstetric care, if needed, based on pap frequency for the
individual patient.
5) Average incubation period of hepatitis B is:
a) 60 to 90 days
b) 30 days
c) 10 to 45 days
d) 180-280 days
e) None of the above
The answer is: 60 to 90 days

150
EMLE Trial Exams December 2020 (1000 Qs) Version (1.0) 22/12/2020

6) On her first prenatal visit, a 17-year-old single woman (gravida 1, para 0), 32 weeks by good dates, is
found to have vital signs as follows: BP, 135/85; P, 84; T, 98.6°F; and R, 20. She also has ankle and hand
edema and a uterine fundus measuring 42 cm with breech concordant twins on ultrasound. She has
normal pelvic measurements and the cervix is closed and soft, with the presenting part at station −1.
Her UA revealed no WBCs or bacteria with 2+ protein. Her hematocrit is 38, and her WBC count is 9800.
The next step in care of this patient should include which of the following?
a) trial of home bed rest for 24 hours, with repeat evaluation at that time
b) hospitalization with bed rest and frequent vital signs
c) oxytocin induction of labor
d) antihypertensive drugs
e) cesarean section because of the twins
The answer is: hospitalization with bed rest and frequent vital signs
7) Which of the following statement about gallstone ileus is not true?
a) The condition is seen most frequently in women older than 70.
b) Concomitant with the bowel obstruction, air is seen in the biliary tree.
c) The usual fistula underlying the problem is between the gallbladder and the ileum.
d) Ultrasound studies may be of help in identifying gallstone of the obstructing agent.
e) When possible, relief of small bowel obstruction should be accompanied bydefinitive repair of the
fistula since there is of significant incidence ofRecurrence if the fistula is left in place.
The answer is: The usual fistula underlying the problem is between the gallbladder and the ileum.
8) Which of the following statements is the most accurate regarding rickets?
a) Craniosynostosis is a clinical feature
b) Exclusive breastfeeding may be protective
c) Hyperphosphataemia is a cause
d) It lead to increased mineralisation of the growing bone
e) Vitamin D deficiency is the most common cause
The answer is: Vitamin D deficiency is the most common cause
9) As regards pituitary follicle stimulating hormone the following are correct EXCEPT:
a) A glycoprotein
b) Excreted in increased amount at menopause
c) FSH stimulates spermatogenesis in male
d) FSH with LH stimulate estrogen secretion and ovulation
e) Identical with human chorionic gonadotrophin
The answer is: Identical with human chorionic gonadotrophin
10) Tear drop sign is seen in:
a) Under screen in plain x-ray in rupture kidney.
b) in IVP in rupture kidney.
c) in U/S in rupture kidney.
d) None of the above.
e) all of the above
The answer is: in IVP in rupture kidney.
11) A 67-year-old woman presents to the emergency department with a 1-month history of malaise, weight
loss, worsening cough and haemoptysis. You notice her saturations are 92% on air and perform a blood
gas test with the follow-ing results: pH 7.37, pO2 6.7, pCO2 7.4, bicarbonate 35, saturations 91.7%.
Reference ranges: pO2 >50mmHg, pCO2 55 mmHg, bicarbonate 22–28 mmol/L, pH 7.35–7.45. What
blood gas picture does this represent?*
a) Fully compensated metabolic acidosis

151
EMLE Trial Exams December 2020 (1000 Qs) Version (1.0) 22/12/2020

b) Fully compensated metabolic alkalosis


c) Fully compensated respiratory acidosis
d) Type I respiratory failure
e) Venous sample.
The answer is: Fully compensated respiratory acidosis
12) A 40-year-old male with no past medical history presents to the outpatient clinic for routine visit. His
Hemoglobin A1C is 8%. The physician started him on glipizide. Which is the most important side effect of
this drug?
a) Lactic acidosis
b) Pancreatitis
c) UTI
d) Vaginal Candida
e) Hypoglycemia
The answer is: Hypoglycemia
13) A 34-year-old man presents to the emergency department with a 10-hour history of abdominal pain
associated with nausea and vomiting. On examination, the patient is lying still and has tenderness with
guarding in the right iliac fossa. His temperature is 37.6°C. He has no significant past medical history.
What is the most likely diagnosis?*
a) Appendicitis
b) Crohn disease
c) Meckel diverticulitis
d) Mesenteric adenitis
e) Renal colic
The answer is: Appendicitis
14) A 36 years-old with 3 children complains of heavy menses and seeking long term contraception. She has
BMI of 27. Her history is suggestive for two attacks of DVT last year. Regarding the contraceptives
suitable for her, Only one statement is NOT correct:
a) Subdermal implants can be used in this patient
b) Copper-IUD is not recommended for this lady
c) Mirena (LNG-IUD) is suitable for this lady
d) Combined oral contraceptive pills are the best choice in this lady.
e) Condom is contraindicated
The answer is: Combined oral contraceptive pills are the best choice in this lady.
15) Which of the following is essential for an informed consent taking?*
a) It could be taken from children
b) The patient should know everything about his disease but not the management
c) If the patient doesn’t speak Arabic, explain to him by your mother language
d) The consent could be oral
e) Competence is the most essential issue
The answer is: Competence is the most essential issue
16) A 44-year-old man presents to the GP having noticed a painless swelling in the right side of the scrotum.
On examination, the scrotum is swollen and non-tender. The swelling transilluminates and the testis
itself is not palpable. What is the most likely diagnosis?*
a) Epididymo-orchitis
b) Hydrocele
c) Inguinal hernia
d) Testicular torsion

152
EMLE Trial Exams December 2020 (1000 Qs) Version (1.0) 22/12/2020

e) Varicocele
The answer is: Hydrocele
17) Arterial blood gas sampling can be done at a point:*
a) Midway between ASIS and symphysis pubis
b) Midway between ASIS and pubic tubercle
c) Midway between ASIS and pubic crest
d) Lateral to the internal inguinal ring
e) None of the above
The answer is: Midway between ASIS and pubic tubercle
18) Which of the following elements is present in the new quick Sepsis – related Organ Function Assessment
( q SOFA ) score and may replace systemic inflammatory response syndrome criteria?*
a) Systolic blood pressure < 100mmHg
b) Glucose > 100mg/dL
c) Fever >38.5 C
d) Heart rate > 90 bpm
e) Hemoglobin < 10g/dL
The answer is: Systolic blood pressure < 100mmHg
19) Dexamethasone is indicated in which of the following condition :
a) Premature labor to prevent neonatal respiratory distress syndrome
b) Ectopic pregnancy to enhance fetal lung maturity
c) Spontaneous rupture of membrane at 39 week
d) At 38 weeks severe abruption placenta
e) Threatened abortion
The answer is: Premature labor to prevent neonatal respiratory distress syndrome
20) Counseling of a pregnant patient during early prenatal care should include detection of & information
on:
a) Smoking.
b) Alcohol abuse.
c) Drug abuse.
d) Avoiding infections.
e) All of the above.
The answer is: All of the above.
21) A 22-year-old man presents with a 6-month history of non-bloody diarrhea, malaise, recurrent
abdominal cramps, and temperature to 38.5 °C. Examination reveals a palpable, ill defined mass in the
right lower quadrant of the abdomen. Palpation causes local tenderness without guarding. Oral ulcers
are also noted. Which of the following is the most likely diagnosis?
a) Celiac disease
b) Chronic appendicitis
c) Crohn‘s disease
d) Diverticulitis
e) Irritable bowel syndrome
The answer is: Crohn‘s disease
22) When should you dispose of a plastic disposable apron?*
a) At the end of each shift
b) Every hour
c) Between caring for each patient
d) After it’s been washed once

153
EMLE Trial Exams December 2020 (1000 Qs) Version (1.0) 22/12/2020

e) none of the above


The answer is: Between caring for each patient
23) A 33-year – old male with a history of type 1 diabetes mellitus presents with nausea , vomiting , and
altered mental status . His chemistry panel shows : Na 125 mEq /L , K 3.8 mEq/L , Cl 90mEq /L , Cr
1.4mg/dl, Glucose 720mg/dl. In addition to insulin and IV fluids , which of the following is the next most
important in management ?
a) Sodium bicarbonate
b) Potassium
c) Fomepizole
d) Glucagon
e) Ceftriaxone plus azithromycin
The answer is: Potassium
24) In the elevator, your senior resident says, “Before I forget, make sure you send Mr. Shady home on his
usual HIV medications.” There are other people in the elevator. What professional responsibility has
your senior resident violated?
a) Commitment to improving quality of care
b) Commitment to maintaining trust
c) Commitment to patient confidentiality
d) Commitment to professional competence
e) Commitment to scientific knowledge
The answer is: Commitment to patient confidentiality
25) Which of the following causes pancytopenia?*
a) Parvovirus B19
b) HAV
c) HBV
d) HCV
e) Pneumococci
The answer is: Parvovirus B19
26) A 29-year-old female is noted to have an elevated bilirubin during a viral illness. Gilbert's syndrome is
suspected. Which one of the following tests may confirm the diagnosis?*
a) Nicotinic acid test
b) Fecal fat excretion
c) Urine analysis
d) CBC
e) HIDA scan
The answer is: Nicotinic acid test
27) A 36-year-old woman presents with a 2-month history of recurrent severe right-side headaches
associated with vomiting and blurring of vision. She feels generally unwell, lethargic and nauseated
about 12-24 hours before each episode. What is the cause of this headache?*
a) Giant cell arteritis
b) Subarachinoid hemorrhage
c) Tension headache
d) Cluster headache
e) Migraine
The answer is: Migraine
28) A 19-year-old primigravida at term presents to labor and delivery reporting irregular contractions and
rupture of membranes 21 hours prior to arrival. She has not received prenatal care but reports that her

154
EMLE Trial Exams December 2020 (1000 Qs) Version (1.0) 22/12/2020

pregnancy was uncomplicated. She is afebrile, and electronic fetal monitoring is reactive with occasional
mild variable decelerations.Which method is the most sensitive test to confirm rupture of membranes?
a) nitrazine test
b) vaginal pooling
c) pelvic examination
d) ferning
e) Coombs’ test
The answer is: ferning
29) A 55-year-old white man presents to ER with severe abdominal pain that has been radiating through to
his back for the past 2 days. He occasionally drinks alcohol. The man appears acutely ill and is sitting in a
markedly bent-over position. He has also recurrent vomiting.. His temperature is 38.1 C, blood pressure
is 85/60 mm Hg, pulse is 120/min and regular, and respirations are 22/min and shallow. Pressure on the
upper portion of the abdomen intensifies the pain. A complete blood count shows a white blood cell
count of 18,000/mm3 with predominately neutrophils. Which of the following is the most appropriate
next step in diagnosis?
a) Barium meal
b) Chest x-ray
c) UGI endoscopy
d) Serum amylase and lipase levels
e) Ultrasonography of abdomen
The answer is: Serum amylase and lipase levels
30) What is the volume of urine that indicates polyuria?
a) 1 L / day
b) 2 L / day
c) 3 L / day
d) 4 L / day
e) 5 L / day
The answer is: 4 L / day
31) A 70-year-old ex-smoker presents to his GP with a 3-month history of weight loss and worsening
dysphagia, where he is now unable to swallow soft foods and liquids. He is also experiencing nausea and
vomiting on several occasions. What is the most likely cause of his vomiting?*
a) Gastric carcinoma
b) Oesophageal carcinoma
c) Opiate analgesia
d) Pancreatitis
e) Gastritis
The answer is: Oesophageal carcinoma
32) Which of the following statements applies to infants with gastroenteritis?
a) In most instances require treatment with an antibiotic which is not absorbed from the gut
b) Always have blood in their stools
c) Should be admitted to a hospital if they are unable to tolerate fluid orally
d) Always develop lactose intolerance
e) Astrovirus is the most common cause
The answer is: Should be admitted to a hospital if they are unable to tolerate fluid orally
33) Dehydration is classified into isotonic, hypertonic or hypotonic according to:
a) S. Potassium level
b) S. Sodium level

155
EMLE Trial Exams December 2020 (1000 Qs) Version (1.0) 22/12/2020

c) PH of blood
d) S. calcium level
e) S. magnesium level
The answer is: S. Sodium level
34) A 63-year-old male with a history of multiple myeloma presents to ICU with acute onset of extreme
weakness , blurry vision , considerable mucosal bleeding and severe headache .A blood sample was sent
for CBC and chemistry panel , but the laboratory is unable to deal with the sample , citing “inappropriate
blood sample “ .Which of the following is the most appropriate emergent definitive management ?
a) Hemodialysis
b) Plasmapheresis
c) Platelet transfusion
d) Packed red blood cell transfusion
e) Erythropoietin
The answer is: Plasmapheresis
35) All of these drugs can be used as tocolytic to stop labor, EXCEPT*^
a) Salbutamol ventolin
b) Diazepam (valium)
c) Calcium channel blocker
d) Indomethacin non steroidal anti inflammatory drugs
e) Ritodrine (β agonist).
The answer is: Diazepam (valium)
36) Which one of the following anti-epileptic drugs is safe in pregnancy?
a) Involve restoration of the normal anatomy whenever possible
b) The aim is to get rid of all implants
c) Operative laparoscopy is suitable in most of the cases
d) Removal of the cyst with adjacent ovarian tissue may affect ovarian reserve.
e) Endometriotic lesions could be dealt with by coagulation
The answer is: The aim is to get rid of all implants
37) What is the average lifespan of an erythrocyte once it has entered the bloodstream?
a) 1 day
b) 10 days
c) 120 days
d) 200
e) 360
The answer is: 120 days
38) Clinical signs of active vitamin D deficiency rickets include all except:
a) Craniotabes
b) Rickety rosary chest
c) Broadening at wrists and ankles
d) Early closure of fontanels
e) Leg deformity
The answer is: Early closure of fontanels
39) ln a case of pleural effusion. The following statements are correct except that:
a) The diagnosis can be made with as little as 100 ml of fluid.
b) The trachea may be displaced to the opposite side.
c) Aspiration of blood-stained fluid is highly suspicious of bronchogenic carcinoma.
d) Open biopsy of the pleura is necessary.

156
EMLE Trial Exams December 2020 (1000 Qs) Version (1.0) 22/12/2020

e) Rapid aspiration of the fluid is not advisable.


The answer is: The diagnosis can be made with as little as 100 ml of fluid.
40) A Nulliparous female 34 years old, presented with pelvic ultrasound report showing single sub-mucus
uterine fibroid 10x8 cm. Her hemoglobin was 8.7 gm/dl. The INCORRECT statement regarding the
management is#
a) The main symptom of this patient is heavy menstrual bleeding (HMB).
b) The operation of choice will be myomectomy
c) Uterine artery embolization can be a choice for the management
d) The next required investigation is true cut needle biopsy
e) An endometrial biopsy is to be examined histopathologically
The answer is: The next required investigation is true cut needle biopsy
41) You are examining a child in the ER after history of head trauma you assessed his Glasgow coma score
found he is 13 what is the best advice?*
a) Neurological examination
b) Urgent CT
c) Ortho assessment
d) Ventilate
e) Urgent MRI
The answer is: Urgent CT
42) What of the following causes normocytic anemia?*
a) Thalassemia
b) Sickle cell anemia
c) Vitamin B12 deficiency anemia
d) Sideroblastic anemia
e) Iron deficiency anemia
The answer is: Sickle cell anemia
43) Child 3 years old fell off from 4 m stairs and then after crying he vomited four times otherwise he was
good and starts to act normal what is your recommendation?#
a) Urgent MRI
b) Urgent CT
c) Neurological examination
d) Reassurance
e) Orthopedic reassessment
The answer is: Urgent CT
44) An 85-year-old woman is being transferred to an acute rehabilitation facility following a hospital
admission for hip replacement surgery. Postoperatively during her hospital stay, she is started on deep
vein thrombosis (DVT) prophylaxis medication with plans to continue the medication upon discharge.
The intern and nurse who are discharging the patient fail to convey this new medication to the receiving
treatment team at the rehabilitation center. The patient is not continued on her anticoagulation
medication and sustains a DVT, leading to a fatal pulmonary embolus 3 weeks after transfer. Which of
the following actions will facilitate quality improvement and the prevention of a similar error in the
future?**
a) Determine which staff member(s) failed to order the medication
b) Develop a process to increase the use of medication reconciliation
c) Send a memo to all staff about the importance of DVT prophylaxis
d) Educate patients about the dangers of DVT following hip surgery
e) Conduct monthly audits to monitor medication errors at transitions of care

157
EMLE Trial Exams December 2020 (1000 Qs) Version (1.0) 22/12/2020

The answer is: Develop a process to increase the use of medication reconciliation
45) A 72-year-old man presents with a 2 × 2 cm swelling in the left groin crease. The swelling is non-tender
and irreducible. On examination the swelling is not mobile, and not tethered to the underlying tissue,
there is a smaller 1 × 1 cm swelling on the contralateral side, and several similar 1 × 1 cm swellings in the
axilla. What is the most appropriate diagnosis?
a) Saphena varix
b) Abscess
c) Femoral hernia
d) Femoral artery pseudoaneurysm
e) Lymphadenopathy
The answer is: Lymphadenopathy
46) An 88-year-old male falls and hits his head, chest, and hip. The most UNLIKELY injury for the patient to
suffer is?
a) Epidural hematoma
b) Hip fracture
c) Odontoid fracture
d) Rib fractures
e) Subdural hematoma
The answer is: Epidural hematoma
47) A positive nitrazine test is:
a) Strong evidence of rupture of the membranes.
b) Presumptive evidence of intact membranes.
c) An evidence of intact membranes.
d) Presumptive evidence of intact membranes.
e) None of the above.
The answer is: Strong evidence of rupture of the membranes
48) Which percentage of gall stones is rodio-opaque?
a) 0.1
b) 0.4
c) 0.7
d) 0.9
e) 0.95
The answer is: 0.1
49) A 6-month-old infant with fever, acute hypotonia, hyporeflexia, respiratory distress and asymmetric
paralysis of both lower limbs most likely has:
a) Congenital myopathy
b) Werding Hoffman disease
c) Meninogoencephalitis
d) Acute poliomyelitis
e) None of the above
The answer is: Acute poliomyelitis
50) Which of the following is not a basic component of an obstetric forceps ?
a) Blade
b) Handle
c) Lock
d) Stem
e) Shank

158
EMLE Trial Exams December 2020 (1000 Qs) Version (1.0) 22/12/2020

The answer is: Stem.


51) Leukemia causes which type of splenomegaly?*
a) Immune hyperplasia
b) RES hyperplasia
c) Congestion
d) Infiltration
e) Neoplastic
The answer is: Neoplastic.
52) A 22- year-old previously healthy female is admitted to ED with severe dyspnea , palpitation and
extreme fatigue. The patient had been diagnosed with viral infection 7 days before .Upon arrival she is
pale and ill . Pulse 143 bpm , RR 28 breath per minute , BP 105/70mmHg , SaO2 90%. X-ray showed fluffy
bilateral infiltrates , lung examination revealed tachypnea and bilateral fine basal crepitation without
wheeze .Which of the following is the most likely diagnosis ?*
a) Supraventricular tachycardia
b) Acute coronary syndrome
c) Acute myocarditis and acute heart failure
d) Infective endocarditis
e) Previously undiagnosed congenital heart disease
The answer is: Acute myocarditis and acute heart failure.
53) A 78-year – old female presents with acute onset of right –sided arm and leg weakness 1 hour prior to
arrival .You suspect stroke and immediately order a noncontrast CT brain . The CT of brain is
Unremarkable .Which of the following is the most appropriate conclusion to draw from this CT results ?*
a) There is no stroke
b) There is no ischemic stroke
c) There is no meningitis
d) There is no intracranial hemorrhage
e) There is no cervical artery dissection
The answer is: There is no intracranial hemorrhage
54) A 79-year-old woman with a history of coronary artery disease who underwent a coronary artery bypass
graft (CABG) surgery 7 years ago is brought to the ED by her family for 2 days of worsening shortness of
breath. She also has not gotten out of bed for 2 days and is confused. She denies chest pain,fever,or
cough. Her BP is 85/50 mmHg, HR is 125 b/m, RR is 26 b/m, and oral temperature is 36.7°C. On
examination reveals tachycardia with no murmurs. With bilateral lung basal rales. The abdomen is soft,
nontender, and nondistended. Lower extremities have 2+ edema to the knees bilaterally. Which of the
following is the most likely diagnosis?
a) Hypovolemic shock
b) Neurogenic shock
c) Cardiogenic shock
d) Anaphylactic shock
e) Septic shock
The answer is: Cardiogenic shock
55) The following drugs cross the placenta to the fetus, EXCEPT :
a) Heparin
b) Tetracycline
c) Warfarin
d) Diazepam
e) Aspirin

159
EMLE Trial Exams December 2020 (1000 Qs) Version (1.0) 22/12/2020

The answer is: Heparin.


56) A 4-year-old girl presents with coryzal symptoms, cough, fever & abdominal pain. There is no dysuria, &
urine dipstick is unremarkable. On examination, the pain is in the left upper quadrant but is not severe.
a) Urinary tract infection (UTI)
b) Appendicitis
c) Pyelonephritis
d) Left basal chest infection
e) Mesenteric adenitis
The answer is: Left basal chest infection
57) A 5-year old boy presents to the assessment unit with a purpruic rash on his legs and buttocks,with
associated swelling and pain in his right knee, resulting in a limp. He had a recent viral upper repiratory-
tract infection(URTI)but is otherwise fit and well A urine dip revels hematuria only. What is the most
likely diagnosis ? select one answer only
a) haemophilia
b) hencho -schonlein (HSP)
c) Septic arthritis
d) tranint aynovitis
e) urinary tract infection (UTI)
The answer is: hencho -schonlein (HSP)
58) A 65-year-old male presents with progressive dysphagia for solid foods that progressed to liquids and is
associated with 6 KG weight loss over the past few weeks. Which is the most likely diagnosis?
a) Esophageal cancer
b) Barret’s esophagus
c) GERD
d) Achalasia
e) Diffuse esophageal spasm
The answer is: Esophageal cancer
59) The infant laughs at:
a) The 1st month
b) The 2nd month
c) The 4th month
d) The 9th month
e) 1 year
The answer is: The 4th month
60) Manifestations of rheumatic carditis include all of the following except:#
a) Tachycardia
b) Muffled heart sounds
c) Gallop rhythm
d) Pericardial rub
e) Accentuated second heart sound
The answer is: Accentuated second heart sound
61) Iron supplementation in a healthy full-term infant should start:
a) At birth
b) At 6 months
c) At 12 months
d) At 24 months
e) 36 months

160
EMLE Trial Exams December 2020 (1000 Qs) Version (1.0) 22/12/2020

The answer is: At 6 months


62) APGAR's score includes all the followings EXCEPT:
a) Skin color.
b) Muscle tone.
c) Blood pH.
d) Heart rate.
e) Respirations
The answer is: Blood pH.
63) The following factors detract from the diagnostic sensitivity of FAST scanning, except
a) Surgical emphysema
b) A patient with high body mass index
c) Operator inexperience
d) Previous surgery
e) Large-volume intraperitoneal blood loss
The answer is: Large-volume intraperitoneal blood loss
64) You notice that you are finding it harder to read the notes at work, including when writing prescriptions.
You made a mistake earlier this week that almost resulted in a patient receiving the wrong dose of
gentamicin. There is a history of hereditary macular degeneration in your family, you are concerned you
may be developing symptoms. What should you do?***
a) Ignore it, you may lose your job if you tell someone
b) Buy some reading glasses online and hope for the best
c) Talk to your clinical/educational supervisor about your concerns
d) Do not take any action, hope that nobody will notice
e) Take time off work, phone in with the flu
The answer is: Talk to your clinical/educational supervisor about your concerns
65) Which of the following statements regarding management of the acute wound is incorrect?#
a) A bleeding wound should be elevated and a pressure pad applied.
b) Clamps may be sometimes needed to be put on bleeding vessels blindly
c) Anesthesia is essentially required in the assessment of wounds.
d) A thorough debridement is essential.
e) Repair of all damaged structures may not be attempted in a tidy wound.
The answer is: Clamps may be sometimes needed to be put on bleeding vessels blindly
66) you are asked to review a 2-day full-term boy who started bleeding excessively following circumcision.
You request a full blood count and clotting. The full blood count is normal and the APTT is prolonged at
95 secounds PT is normal. His uncle has a similar condition what is the most likely diagnosis ?
a) Factor V Leiden disease
b) Haemophilia
c) ITP
d) Severe von Willibrand's disease
e) Vitamine K defecincy
The answer is: Haemophilia
67) A pregnant patient is receiving magnesium infusion for pre-eclampsia .Which of the following is the
earliest indicator of significant toxicity ?#
a) Diaphoresis
b) Loss of deep tendon reflexes
c) Slowed respiratory rate
d) Sinus tachycardia

161
EMLE Trial Exams December 2020 (1000 Qs) Version (1.0) 22/12/2020

e) Visual floaters
The answer is: Loss of deep tendon reflexes
68) A blind-ending epithelialized tract, which may lead to a cavity. What is the most likely surgical
pathology?*
a) Fissure
b) Hernia
c) Sinus
d) Seroma
e) Abscess
The answer is: Sinus.
69) A 70-year-old alcoholic presents with a tender upper abdominal mass. CT shows athick-walled, rounded,
fluid-filled mass adjacent to the pancreas.the most likely cause for the clinical findings.
a) renal cell carcinoma
b) ovarian carcinoma
c) gastric carcinoma
d) sigmoid carcinoma
e) pancreatic pseudocyst
The answer is: pancreatic pseudocyst
70) A 32-year-old male presents to the outpatient clinic complaining of recurrent attacks of sudden onset of
left-hand contractions that fades on its own. His consciousness in intact during the attacks. Which is the
best treatment?
a) Lorazepam
b) phenytoin
c) Valproate
d) Carbamazepine
e) Lamotrigine
The answer is: Carbamazepine
71) A 23-year-old male presents to the colorectal outpatient clinic having been referred by his GP. He has
noticed it increasingly painful to pass stool. He experiences an acute sharp pain in his anus on defecation
which persists for hours after evacuation. He has not experienced any weight loss; however, has been
increasingly constipated. There is some bright red blood on the toilet paper when he wipes. You
examine his anus and rectum; however, despite a normal external examination, you are unable to pass a
gloved finger to perform a digital examination due to severe discomfort and sphincter spasm. From the
following list, what is the likeliest cause of this man’s symptoms?
a) Anal squamous cell carcinoma
b) Acute anal fissure
c) Grade IV haemorrhoids
d) Diverticular disease
e) Anal fistula
The answer is: Acute anal fissure
72) Which is the best initial step for a patient presenting with chest pain?*
a) Chest Xray
b) Arterial blood gases
c) Cardiac Enzymes
d) ECG
e) Complete blood picture
The answer is: ECG.

162
EMLE Trial Exams December 2020 (1000 Qs) Version (1.0) 22/12/2020

73) In mumps, all of the following statements are true except:


a) The incubation period is usually 21 days
b) Orchitis is typically bilateral
c) A septic meningitis is a complication
d) Amylase may be increased despite no evidence of pancreatitis clinically
e) Sublingual swelling may occur
The answer is: Orchitis is typically bilateral
74) A 23-year-old man presents with a week’s history of fever and sore throat. He developed a macular rash
after being prescribed ampicillin by his GP. On examination he has enlarged posterior cervical nodes,
palatal petechiae and splenomegaly. The most likely cause for the clinical findings :
a) spherocytosis
b) infectious mononucleosis
c) cutaneous leishmaniasis
d) idiopathic thrombocytopenic purpura
e) myeloma
The answer is: infectious mononucleosis
75) Which of the following is the most common cause of postartum hemorrhage after a cesarean delivery?
a) Uterine atony.
b) Uterine laceration.
c) coagulopathy.
d) Uterine inversion.
e) Retained placenta.
The answer is: Uterine atony.
76) A 35 –year –old female presents to the ER with palpitation , chest pain and lightheadedness . Her BP
70/40mmHg , pulse regular , 175bpm, ECG shows narrow complex atrioventricular nodal re-entrant
tachycardia ( AVRNT ) .Which of the following next step in management ?
a) A denosine 6mg IV
b) Diltiazem 20 mg IV
c) Amiodarone 300mg IV
d) Synchronized cardioversion at 50 J
e) Defibrillation at 200 J
The answer is: Synchronized cardioversion at 50 J
77) An abnormal connection containing chronic granulation tissue between two epithelial lined surfaces.
What is the most likely surgical pathology?
a) Cyst
b) Pseudoaneurysm
c) Fistula
d) Haematoma
e) Abscess
The answer is: Fistula.
78) Which statement about acute non calcular cholecystitis is correct?
a) The disease is often accompanied by or associated with other conditions.
b) The diagnosis is often difficult.
c) The mortality rote is higher than that for acute calculous cholecystitis.
d) The disease has been treated successfully by percutaneous cholecystectomy.
e) All of the above
The answer is: All of the above.

163
EMLE Trial Exams December 2020 (1000 Qs) Version (1.0) 22/12/2020

79) In twin deliveries: Which is true?


a) The first twin is at greater risk than the second
b) They usually go post date
c) Epidural analgesia is best avoided
d) Commonest presentation is cephalic and second breach
e) There is increased risk of postpartum hemorrhage
The answer is: There is increased risk of postpartum hemorrhage
80) A 22-year-old patient presents with a hematocrit of 31% at 28 weeks’ gestation. Her mean corpuscular
volume (MCV) is 105, her mean corpuscular hemoglobin (MCH) is 33, and her mean corpuscular
hemoglobin concentration (MCHC) is 36. Serum iron is 100 mg/dL. There is no evidence of abnormal
bleeding.The most likely cause of anemia in this patient is which of the following?
a) gastrointestinal (GI) bleeding
b) G6PD deficiency
c) iron deficiency
d) folic acid deficiency
e) pernicious anemia
The answer is: folic acid deficiency
81) The proper age to start weaning is:
a) 6 months
b) 12-15 months
c) 1-3 months
d) 9-12 months
e) 2-4 months
The answer is: 6 months.
82) A 55-year-old man presents to the Emergency Department with a 2-day history of vomiting and severe
epigastric pain radiating through to his back. He admits to a large alcohol binge the night before the
onset of the pain. On examination, he is tachycardic with generalized abdominal tenderness and
localised guarding in the epigastrium. An erect chest X-ray shows no obvious abnormality. You suspect a
diagnosis of acute pancreatitis. The surgical registrar asks you to ‘score’ the patient. Which of the
following forms part of the Glasgow Coma Score for acute pancreatitis?#
a) Heart rate
b) Systolic blood pressure
c) Temperature
d) PaO2
e) Amylase
The answer is: PaO2
83) Disseminated intravascular coagulation (DIC)*^
a) Can be managed by leukocyte transfusion
b) Can cause decrease fibrinogen degeneration products
c) Can cause decrease PT , PTT
d) Is characterized by increase platelet count
e) Can be encountered in case of IUFD
The answer is: Can be encountered in case of IUFD
84) Which of the following quality measures is used for counting HbA1C for diabetic patients?#
a) Structural
b) Process
c) Outcome

164
EMLE Trial Exams December 2020 (1000 Qs) Version (1.0) 22/12/2020

d) balancing
e) Non-specific
The answer is: Outcome.
85) A 34-year old female with seizure disorder presents to the ER with a generalized tonic – clonic seizure .
She is on phenytoin for her seizures . Postictal examination revealed mild fever , pulse 85 pbm , BP
150/80 . Physical examination is normal . Which of the following is the most likely cause for her
seizures?
a) Alcohol use
b) Febrile seizure
c) Medication non compliance
d) Central nervous system infection
e) Mass lesion
The answer is: Medication non compliance
86) Protein caloric malnutrition is characterized by all except:
a) Decreased body weight
b) Anemia and signs of vitamin deficiencies
c) Edema starting in the dorsa of the hands and feet
d) High IQ
e) Repeated infection
The answer is: High IQ.
87) A 36-year – old man rescued from a building on fire presents with headache , vertigo , nausea , vomiting
and mild confusion( GCS 14 ) .He is without any evidence of burns or external injury . What is the most
appropriate next step in his management plan?*
a) Tight –fitting mask with 100% oxygen
b) 24% oxygen by nasal cannula
c) Hyperbaric oxygen in a hyperbaric chamber
d) Intubate and provide IPPV on 100% oxygen
e) Secure venous access
The answer is: Tight –fitting mask with 100% oxygen
88) What is the next step in the management of stroke?
a) CT brain
b) MRI brain
c) Cranial US
d) Duplex US on neck
e) Diagnostic angiography
The answer is: CT brain.
89) A 55-year-old obese diabetic man presented to the outpatient clinic for checkup. On examination he was
clinically free. Laboratory investigations revealed SGPT 80 U/L (n = 0-49) and SGOT 60 U/L (n = 0-34).
Other laboratory tests were normal. Abdominal ultrasonography revealed mildly enlarged fatty liver.
Which of the following is the next step in management?
a) Advice weight reduction
b) Ask for viral markers
c) Assurance of the patient with no further management
d) Silymarin for one month and repeat SGPT and SGOT
e) Ursodeoxycolic acid
The answer is: Ask for viral markers
90) Complications of pancreatitis include:*

165
EMLE Trial Exams December 2020 (1000 Qs) Version (1.0) 22/12/2020

a) Pseudocyst
b) Retroperitoneal haematoma
c) Tetany
d) Abscess formation
e) All of the above
The answer is: All of the above.
91) A 50-year-old man who is obese and heavy smoker presents with occasional heartburn after heavy meal
and occasionally awakens him from sleep since several years. The heartburn is relieved by over the
counter antacids. Recently he complained of recurrence of symptoms and is not relieved by antacids.
Which of the following is the best line of management?
a) Avoid smoking
b) H2 antagonists
c) Proton pump inhibitors
d) Raise bed head 15 cm
e) Weight reduction
The answer is: Proton pump inhibitors
92) A 56-year-old male presents with postoperative bloating. The consulting surgeon suspects an ileus.
Which of the following lab findings is most likely contributing to the patient’s symptoms?
a) Lactic acidosis
b) Hypokalemia
c) Hypomagnesemia
d) Hypercalcemia
e) Uremia
The answer is: Hypokalemia.
93) Which of the following elements is present in the new quick Sepsis – related Organ Function Assessment
( q SOFA ) score and may replace systemic inflammatory response syndrome criteria?*
a) Systolic blood pressure < 100mmHg
b) Glucose > 100mg/dL
c) Fever >38.5 C
d) Heart rate > 90 bpm
e) Hemoglobin < 10g/dL
The answer is: Systolic blood pressure < 100mmHg
94) Which of the following drugs is most likely to cause impaired glucose tolerance?*
a) Interferon alpha
b) Bromocriptine
c) Imipramine
d) Montelukast
e) Strontium
The answer is: Interferon alpha
95) The principal single cause of bronchiolitis is:
a) Haemophilus influenza
b) Pneumococcus
c) Para influenza
d) Coxsackie virus A
e) Respiratory syncytial virus (RSV)
The answer is: Respiratory syncytial virus (RSV)
96) Leukemia causes which type of splenomegaly?*

166
EMLE Trial Exams December 2020 (1000 Qs) Version (1.0) 22/12/2020

a) Immune hyperplasia
b) RES hyperplasia
c) Congestion
d) Infiltration
e) Neoplastic
The answer is: Neoplastic.
97) Regarding Hyperemesis gravidarum, which one of the following items is TRUE?
a) Is a complication of multiple pregnancy
b) Not known to happen in molar pregnancy
c) Worsen in missed abortion
d) Liver function test is not required
e) Urine for culture and sensitivity is not important
The answer is: Is a complication of multiple pregnancy
98) Most infants start teething at:
a) 10 kg
b) 12 kg
c) 14 kg
d) 16 kg
e) 7 kg
The answer is: 7 kg.
99) Which of the following is the narrowest portion of the pediatric airway?
a) Nasopharynx
b) Oropharynx
c) True cords
d) False cords
e) Cricoid cartilage
The answer is: Cricoid cartilage
100) A 68 – year-old male with a history of chronic obstructive pulmonary disease ( COPD ) presents to
the emergency department with infective exacerbation . He experienced similar condition 6 months ago
and was intubated and added (I do not ever want to be intubated ).You discuss the use of non invasive
positive pressure ventilation ( NIPPV) and he is agreeable . With which of the following comorbidities is
this type of ventilation is most appropriate ?
a) Excessive secretion
b) Decreased sensorium
c) Hypertension
d) Midfacial trauma
e) In-cooperative patient
The answer is: Hypertension

167
EMLE Trial Exams December 2020 (1000 Qs) Version (1.0) 22/12/2020

Exam 10
1) A 40-year-old woman has lipid investigations suggesting familial hypercholesterolemia. This condition is
characterized by increased risk for premature atherosclerosis and by the occurrence of tuberous and
tendon xanthomas. Before making the assumption of familial hypercholesterolemia, secondary causes
need to be considered. Which of the following conditions is most likely to cause secondary
hyperlipidemia?
a) cholestatic liver disease
b) alcoholism
c) malabsorption syndromes
d) chronic lung disease
e) estrogen replacement
The answer is: cholestatic liver disease
2) A pregnant woman not previously known to be diabetic, who is at 26 weeks’ gestation, had a routine50-
g (GTT) with a 1-hour blood glucose value of 144 mg/dL. A follow-up 100-g, 3-hour oral GTTrevealed
plasma values of fasting blood sugar of 102; 1 hour, 180; 2 hours, 162; and 3 hours, 144. Youshould do
which of the following?
a) begin American Diabetes Association (ADA) diet and daily glucose monitoring
b) repeat the GTT in early or mid-third trimester
c) start oral hypoglycemic agents in the diet
d) perform an immediate Contraction Stress Test (CST)
e) treat the patient as one with normal gestation
The answer is: begin American Diabetes Association (ADA) diet and daily glucose monitoring
3) A 55-year-old male with history of Diabetes and hypertension presents to the emergency department
with chest pain that started 1 hour ago. ECG showed ST elevation. Which is the most definitive
treatment?*
a) Aspirin
b) Percutaneous coronary angiography
c) Warfarin
d) Metoprolol
e) Oxygen
The answer is: Percutaneous coronary angiography
4) A 30-year-old woman is admitted with a massive post-partum haemorrhage. She is shocked and
confused. Her husband informs you they are both Jehovah’s Witnesses and will not accept blood
transfusions even in life-saving circumstances. He produces a signed and witnessed document
confirming her refusal of blood products. Which of the following is the most suitable course of action?
a) Follow patient’s advanced directive
b) Proceed under presumed consent
c) Written consent
d) Treat in patient’s best interests
e) Court order
The answer is: Follow patient’s advanced directive
5) A 9-year-old boy presents with a right iliac fossa pain, vomiting, anorexia and pyrexia. You diagnose
appendicitis and ask the boy’s parents for consent for an appendicectomy. The parents refuse consent
as they do not believe in 'modern medicine'. Which of the options above apply to the following
situation?
a) Written parental consent
b) Proceed under presumed consent

168
EMLE Trial Exams December 2020 (1000 Qs) Version (1.0) 22/12/2020

c) Written consent
d) Treat in patient's best interests
e) Court order
The answer is: Court order
6) A 56-year- female with long standing hypertension, renal failure on dialysis, and a seizure disorder
presents to the emergency department after a generalized seizure. .After control of seizure, there was
persistent bleeding tongue wound. Platelet count was 162000/uL , hemoglobin was 8.5 g/dL .Which of
the following is the best approach to stop this bleeding ?
a) Platelet transfusion
b) Emergent hemodialysis
c) IV desmopressin ( dDAVP )
d) Suture repair of the tongue wound
e) Estrogen
The answer is: IV desmopressin ( dDAVP )
7) ITP characterized by:
a) Decreased platelet number
b) Decreased platelet function
c) Vascular wall defect
d) Deficient factor VIII
e) Deficient factor VII
The answer is: Decreased platelet number
8) A 35-year-old orthopedic resident presented with fever 38.5 °C, severe crampy abdominal pain and
bloody diarrhea since 2 days. He was afraid of having colorectal cancer. Which of the following is the
most likely diagnosis?
a) Acute appendicitis
b) Acute infectious diarrhea
c) Celiac disease
d) Diverticulosis
e) Irritable bowel syndrome
The answer is: Acute infectious diarrhea
9) In normal pregnancy, the value of β-hCG doubles every:
a) 2 days.
b) 4 days.
c) 8 days.
d) 10 days.
e) 14 days.
The answer is: 2 days.
10) A 27-year-old male is hammering nails at a construction site without eye protection and feels something
strike his right eye. He is presented to the ED complaining of painful eye. Which of the following is the
safest and most accurate for him to locate the potential foreign body?*
a) X-ray
b) Ultrasonography
c) MRI
d) CT scan
e) Nuclear medicine scan
The answer is: CT scan
11) What is the best management of pain in a patient with CKD?*

169
EMLE Trial Exams December 2020 (1000 Qs) Version (1.0) 22/12/2020

a) Give him morphine


b) Give him ibuprofen
c) Give him fentanyl
d) Give him tramadol
e) Give him oxycodone
The answer is: Give him fentanyl
12) A 32-year old male is brought to the ED after collapsing due to exhaustion while running a marathon.
Which of the following features indicate a diagnosis of heat stroke rather than simple exhaustion?
a) Temperature > 39.5C
b) Myalgia
c) Dehydration
d) Altered mental status
e) Parethesias
The answer is: Altered mental status
13) A 60-year-old man is admitted to the coronary care unit with a large anterior wall myocardial infarction.
On his second hospital day, he begins to complain of the sudden onset of numbness in his right foot and
an inability to move his right foot. On physical examination, the right femoral, popliteal, and pedal
pulses are no longer palpable. The left lower extremity is normal. Which of the following is the most
appropriate management of this patient?
a) Duplex imaging of the right lower extremity arteries
b) CT angiogram of the right lower extremity
c) CT angiogram of bilateral lower extremities
d) Embolectomy of the right femoral artery
e) Embolectomy of the right femoral artery with exploration of the contralateral femoral artery
The answer is: Embolectomy of the right femoral artery
14) Weaning means:
a) Stop artificial feeding
b) Stop breast feeding
c) Introduction of adult food
d) Introduction of cow milk
e) None of the above
The answer is: Introduction of adult food
15) These conditions are always an indication for caesarean section. Which one of these is an absolute
indication for C/S :
a) Twin pregnancy
b) Breech presentation
c) Severe PET
d) Major degree placenta previa
e) IUG
The answer is: Major degree placenta previa
16) The following are causes of short stature except:
a) Pott’s disease of the spine
b) Marfan syndrome
c) Emotional deprivation
d) Hypothyroidism
e) Achondroplasia
The answer is: Marfan syndrome

170
EMLE Trial Exams December 2020 (1000 Qs) Version (1.0) 22/12/2020

17) Which is the best study design to calculate incidence?*


a) Case report
b) Case series
c) Cross-sectional
d) Cohort
e) Case control
The answer is: Cohort
18) A 47-year-old man with a history of upper abdominal pain presents with constant severegeneralized
abdominal pain. On examination he is distressed and has a rigidabdomen. Pulse is 120/min, BP 90/60
mmHg
a) large bowel obstruction
b) acute pancreatitis
c) perforated viscus
d) appendicitis
e) small bowel obstruction
The answer is: perforated viscus
19) The 16-year-old daughter consents to medication, but the mother refuses what is the most appropriate
action to do?**
a) Write the prescription
b) Do not write the prescription
c) Postpone the prescription
d) Convince the daughter
e) Report the event to the police
The answer is: Do not write the prescription
20) Most common cause of first trimester abortion*^
a) Chromosomal abnormalities
b) Syphilis
c) Rhesus isoimmunization
d) Cervical incompetence
e) Bifurcate uterus
The answer is: Chromosomal abnormalities
21) The most common pediatric cancer is:#
a) Colon cancer
b) Breast cancer
c) Acute lymphoblastic leukemia
d) Lung cancer
e) Adrenocortical carcinoma
The answer is: Acute lymphoblastic leukemia
22) Which of these features is associated with renal osteodystrophy?
a) Increased gut absorption of calcium
b) Distal myopathy
c) Hypophosphataemia
d) Decreased serum alkaline phosphatase
e) Skeletal features of osteomalacia and hyperparathyroidism
The answer is: Skeletal features of osteomalacia and hyperparathyroidism
23) The most characteristic finding in urine analysis in poststreptococcal glomerulonephritis is:
a) Small volume

171
EMLE Trial Exams December 2020 (1000 Qs) Version (1.0) 22/12/2020

b) High specific gravity


c) Red cells in excess
d) Red cell casts
e) Proteinuria
The answer is: Red cell casts
24) Signs of rickets include all except:
a) Craniotabes
b) Delayed closure of anterior fontanelle
c) Delayed teething delayed walking
d) Bone deformities
e) Tall stature
The answer is: Tall stature
25) A 43-year-old woman presented to the General Surgical Outpatient department complaining of a lump
in her right groin which has been present for 1 year. She has a past medical history of asthma and is a
heavy smoker. On physical examination, a 3 × 4 cm blue non-tender, non-expansile mass is palpable
medial to the femoral pulse. It disappears on lying flat and there is an associated weak cough impulse
when the patient is standing. The presence of dilated tortuous veins in the long saphenous vein
distribution is also noted. From the following list, select the most appropriate diagnosis consistent with
this lump.
a) Femoral aneurysm
b) Saphenovarix
c) Lipoma
d) Femoral hernia
e) Inguinal lymph node
The answer is: Saphenovarix
26) In addition to irritability, sweating and difficulty in breathing with feeding. The symptom(s) that is most
indicative of congestive heart failure in a 3-week-old infant is:
a) Ascites
b) Cough
c) Cyanosis
d) Diminished feeding volume
e) Pretibial edema
The answer is: Diminished feeding volume
27) 20-year-old G1P0 patient is having a protracted active phase with irregular contractins that appear to be
of inadequate intensity. It is determined that the patient would benefit from oxytocin (pitocin)
augmentation. The patient is very nervous about this and asks a number of questions about the drug.
Which of the following is a characteristic of oxytocin?
a) half-life of about 8 minutes
b) prolonged effect
c) immediate hypertensive effect if given intravenously
d) inactivated by oxytocinase
e) inhibited by nonsteroidal anti-inflammatory agents
The answer is: inactivated by oxytocinase
28) A 27-year-old man presents to the GP practice having noticed a painless swelling of his right testicle. He
is otherwise well. On examination, the testis is enlarged, firm and has a nodular texture. What is the
most likely diagnosis?
a) Epididymal cyst

172
EMLE Trial Exams December 2020 (1000 Qs) Version (1.0) 22/12/2020

b) Gumma
c) Haematocele
d) Orchitis
e) Testicular cancer
The answer is: Testicular cancer
29) A 56-year – old male presents with generalized fatigue, weakness and vomiting. He tells you that he has
taken an overdose of his tricyclic antidepressant medication. His BP 150/90 mmHg, ECG demonstrates a
regular, wide-complex tachycardia. Which of the following is the most appropriate next step in
management?*
a) Cardioversion at 100 J
b) Lidocaine IV
c) Procainamide IV
d) Sodium Bicarbonate infusion
e) Propafenone
The answer is: Sodium Bicarbonate infusion
30) the term DKA (diabetic keto acidosis) means all of the following except:
a) Presence of ketone in the urine
b) Bicarbonate less than 15 mEq/L
c) PH less than 7.30
d) Decreased ketone bodies in blood
e) Serum glucose is more than 400 mg/D
The answer is: Decreased ketone bodies in blood
31) 47-years-old male presented to ER by acute abdominal pain & distension, vomiting and constipation for
4 days. The patient gives history of surgery 5 years ago for excision of abdominal mass which was of
benign nature on histopathological examination. Also there is history of similar manifestations occurred
twice 1 & 3 years after operation. X ray, U/S and CT were done which does not detect pathology except
dilated bowel loops. The etiology is most probably:
a) Recurrence of the tumour
b) Bowel ischemia
c) Adhesive intestinal obstruction
d) Cancer colon
e) Cancer stomach
The answer is: Adhesive intestinal obstruction
32) 55-year-old man with past medical history of tobacco abuse presents with shortness of breath. His vitals
are as follows Temperature 37, blood pressure 120/80, heart rate 120, O2 saturation 88%, respiratory
rate 33 On examination, lungs were clear to auscultation. CXR failed to show any infiltrates. Which is the
most likely treatment?#
a) Antibiotics
b) Steroids and bronchodilators
c) Observation
d) Anticoagulants
e) needle thoracostomy
The answer is: Anticoagulants
33) A new test was developed to diagnose COVID 19 virus in a cheaper and faster way than PCR. The
comparison between the new test and PCR is outlined in the following table. Please calculate the
positive predictive value of the new test?
a) 0.8

173
EMLE Trial Exams December 2020 (1000 Qs) Version (1.0) 22/12/2020

b) 0.6
c) 0.75
d) 0.2
e) 0.4
The answer is: 0.75
34) A 36-year – old man rescued from a building on fire presents with headache, vertigo, nausea, vomiting
and mild confusion (GCS 14) .He is without any evidence of burns or external injury . What is the most
appropriate next step in his management plan?*
a) Tight –fitting mask with 100% oxygen
b) 24% oxygen by nasal cannula
c) Hyperbaric oxygen in a hyperbaric chamber
d) Intubate and provide IPPV on 100% oxygen
e) Secure venous access
The answer is: Tight –fitting mask with 100% oxygen
35) Precocious puberty associated with bony dysplasia and café au lait spots on skin is noted in:
a) Laurence-Moon-Biedl syndrome
b) McCune-Albright syndrome
c) Alport’s syndrome
d) Frohlich’ssyndrome
e) None of the above
The answer is: McCune-Albright syndrome
36) A 65-year-old man is to undergo an elective sigmoid colectomy for malignancy. However, during the
clinic consultation he refuses blood products despite the risk of potentially requiring a post-operative
blood transfusion because he is a Jehovah’s Witness. His wife thinks that he should have a blood
transfusion if he needs it. What is the most appropriate course of action?
a) Obtain a court order
b) Obtain under the Mental Health Act
c) Respect the next of kin request
d) Respect the patient’s refusal for treatment
e) Withhold treatment until they change their view
The answer is: Respect the patient’s refusal for treatment
37) What is the best drug to give in variceal bleeding?*
a) Octreotide
b) Frusemide
c) Spironolactone
d) Indipamide
e) Propranolol
The answer is: Octreotide
38) Which of the following symptoms suggest a foreign body in the nasal passage?
a) Severe Pain and high fever
b) Clubbing
c) Mucopurulent, malodorous discharge
d) Crepitations
e) Wheezes
The answer is: Mucopurulent, malodorous discharge
39) About 12 days after a mild upper respiratory infection, a 12-year-old boy complains of weakness in his
lower extremities. Over several days, the weakness progresses to include his trunk. On physical

174
EMLE Trial Exams December 2020 (1000 Qs) Version (1.0) 22/12/2020

examination, he has the weakness described and no lower extremity deep tendon reflexes, muscle
atrophy, or pain. Spinal fluid studies are notable for elevated protein only. Which of the following is the
most likely diagnosis in this patient?
a) Bell palsy
b) Muscular dystrophy
c) Guillain-Barré syndrome
d) Cerebral palsy
e) Warding-Hoffmann disease
The answer is: Guillain-Barré syndrome
40) Hepatitis can turn chronic in:
a) Hepatitis A
b) Hepatitis E
c) Hepatitis B
d) Neonatal ideopahic hepatitis
e) Hepatitis caused by STORCH
The answer is: Hepatitis B
41) A 20-year- old man presents to emergency hospital after having severe injuries from a road traffic
accident. On presentation, he is restless, marked tachypnea, and has severe bone pain. His BP 70/45
mmHg, pulse 130 bpm. What is the most next appropriate step in his management plan?
a) Antibiotics
b) Analgesia
c) High flow oxygen
d) Secure venous access
e) Blood sampling for grouping and crossmatch
The answer is: High flow oxygen
42) You are called to see a 56-year-old man who is one day post appendicec-tomy because he became
acutely short of breath. He has just been given his first dose of cyclizine to relieve nausea. On arrival, the
patient is breath-less with the following observations: heart rate 122/min, blood pressure 86/48 mmHg
and saturations 85% in air. Which of the following would you administer first?
a) Adrenaline
b) Chlorphenamine
c) Fluids
d) Hydrocortisone
e) Salbutamol
The answer is: Adrenaline
43) Antenatal fetal monitoring can NOT be accomplished by:
a) Fetal kick chart.
b) Fetal scalp sampling.
c) Non-stress test.
d) Obstetric U/S & Biophysical profile.
e) Acoustic stimulation.
The answer is: Fetal scalp sampling.
44) Which one of the following conditions is least associated with pruritus?*
a) Pemphigus vulgaria
b) Chronic kidney disease
c) Iron deficiency anemia
d) Polycythemia

175
EMLE Trial Exams December 2020 (1000 Qs) Version (1.0) 22/12/2020

e) Scabies
The answer is: Pemphigus vulgaria
45) An Ultrasound in the 1st trimester of pregnancy is done for
a) Placental localization
b) Detecting of fetal weight
c) Assessment of amniotic fluid volume
d) Detection of fetal breathing
e) Dating of the pregnancy
The answer is: Dating of the pregnancy
46) A 21-year-old female patient arrives with her older sister after having hit her head on a night out. The
patient's sister is being openly flirtatious with you and asks for your phone number. You are aware you
feel attracted to her as well. What is the most appropriate action in this scenario?**
a) Excuse yourself for a moment, while you ask a registrar colleague for advice
b) Politely decline to give her your number, explaining it would be inappropriate to do
c) Give her your number as she is not your patient
d) Explain you cannot give her your number whilst you are working, but arrange to meet another
time outside of the hospital
e) Refuse to give her your phone number and ask hospital security to remove her from the hospital
premises
The answer is: Politely decline to give her your number, explaining it would be inappropriate to do
47) A 74-year old man presents with frontal headache, pain on jaw movement and weight loss. He had had a
similar problem 3 years earlier which had responded to prednisolone. Temporal artery biopsy is normal.
What is the most common cause of headache?
a) Brain tumor
b) Subarachinoid hemorrhage
c) Tension headache
d) Cluster headache
e) Migraine
The answer is: Tension headache
48) Complication of pregnancy that predispose to preterm labor includes all the following EXCEPT :
a) polyhydramnios
b) Urinary tract infection
c) 25 years old primigravida.
d) Premature rupture of membranes
e) Multiple pregnancy
The answer is: 25 years old primigravida.
49) Which of the following is NOT a presumptive symptom/sign of pregnancy:
a) Cessation of menstruation.
b) Quickening.
c) Nausea & vomiting.
d) Breast changes.
e) Darkening of the skin on the palms of the hands.
The answer is: Quickening.
50) Causes of urticaria are all except:
a) Parasitic infections
b) Food additives
c) Insect bites

176
EMLE Trial Exams December 2020 (1000 Qs) Version (1.0) 22/12/2020

d) Breast milk
e) Drugs
The answer is: Breast milk
51) A 67-year-old woman presents to the emergency department with a 1-month history of malaise, weight
loss, worsening cough and haemoptysis. You notice her saturations are 92% on air and perform a blood
gas test with the follow-ing results: pH 7.37, pO2 6.7, pCO2 7.4, bicarbonate 35, saturations 91.7%.
Reference ranges: pO2 >50mmHg, pCO2 55 mmHg, bicarbonate 22–28 mmol/L, pH 7.35–7.45. What
blood gas picture does this represent?*
a) Fully compensated metabolic acidosis
b) Fully compensated metabolic alkalosis
c) Fully compensated respiratory acidosis
d) Type I respiratory failure
e) Venous sample.
The answer is: Fully compensated respiratory acidosis
52) Individuals with impaired fasting glucose (IFG) are at increased risk of developing?
a) Type 1 DM
b) Type 2 DM
c) Pancreatitis
d) Hypothyroidism
e) cardiovascular complication
The answer is: Type 2 DM
53) Blunt liver trauma can be treated non surgically if#
a) No peritoneal signs
b) Low Grade injury on CT scan
c) Severe COPD
d) Haemodynamically stable
e) US confirms <500mls peritoneal fluid collection
The answer is: Haemodynamically stable
54) Which of the following quality measures is used for calculating Percentage of diabetic patients whose
HbAlc was measured in the past 6 months**
a) Structural
b) Process
c) Outcome
d) balancing
e) Non-specific
The answer is: Process
55) Following a sudden impact in an accident, the 34-year-old race car driver becomes unconscious and is
admitted to the hospital. A CT scan is performed, and a right space-occupying lesion is noted (Fig. 11–1).
What is the most likely diagnosis?
a) Corpus callosum injury
b) Pituitary apoplexia
c) Acute subdural hematoma
d) Acute epidural hematoma
e) Chronic subdural hematoma
The answer is: Acute subdural hematoma
56) A 27-year –old male is brought to the ED due to taking the entire bottle of propranolol because he has
been feeling depressed. The patient is somnolent, lightheadedness, Bp 65/45mmHg, pulse 44bpm, RR

177
EMLE Trial Exams December 2020 (1000 Qs) Version (1.0) 22/12/2020

19breath per minute, and SaO2 98% on room air. Rapidly aggressive IV hydration, IV atropine, as well as
glucagon, calcium and epinephrine drip were started. Which of the following is the best next step?
a) IV lipid emulsion therapy
b) Emergent hemodialysis
c) Intravenous milrinone infusion
d) High dose IV insulin infusion and IV glucose infusion
e) Intra-aortic balloon pump and temporary transvenous pacing
The answer is: High dose IV insulin infusion and IV glucose infusion
57) Which of the following is not a natural barrier to infection?*
a) Intact epithelial surface
b) High gastric pH
c) Antibodies
d) Natural killer cells
e) Macrophages.
The answer is: High gastric pH
58) A 16-year-old daughter refuses medication but her mother consents. What is the most appropriate
action to do?**
a) Write the prescription
b) Do not write the prescription
c) Postpone the prescription
d) Convince the daughter
e) Report the event to the police
The answer is: Write the prescription
59) Disinfection is a process in addition to cleaning. When is it necessary?*
a) Autonomy
b) Beneficence
c) Non-maleficence
d) Justice
e) Maleficence
The answer is: Non-maleficence
60) The INCORRECT statement regarding conservative surgical treatment of endometriosis
a) Involve restoration of the normal anatomy whenever possible.
b) Removal of the cyst with adjacent ovarian tissue may affect ovarian reserve
c) The aim is to get rid of all implants.
d) Endometriotic lesions could be dealt with by coagulation
e) Operative laparoscopy is suitable in most of the cases.
The answer is: The aim is to get rid of all implants.
61) A 22-year- old female presents with anticholinergic crisis .She is delirious, agitated, and requires
sedation. Which of the following medications would be the most appropriate to sedate this patient?
a) Chlorpomazine
b) Fluphenazine
c) Lorazepam
d) Etomidate
e) Ketamine
The answer is: Lorazepam

178
EMLE Trial Exams December 2020 (1000 Qs) Version (1.0) 22/12/2020

62) A 19-year-old man presents to the ENT outpatient clinic with a gradually increasing midline neck
swelling. On examination, the swelling is 1 x 1 cm, non-tender and immobile. It moves on protrusion of
the tongue. What is the most appropriate diagnosis?*
a) Goitre
b) Thyroglossal cyst
c) Pleomorphic adenoma
d) Cervical rib
e) Pharyngeal pouch
The answer is: Thyroglossal cyst
63) When evaluating a trauma patient with a focused assessment with sonography in trauma (FAST) scan,
which of the following views is ideally performed first?
a) Right flank
b) Left flank
c) Pericardial
d) Suprapubic
e) Thoracic
The answer is: Pericardial
64) A 23-year-old female with a history of sickle cell disease presents with fever, chills, cough, and dyspnea.
A chest x-ray demonstrates a focal infiltrate in the right lower lobe. Which of the following is the most
appropriate management at this time?
a) Heparin, intravenous fluids, and antibiotics
b) Tissue plasminogen activator to treat pulmonary veno-occlusion
c) Intravenous fluids and oxygen
d) Intravenous fluids, albuterol, prednisone, and antibiotics
e) Intravenous fluids and antibiotics alone
The answer is: Intravenous fluids and antibiotics alone
65) These reflexes are essential for successful breastfeeding except:#
a) Moro reflex
b) Suckling reflex
c) Swallowing reflex
d) Rooting reflex
e) Milk ejection reflex
The answer is: Moro reflex
66) The thyroid gland moves with deglutition due to:
a) Platysma
b) Cervical investing fascia
c) Pretracheal fascia
d) Thyrohyoid muscle
e) Pharyngeal muscles
The answer is: Pretracheal fascia
67) A 47 –year –old male with hypertension and hyperlipidemia presents to the emergency department with
a history of 10 minutes of right sided weakness that has just resolved. Which of the following should be
included in his treatment in the department before admission for further evaluation?
a) Clopidogrel
b) Ticlopidine
c) Heparin
d) Aspirin

179
EMLE Trial Exams December 2020 (1000 Qs) Version (1.0) 22/12/2020

e) Warfarin
The answer is: Aspirin
68) A 40-year-old male with history of type 2 diabetes to the outpatient clinic for routine visit. His
Hemoglobin A1C is 9.5%. He is taking metformin, glipizide and exenatide. What is the most likely next
step in treatment?
a) Diet and exercise only
b) Diet and exercise and increase metformin dose
c) increase Glipizide dose
d) Insulin
e) Metformin, Glipizide and exenatide
The answer is: Insulin
69) Which is the best study design to calculate incidence?*
a) Case report
b) Case series
c) Cross-sectional
d) Cohort
e) Case control
The answer is: Cohort
70) Modern management of preterm prelabor rupture of membranes at 32 weeks’ gestation includes
a) Prophylactic Antibiotics
b) Tocolysis till 37 weeks
c) Induction of labor at 34 weeks’ gestation
d) Cesarean delivery at 34 weeks’ gestation
e) None of the above
The answer is: Prophylactic Antibiotics
71) Which of the following causes pancytopenia?*
a) Parvovirus B19
b) HAV
c) HBV
d) HCV
e) Pneumococci
The answer is: Parvovirus B19
72) An 18-year-old single, sedentary, obese female (gravida 1, para 0) is first seen by you for prenatal care at
16 weeks’ gestation. Her history is unremarkable, and she claims to be in good health. Her dietary
history includes high carbohydrate intake with no fresh vegetables. Physical examination is within
normal limits except that she is somewhat pale. Suggested nutritional counseling should include which
of the following?
a) a strict diet to maintain her current weight
b) 25–30 g of protein in the diet everyday
c) intake of 1,200 calories a day
d) folic acid supplementation
e) at least 1 hour of vigorous aerobic exercise daily
The answer is: folic acid supplementation
73) A pregnant patient is receiving magnesium infusion for pre-eclampsia .Which of the following is the
earliest indicator of significant toxicity?
a) Diaphoresis
b) Loss of deep tendon reflexes

180
EMLE Trial Exams December 2020 (1000 Qs) Version (1.0) 22/12/2020

c) Slowed respiratory rate


d) Sinus tachycardia
e) Visual floaters
The answer is: Loss of deep tendon reflexes
74) What is the first line treatment of ulcerative colitis patient who complains of 3 motions of bloody
diarrhea per day?
a) IV steroid
b) Oral steroid
c) Rectal steroid
d) Oral azathioprine
e) Rectal ASA
The answer is: Rectal ASA
75) A 45-year-old business man presents to the ED with a sudden onset, severe right sided abdominal pain.
The pain is sharp and intermittent, and is felt worse in the right iliac fossa/right groin region. On
examination, his abdomen is soft. What is the most likely diagnosis?*
a) Ruptured ovarian cyst
b) Crohn’s disease
c) Acute appendicitis
d) Ureteric colic
e) Caecal adenocarcinoma
The answer is: Ureteric colic
76) Subnormal body temperature is a common finding in:
a) Mild ABO incompitability
b) Preterm babies
c) Full term babies
d) Physiological anemia
e) Physiological jaundice
The answer is: Preterm babies
77) A 59-year-old bank manager with a long history of indigestion presents with hematemesis, severe
constant epigastric pain, anorexia, and fatigue and weight loss. Which of the following is the most likely
cause for the clinical findings?
a) Angiodysplasia
b) Duodenal ulcer
c) Esophagitis
d) Gastric cancer
e) Gastric erosions
The answer is: Gastric cancer
78) You notice that you are finding it harder to read the notes at work, including when writing prescriptions.
You made a mistake earlier this week that almost resulted in a patient receiving the wrong dose of
gentamicin. There is a history of hereditary macular degeneration in your family, you are concerned you
may be developing symptoms. What should you do?***
a) Ignore it, you may lose your job if you tell someone
b) Buy some reading glasses online and hope for the best
c) Talk to your clinical/educational supervisor about your concerns
d) Do not take any action, hope that nobody will notice
e) Take time off work, phone in with the flu
The answer is: Talk to your clinical/educational supervisor about your concerns

181
EMLE Trial Exams December 2020 (1000 Qs) Version (1.0) 22/12/2020

79) A 75-year-old male presents with a fall on an outstretched hand. Radiographs demonstrate a distal
radius fracture with dorsal displacement of the distal segment. Which of the following is the most likely
nerve injury?
a) Median
b) Radial
c) Ulnar
d) Axillary
e) Vagus
The answer is: Median
80) A 45-year-old business man presents to the ED with a sudden onset, severe right sided abdominal pain.
The pain is sharp and intermittent, and is felt worse in the right iliac fossa/right groin region. On
examination, his abdomen is soft. What is the most likely diagnosis?*
a) Ruptured ovarian cyst
b) Crohn’s disease
c) Acute appendicitis
d) Ureteric colic
e) Caecal adenocarcinoma
The answer is: Ureteric colic
81) You are seeing a 25-year-old woman (gravida 1, para 0) with sickle cell disease at 12 weeks’ gestation for
her first prenatal visit. Suggestions for her care should include which of the following?
a) folic acid
b) transfusions of fresh hemoglobin for hematocrit less than 25%
c) antibiotic prophylaxis to prevent UTI
d) oral iron in double the usual dosage (650 mg tid)
e) delivery at 36 weeks after documentation of fetal lung maturity
The answer is: folic acid
82) A 2-year-old boy who presents with weight loss and lethargy. On examination he has splenomegaly. You
are worried that he has leukemia. You perform an urgent full blood count. What are the most likely
hematology results if he has leukemia?
a) Low hemoglobin and low platelets
b) High hemoglobin and low platelets
c) Normal hemoglobin and normal platelets
d) High hemoglobin and raised platelets
e) Low hemoglobin and raised platelets
The answer is: Low hemoglobin and low platelets
83) A 26-year-old pregnant woman at 8 weeks presents to the ED complaint from nausea and vomiting. Her
abdomen and pelvic examinations are normal and her ultrasonogram reveals a normal live intrauterine
pregnancy at 8 weeks. Her urine reveals no ketones. Which of the following is the best recommendation
for reducing her symptoms?
a) Diazepam
b) Promethazine
c) Vitamin B6
d) Ondansetron
e) Dexamethasone
The answer is: Vitamin B6
84) A blind-ending epithelialized tract, which may lead to a cavity. What is the most likely surgical
pathology?*

182
EMLE Trial Exams December 2020 (1000 Qs) Version (1.0) 22/12/2020

a) Fissure
b) Hernia
c) Sinus
d) Seroma
e) Abscess
The answer is: Sinus
85) A 24-year-old woman G0 married for 5 months presents with a non-tender cystic mass in her right vulva
that causes some discomfort when walking and during coitus. The mass was at the posterior part of
labium major and was about 2.0 X 2.0 cm dimensions. What is the MOST APPROPRIATE initial decision?#
a) Marsupialization
b) Administration of antibiotics
c) Surgical excision
d) Incision and drainage
e) Observation
The answer is: Marsupialization
86) Premature rupture of membrane is most strictly defined as rupture at any time prior to:
a) A stage of fetal viability.
b) The 2ND stage of labor.
c) The 32ND week of gestation.
d) The onset of labor.
e) The 38TH week of gestation.
The answer is: The onset of labor.
87) The following are known causes of preterm labor EXCEPT:
a) Maternal hypoxia
b) Intrauterine fetal death
c) Polyhydramnios
d) Multiple pregnancy
e) Cervical incompetence
The answer is: Maternal hypoxia
88) A 63-year-old male with a history of multiple myeloma presents to ICU with acute onset of extreme
weakness, blurry vision, considerable mucosal bleeding and severe headache .A blood sample was sent
for CBC and chemistry panel, but the laboratory is unable to deal with the sample, citing “inappropriate
blood sample “.Which of the following is the most appropriate emergent definitive management?
a) Hemodialysis
b) Plasmapheresis
c) Platelet transfusion
d) Packed red blood cell transfusion
e) Erythropoietin
The answer is: Plasmapheresis
89) Which of the following statements about surgical site infections (SSIs) are true?
a) Infection in the musculofascial tissues is known as deep SSI.
b) The patient may have systemic signs in a minor SSI.
c) Infection causing delay in hospital discharge is a major SSI.
d) The differentiation between major and minor SSIs is not important.
e) Surveillance for surgical site infection should be done for a year after implanted joint surgery.
The answer is: The patient may have systemic signs in a minor SSI.

183
EMLE Trial Exams December 2020 (1000 Qs) Version (1.0) 22/12/2020

90) A 28-year-old professional football player has sudden pain and swelling in the right groin when
attempting to intercept a pass. He is admitted to the local emergency department. On examination,
there is a tender swelling in the right groin. The scrotum and penis show no abnormality. What is the
next step in management?*
a) Needle aspiration to exclude hematoma
b) Forceful manual reduction
c) Laparotomy within 20 minutes
d) Preoperative preparation and exploration of the groin with hernia repair
e) Morphine and re-evaluation within 12 hours
The answer is: Preoperative preparation and exploration of the groin with hernia repair
91) What is the best drug to be given to a 42-year-old diabetic patient came to the hospital complaining of
periorbital oedema and his lab investigations showed high urine proteins?
a) Spironolactone
b) Indapamide
c) Metoprolol
d) Bumetanide
e) Enalapril
The answer is: Enalapril
92) A 23-year-old woman consults a physician because she has just learned that her husband has chronic
hepatitis B infection. She is not clinically ill, but she is very worried that she may have been exposed to
the virus. Which of the following offers evidence that she has been infected?
a) Anti-HBc
b) Anti-HBe
c) Anti-HBs
d) Anti-HDV
e) HBs Ag
The answer is: HBs Ag
93) A 64-year-old woman presents with several episodes of profuse dark per rectal bleeding. She has
experienced intermittent symptoms of central abdominal discomfort and bloating for the past 2 years,
and admits to being prone to constipation. She undergoes a single contrast barium enema, which is
shown below. What is the diagnosis?*
a) Familial adenomatous polyposis coli
b) Crohn’s disease
c) Diverticular disease
d) Adenocarcinoma
e) Peutz–Jeghers syndrome
The answer is: Diverticular disease
94) You notice that you are finding it harder to read the notes at work, including when writing prescriptions.
You made a mistake earlier this week that almost resulted in a patient receiving the wrong dose of
gentamicin. There is a history of hereditary macular degeneration in your family, you are concerned you
may be developing symptoms. What should you do?***
a) Ignore it, you may lose your job if you tell someone
b) Buy some reading glasses online and hope for the best
c) Talk to your clinical/educational supervisor about your concerns
d) Do not take any action, hope that nobody will notice
e) Take time off work, phone in with the flu
The answer is: Talk to your clinical/educational supervisor about your concerns

184
EMLE Trial Exams December 2020 (1000 Qs) Version (1.0) 22/12/2020

95) The pathophysiology of preeclampsia is characterized by:


a) Vasospasm
b) Hypervolemia
c) Thromboxane is decreased.
d) Hemodilution
e) Vasodilatation followed by vasospasm
The answer is: Vasospasm
96) A biophysical profile includes all of the following assessment parameters EXCEPT:
a) Fetal movement.
b) Fetal weight.
c) Fetal tone.
d) Fetal breathing movements.
e) Amniotic fluid volume.
The answer is: Fetal weight.
97) Mark the incorrect statement:#
a) Bronchial breathing and consonating crepitation are auscultated in pneumonia
b) The chest can be silent on auscultation if the attack of asthma is severe
c) Bronchiolitis has the peak incidence at 6 years of age
d) Clubbing of fingers usually occurs in cases of bronchiectasis
e) Caused by RSV
The answer is: Bronchiolitis has the peak incidence at 6 years of age
98) What is the most appropriate first-line treatment for myxedema coma?#
a) Adrenaline and levothyroxine
b) Prednisolone and levothyroxine
c) Hydrocortisone and levothyroxine
d) Hydrocortisone and fludrocortisone
e) Adrenaline and hydrocortisone
The answer is: Hydrocortisone and levothyroxine
99) Hepatitis can be the result of all except:
a) Overactive immune system
b) Drugs
c) Chemicals
d) Environmental toxins
e) Gilbert
The answer is: Gilbert
100) A 10-year-old girl moves into your catchment area and presents to your clinic saying she has been
monitored in the past for her chronic renal failure from reflux nephropathy. You advise her of symptoms
that can be result of chronic renal disease and arrange appropriate monitoring. Which of the following is
not seen in chronic renal disease?
a) Anaemia
b) Hypertension
c) Hypocalcaemia
d) Hypokalaemia
e) Short stature
The answer is: Hypokalemia

185

You might also like